You are on page 1of 95

CRIMINAL LAW OUTLINE I. Foundations of Criminal Law A. Crime violation of statute, anything that causes social harm B.

. Punishment 1. D may not be punished twice for the same crime 2. D may not be punished retroactively due process 3. No cruel and unusual punishment C. Proportionality 1. Force is not justified unless proportional to the threat 2. Punishment is not justified unless proportional to the crime D. Sentencing 1. Indeterminate judge imposes sentence upon convicted person using bounds of written statutes, parole board has power to change sentence 2. Determinate judges sentence stands (18 years = 18 years) most jurisdictions WI follows E. PRINCIPLE OF LEGALITY roots of due process cannot be convicted of crime that doesnt exist 1. No crime without law, no punishment without law 2. Not a rule of law, but an ideal 3. There will be no more common law if an offense has not yet been defined, it will not be considered a crime 4. Majority rule 5. Gives person opportunity to conform his conduct to the law 6. Due Process courts prohibited from making retroactive laws or from changing current laws in order to obtain conviction 7. Due Process forbids enforcement of any law which rests virtually complete discretion in the hands of police to determine whether D has satisfied its requirements F. Specificity 1. Law must be passed by the legislature 2. It must define the conduct prohibited and thus provide notice constructive notice 3. D is denied due process if convicted under statute which lacks clarity 4. A statute or ordinance must establish minimal guidelines to govern law enforcement 5. Rule of Lenity: Court must give statute its plain and definite meaning; if ambiguous, court must ascertain intent of legislature 6. MPC does not recognize lenity; law will be interpreted in way which will further the purposes of the code G. Sources of Law 1. Common Law Common law is judge-made law. Even when superseded by statutory law, common law may serve to interpret ambiguous statutory terms. 2. Criminal Statutes Today, statutory law is the prevailing source of criminal law and essentially has replaced common law. Although most states have abolished common law crimes, a few have enacted reception statutes, expressly recognizing common law offenses when statutory law does not provide a punishment for such offense. In effect, such a statute receives the common law offenses in place at the time of the statutes enactment. a. Generally speaking, statutory law classifies a crime as a felony or a misdemeanor, both of which may be subdivided into degrees. A felony is punishable by death or imprisonment in a state or federal prison. The maximum punishment for a misdemeanor is a monetary fine, incarceration in a local jail, or both. Some jurisdictions also have an additional classification of violation or infraction for which only a monetary fine is authorized. 3. Model Penal Code Although the Code published by the American Law Institute is not the law in any jurisdiction, it stimulated adoption of revised penal codes in at least

thirty-seven states. Although some state legislatures have adopted only small portions of the Model Code as their own, other jurisdictions (including New Jersey, New York, Pennsylvania, and Oregon) have enacted many of its provisions. Courts, on their own, sometimes turn to the Model Code and its supporting commentaries for guidance in interpreting non-Code criminal statutes. H. Criminal Procedure 1. 1st Phase a. decision by prosecutor to charge someone b. files a criminal complaint which lists charges 2. 2nd Phase a. testing of prosecutors decision by someone other than him (Judge or grand jury) b. preliminary hearing showing probably cause for indictment could a reasonable jury convict under this evidence 3. 3rd Phase a. Discovery exchange of info between parties big part of case b. Motions are litigated (e.g. illegal entry? Miranda warnings?) c. Plea bargaining/negotiating (90% disposed by guilty plea) 4. 4th Phase Trial a. jury selection b. opening statements c. state case-in-brief d. defense may ask for motion to dismiss if prosecution has not provided enough evidence e. Defense case-in-chief f. Rebuttal cases g. Defense may ask for another motion to dismiss h. Jury instruction conference what law should be given to jury? i. Closing arguments j. Jury instructions k. Deliberation 5. 5th Phase a. sentencing of D determinant and indeterminate 6. Appeal? Further proceedings? I. Elements of Offenses Statutory evaluation 1. Defined as essential ingredients of the offense via statute - know all elements, penalty is not an element 2. May be difficult to define, may have their own definitions 3. Penalty for offense is not an element (guilty of Class A misdemeanor) 4. Penalty Enhancers sometimes crimes are committed under aggravated circumstances and thus deserve more severe penalties (enhancer is an element of the offense) a. Must be determined by a jury b. Exception habitual criminality (repeater statute), can be determined by a judge c. Ex. Selling drugs in a city park here city park is enhancer J. Elements of Defenses 1. Negative yes you did, no I didnt directly attack elements of crime 2. Affirmative youre right, I did, however admit elements and introduce some other reason for innocence (self-defense, insanity) K. Burdens of Proof -The fact-finding process imposes two types of burdens of proof: (1) the burden of production (sometimes called the burden of going forward (with evidence)); and (2) the burden of persuasion. 1. Burden of Production a. Prosecution Burden of Production Prior to trial the prosecution must file a document with the court that indicates the crime or crimes it believes that the defendant has committed. This document provides the accused with notice of the essential elements of the offense(s) charged, and the basic facts that the

prosecutor intends to prove at trial to support his allegation that the defendant committed the crime(s). The prosecutor must produce enough evidence that a rational trier-of-fact may fairly determine that the elements of the crime have been proved beyond a reasonable doubt. i. If the judge concludes that the prosecutor failed to satisfy the burden of production regarding any element of the offense charged, the defendant is entitled to a directed verdict of acquittal at the conclusion of the prosecutors case-in-chief or at the end of the trial. If the prosecutor failed to introduce enough evidence to support a jury finding beyond a reasonable doubt that the defendant committed the crime, there is no reason for it to deliberate on the matter. b. Defendants Burden of Production The defendant is sometimes required to provide advance notice to the prosecution of defenses he intends to assert at trial. The amount of evidence required to satisfy the burden of production on affirmative defenses varies by jurisdictions. In some jurisdictions the defendant meets his burden of production (and, thus, is entitled to an instruction to the jury on the defense) if he produces more than a scintilla of evidence regarding an affirmative defense; in other jurisdictions the defendant must introduce enough evidence to raise a reasonable doubt on the issue of the defense claimed. i. If the defendant fails to meet his burden of production regarding an affirmative defense, the judge will not instruct the jury on the law pertaining to the defense, and the defendant is not entitled to have the issue considered by the jury in its deliberations. 2. Burden of Persuasion Once a party satisfies his burden of production pertaining to an issue, that matter is properly before the jury as fact-finder, i.e., it will decide whose factual claims are more persuasive. a. Prosecutions Burden of Persuasion (the Winship doctrine) Pursuant to the due process clause, a person charged with a crime is presumed innocent and, to enforce this presumption, the Supreme Court held in In re Winship that the prosecution must persuade the fact-finder beyond a reasonable doubt of every fact necessary to constitute the crime charged. This rule has come to be known as the Winship doctrine. i. If the prosecution fails to meet its burden of persuasion, the defendant must be acquitted. Procedurally, the acquittal may occur in either of two ways. First, after the prosecution completes its presentation of evidence or immediately before the case is due to be submitted to the jury, upon motion of the defendant, the trial court must direct a verdict of acquittal if the evidence, viewed in the manner most favorable to the prosecution, can support no reasonable verdict other than acquittal. Alternatively, if the judge believes that reasonable minds can differ and, therefore, permits the case to go to the jury, the jury must acquit if it possesses a reasonable doubt regarding one or more elements of the offense charged. b. Defendants Burden of Persuasion Jurisdictions differ in their allocation of the burden of persuasion regarding affirmative defenses. Some states require the prosecution to disprove beyond a reasonable doubt some or all defenses, once the defendant has met his burden of production. In states that allocate to the defendant the burden of persuasion regarding defenses, it is typical to require the defendant to prove the validity of the claimed defense by the less strict preponderance-of-the-evidence standard. i. If a defendant presents sufficient evidence to meet his burden of production regarding a defense to the crime charged, the jury must be permitted to evaluate the defense claimed. When the defendant also has the burden of persuasion, a jury should reject the claimed defense if he fails to satisfy the stated burden.

If the prosecution has the burden of disproving a defense, the jury must acquit the defendant if the prosecution fails to persuade the jury beyond a reasonable doubt of the defenses non-existence. 3. Determining if a Fact Relates to an Element or an Affirmative Defense A prosecutor must prove every element of an offense beyond a reasonable doubt. The legislature may allocate to the defendant the burden of persuasion regarding facts not formally identified as elements of the offense charged. Thus, generally speaking, a legislature may allocate to the defendant the burden of persuasion regarding facts that relate to an affirmative defense. However, a court interpretation of the statute may be required to determine whether a particular fact relates to an element of an offense or to an affirmative defense. 4. Model Penal Code Except for defenses that the Code expressly requires the defendant to prove with the standard of proof being a preponderance of the evidence the prosecution must prove every element of an offense beyond a reasonable doubt, [MPC 1.12(1)] including conduct that negates an excuse or justification for the action. [MPC 1.13(9)(c)] That is, the Model Penal Code allocates to the prosecution the duty to disprove defenses, assuming that the defendant has satisfied his burden of production. L. Historical Background 1. Prasad: Common Perspective a. Poor are coddled-facts dont bear this out b. People work harder for less money c. Gap between rich and poor has grown d. Poor supplement income in extra-legal ways e. Corporate incentives and economic conditions contribute to this difficulty 2. Ruth & Reitz: Criminal Justice System a. Evolution from the institutions of the 19th century i. Everything began small and uncontrolled, then became really big and was hyper controlled by the government, and now its shifting again toward more privatization b. Constant belief that crime is more out of control than it actually is (except for 1950-1970s when homicides were actually high) c. Foucault-Ian discipline i. Technology tends to submit people to its will ii. Controls are used to manage people 3. Wilson (who is now not really followed) a. Criminals make choices to commit crimes and are accountable for their actions b. People that commit crimes are largely wicked i. Acknowledges that some are not, but proceeds in his argument as if all are c. Rehabilitation does not work d. Liberal Perspective was blind to victims feelings and rights e. Attempted to argue that there was no correlation between poverty and crime rate, also that handouts to the poor increase the crime rate f. Ignored social conditions that caused crime to rise i. Poor schooling, family breakdown, etc. 4. Garland a. 2 ways of controlling people: i. Formal controls (police, etc.) ii. Informal controls (Foucault discipline), run by more private companies a. May be better at controlling crime, and theyre much cheaper b. Criminal law is a field without a theory c. We get used to the way things are and I dont see a need for a change in the CJS d. Urges us to listen to people who dont have someone speaking for them e. Decline of the rehabilitative ideal, reemergence of just deserts ideals through punitive statements f. Criminals are seen as predatory, brilliant and wicked

c.

Return of the victim with a twist: Is now a zero sum gain: any niceness given to either side (victim or criminal) must been given to the other side, otherwise it seems unfair h. Crime control is politicized rather than left to experts i. The philosophy is that criminals act because they can because there are no social control preventing them from acting M. Purposes of Punishment 1. Utilitarianism a. Deterrence The utilitarian theory is essentially one of deterrence punishment is justifiable if, but only if, it is expected to result in a reduction of crime. Punishment must be proportional to the crime, i.e., that punishment be inflicted in the amount required (but no more than is required) to satisfy utilitarian crime prevention goals. i. Utilitarians consider the effect of a form of punishment in terms of both general deterrence and specific (or individual) deterrence. When the goal is general deterrence, punishment is imposed in order to dissuade the community at large to forego criminal conduct in the future. When the goal is specific deterrence, punishment is meant to deter future misconduct by an individual defendant by both preventing him from committing crimes against society during the period of his incarceration (incapacitation), and reinforcing to him the consequences of future crimes (intimidation). b. Rehabilitation Another form of utilitarianism is rehabilitation (or reform). Examples of rehabilitative punishment include: psychiatric care, therapy for drug addiction, or academic or vocational training. 2. Retributivism Under a retributive theory of penal law, a convicted defendant is punished simply because he deserves it. There is no exterior motive such as deterring others from crime or protecting society here the goal is to make the defendant suffer in order to pay for his crime. Retributive theory assigns punishment on a proportional basis so that crimes that cause greater harm or are committed with a higher degree of culpability (e.g., intentional versus negligent) receive more severe punishment than lesser criminal activity 3. Denunciation (Expressive Theory) The denunciation theory which holds that punishment is justified as a means of expressing societys condemnation of a crime has both utilitarian and retributive components. Under a utilitarian theory, denunciation is desirable because it educates individuals that the community considers specific conduct improper, channels community anger away from personal vengeance, and serves to maintain social cohesion. Under a retributive theory, denunciation serves to punish the defendant by stigmatizing him. N. Presumptions 1. Common Law Mandatory Presumptions If a jury is instructed that it must presume Fact B upon proof of basic Fact A, the presumption is a mandatory presumption a. Rebuttable Mandatory Presumptions A mandatory rebuttable presumption requires a finding of the presumed fact upon proof of the basic fact, unless that finding is rebutted by the opposing party. The procedural effect of a mandatory rebuttable presumption is to shift to the defendant the burden of persuasion regarding the presumed fact, upon proof by the prosecution of the basic fact. Rebuttable mandatory presumptions are unconstitutional when the presumed fact is an element of the crime charged. b. Irrebuttable (Conclusive) Presumptions An irrebuttable or conclusive presumption requires the jury to find the presumed fact upon proof of the basic fact, even if the opposing party introduces rebutting evidence. (NB: True irrebuttable presumptions are rare.) A mandatory irrebuttable presumption pertaining to an element of an offense is unconstitutional for the same reasons that mandatory rebuttable ones are impermissible.

g.

2. Common Law Permissive Presumptions (Inferences) A permissive presumption or inference is one in which the fact-finder may, but need not, find the existence of the presumed fact upon proof of the basic fact. As such, it does not formally shift the burden of proof from one party to another, as a true mandatory presumption does. While not unconstitutional per se, to be constitutionally permissible, there must be a rational connection between the basic fact and the presumed (inferred) fact; i.e., the presumed fact more likely than not flows from the basic fact. 3. Model Penal Code The Model Penal Code [ 1.12(5)(b)] does not recognize mandatory presumptions but permits permissive presumptions regarding elements of an offense. II. Actus Reus A. Book Notes 1. Criminal punishment requires that the individual have committed an act, and that the actus reus (criminal or bad act) is a necessary element in the equation establishing criminal liability 2. TF: There can be no criminal liability unless D has engaged in some conduct or has failed to engage in conduct when the law requires him/her to do so (conduct, possession, omission) 3. There are three components: Voluntary Act (or omission), Cause and Social Harm 4. Two requirements a. Requirement of an act b. Prove beyond a reasonable doubt in order to establish liability for a particular crime 5. Assumption: Human beings are capable of critically evaluating their own desires and resisting them if, on reflection, it appears that pursuing those desires would be wrong a. Punishable crime, should be confined to failures of self-control 6. Punishment must be for some conduct-either action or failure to act 7. Punishment must be conditioned on inherently bad conduct that causes or threatens harm to some important social interest 8. Punishment must be for a PAST ACT 9. VOLUNTARY ACT a. Subject to limited exceptions, a person is not guilty of a crime unless his conduct includes a voluntary act b. Act- An act involves physical behavior. It does not include the mental processes of planning or thinking about the physical act that gives rise to the criminal activity (such is the domain of mens rea). c. Voluntary- In the context of actus reus, voluntary may be defined simply as any volitional movement. Habitual conduct even if the defendant is unaware of what he is doing at the time may still be deemed voluntary. Acts deemed involuntary may include: spasms, seizures, and bodily movements while unconscious or asleep. d. Burden of Proof: Although a defendant may raise as a defense that his conduct was not voluntary, the voluntariness of an act proscribed by criminal law is in fact an element of the crime, and as such, the prosecution bears the burden of proving such fact. i. The prosecution does not need to show, however, that every act was voluntary in order to establish culpability. It is sufficient that the defendants conduct which is the actual and proximate cause of the social harm included a voluntary act. e. Constitutional Law: A state may not dispense with the criminal law requirement of an actus reus. That is, the government may not punish a person for his thoughts alone or for his mere propensity to commit crimes. The special rule of omissions aside, some conduct by the defendant is constitutionally required. i. Robinson v. California- Robinson was convicted under a California statute that made it an offense for a person to be addicted to the use of

narcotics. The Supreme Court struck down the statute on Eighth and Fourteenth Amendment grounds. Essentially, the Court held that, although a legislature may use criminal sanctions against specific acts associated with narcotics addiction, e.g., the unauthorized manufacture, sale, purchase, or possession of narcotics, it could not criminalize the status of being an addict, which the Court analogized to other illnesses. ii. Powell v. Texas- Powell was charged with violating a Texas statute that prohibited drunkenness in a public place. Powell argued that he was a chronic alcoholic and was thus unable to prevent appearing drunk in public and sought relief under the reasoning of Robinson. The Court upheld his conviction, distinguishing the case from Robinson on the ground that Powell was being punished for the act of public drunkenness and not for his status as a chronic alcoholic. f. Model Penal Code- Similar to the common law, MPC 2.01 requires that criminal conduct include a voluntary act. It does not define the term voluntary, but Comments list bodily movements that are involuntary: reflexes, convulsions, conduct during unconsciousness, sleep, or due to hypnosis, as well as any conduct that is not a product of the effort or determination of the defendant, either conscious or habitual. Excluded from the requirement that the act be voluntary are offenses that constitute a violation [2.05], defined as an offense for which the maximum penalty is a fine or civil penalty. 10. Voluntary conduct requirement as a way of ensuring equal statue before the law a. Autonomy b. Privacy c. Equality 11. Specific acts must be stated in advance that they are criminal a. Criminal law must have consent of people subject to it 12. The requirement of act may refer to some or all of these following conditions for just punishment a. Past b. Voluntary c. Wrongful or potential harmful d. Conduct e. Specified f. In advance g. By statute B. The need for an Actus Reus 1. Proctor v. State a. Facts: The plaintiff was convicted of owning a 2 story building with the intent and for the purpose of selling liquor unlawfully. P brings this case because he challenges the statute that he was convicted of violating that makes a simple intention a crime. b. Rule: Does NOT include punishment of thoughts; there must be some overt act that is proven beyond the intention to make something illegal. It fails to connect bad intent with an overt act. C. Omissions 1. Common Law-Subject to a few exceptions, a person has no legal duty to act in order to prevent harm to another. The criminal law distinguishes between an act that affirmatively causes harm, and the failure of a bystander to take measures to prevent harm 2. Exceptions-A failure to act is punishable when there is a LEGAL DUTY TO ACT a. Jones v. US: i. Facts: Defendant Jones was found guilty of involuntary manslaughter of his friends 10-month-old baby where he failed to provide for the child and such failure resulted in the childs death.

Rule: Where a penal statute does not impose a legal duty to perform a particular action, criminal liability for the omission of such action only arises where legal duty is imposed by some other law; not a moral obligation 3. Proximate Cause-Must happen right before something is going to happen 4. Common-Law Duties to Act a. Duty based on special relationship- One may have a common law duty to act to prevent harm to another if he stands in a special status relationship to the person in peril. Such a relationship is usually founded on the dependence of one party to the other e.g., a parent to his minor child or on their interdependence e.g., spouses. b. Duty based on contractual obligation- A duty to act may be created by implied or express contract. E.g., a person who undertakes the care of a mentally or physically disabled person and fails to do so may be found criminally liable based on omission for his wards injury or death. c. Duty based on creation of risk- A person who harms another or places a person in jeopardy of harm, or who damages property, even if unintentionally, has a common law duty to render assistance. E.g., one who accidentally starts a house fire may be convicted of arson if he fails to extinguish the fire or take other steps to prevent or mitigate the damage. As another example, there is a split of authority regarding whether one who justifiably shoots an aggressor in selfdefense has a subsequent duty to obtain medical attention for the wounded aggressor. d. Duty based on voluntary assistance- One who voluntarily renders assistance to another already in danger has a duty to continue to provide aid, at least if the subsequent omission would put the victim in a worse position than if the defendant had not commenced the assistance at all. e. Duty based on statute- Some duties are statutorily imposed, e.g., a driver involved in an accident must stop his car at the scene; parents must provide food and shelter to their minor children. A few states have enacted so-called Bad Samaritan laws, which make it an offense (usually a misdemeanor) for a person to fail to come to the aid of a person in need under specified circumstances. 5. Model Penal Code- The Model Penal Code is consistent with the common law regarding omissions. Liability based on an omission may be found in two circumstances: a. (1) if the law defining the offense provides for it; or b. (2) If the duty to act is otherwise imposed by law. c. The latter category incorporates duties arising under civil law, such as torts or contract law. 6. Wis. Statute 940.34-Duty to aid victim or report crime a. Any person who knows that a crime is being committed and that a victim is exposed to bodily harm shall summon law enforcement officers or other assistance or shall provide assistance to the victim b. WI- babysitter does have a duty to a child to protect it(948.04) c. Duty to aid victim statuteWI is only state to have. Must call cops or give aid unless someone else is helping.(940.32) d. A person need not comply with this subsection if any of the following apply i. Compliance would place him or her in danger ii. Compliance would interfere with the duties the person owes to others iii. In the circumstances, assistance is being summoned or pvodie4d by others. D. Possession 1. MPC-Possession is an actif the possessor knowingly procured or received the thing possessed or was aware of his control thereof for a sufficient period to have been able to terminate his possession a. Does not require the D to act, only passive possession of prohibited objects

ii.

2. 3. 4. 5.

Not necessary to own the object to possess it Defense of mistaking one object for another will not work Actual Possession-Prohibited object is actually on the person Constructive Possession- the power and intention to exercise control, or dominion and control, over an object not in one's "actual" possession (can be hidden and still have constructive possession) a. Know location b. Intent to exercise dominance and control c. Ability to exercise dominance and control i. United States. v. Maldonado ii. Facts: Santos, working with the police who were trying to track the drugs to their destination, was given packages of cocaine, and was instructed to deliver them to someone named Palestino. When he went to the hotel and asked for Palestino, Zavala (Maldonado) appeared and gestured to Santos to follow him to the hotel room. Zavala them placed several calls, supposedly to Palestino to have him come to the hotel. Palestino still didnt arrive, and Santos asked if they could leave the room for a soda. Then Santos placed the cocaine in the room, in a closet or dresser, and they both left. The police finally arrested Zavala and he was convicted of possession of cocaine with intent to distribute. Zavala appealed, arguing that there was insufficient evidence to prove he was in possession. There was no evidence he ever touched the cocaine, or even saw the cocaine in the bag, that he was ever in the room alone with the cocaine, or that he ever had a practical opportunity to remove it from the room. iii. So because the drugs were left in Zavala's room, while he was waiting for Palestino to come pay for them and with Zavala's consent and knowledge, the Court held that they were in his possession. Also, just because the hotel room was under Palestino's name, Zavala was still occupying it, and could easily return there at will. d. Proximity and Control i. US v. Jenkins a. Facts-D challenged sufficiency of evidence proving his constructive possession of drugs found on a coffee table in an acquaintances apartment in close proximity to the D b. Rule: Dominion and control are not established by mere proximity to the drug, or mere presence on the property where it is located or mere association with the person who does control the drug of property c. Evidence that can establish dominion & control includes 1. D attempting to hide or destroy contraband 2. D lying about identity or large amounts of cash on his person d. Juries are allowed to evaluate the surrounding circumstances and determine whether the D was otherwise exercising dominion or control over the item e. Location of Possession i. State v. Lewis-There must be some connection between the drugs and zone to permit a reasonable inference that D constructively possessed the drugs with intent to distribute them within the drug-free zone. f. Intercepted Possession i. State v. Clark-Police intercepted drugs and delivered a much smaller amount. D was not charged with constructive possession b/c they did not find evidence of dominion and control over package

Duration of Possession i. Length of possession is distinguished by the object being possessed ii. Us. V. Lane a. Felon holding a gun is a factor in itself indicating control b. Possession must be done voluntary and intentionally and not because of any mistake or accident h. Dominion and Control over images i. U.S. v. Tucker-Held that D had control over pornographic images a. Ability to destroy is definitive evidence of control 6. MPC: Possession is an act if the possessor either a. Knowingly obtained the object possessed b. Or knew she was in control of it for a sufficient period to have been able to terminate possession 7. WI: Possession occurs when the D has the object in an area over which he exercises control and the D intends to exercise control over the object (pot in trunk) E. Requirement of Harm-Defined as the negation, endangering, or destruction of an interest that is socially valuable 1. Elements of a Social Harm- The social harm of an offense, as defined by statute or at common law, may consist of wrongful conduct, wrongful results, or both. Moreover, the offense will contain so-called attendant circumstance elements. a. Conduct Elements- Some crimes establish social harm in terms of conduct, irrespective of any harmful results, e.g., driving under the influence of alcohol (do not require result) b. Result Elements- An offense may be defined in terms of a prohibited result. For example, murder is a result crime, because the social harm is the death of another human being, irrespective of the nature of the conduct that resulted in such death (e.g., whether the death occurred by shooting, stabbing, or poisoning) (does not matter how results occurs) c. Combined Conduct and Result Elements- Some offenses contain both conduct and result elements. For example, a statute may define first-degree murder as the killing of another human (the result) by means of a destructive device or explosive (the conduct). d. Circumstance Elements- An attendance circumstance is a fact or condition that must be present at the time the defendant engages in the prohibited conduct and/or causes the prohibited result that constitutes the social harm of the offense. Often an attendant circumstance is an element of the offense, e.g., the crime of burglary the breaking and entering of the dwelling house of another at nighttime contains an elemental attendant circumstance that the crime must occur at night i. Included in the definition of a particular offense may be one or more conditions that must be present during the prohibited conduct, or must be part of the prohibited result, in order for actor to be guilty of the crime. 2. Constitutional Due Process Limitsa. Various constitutional provisions limit the extent to which a legislature may proscribe social harm b. For example, the First Amendment bars a state from criminalizing most forms of speech. Even where some social harm may occur such as some persons may find a given form of speech offensive the law deems that the integrity of constitutional rights outweighs the societys interest in preventing the harm. c. Privacy rights, such as reproductive choice and sexual conduct of consenting adults, have also been protected from state attempts to criminalize such conduct 3. Lawrence v. Texas a. Facts: Houston police responded to the Petitioners address mistakenly after receiving a weapons disturbance call. They saw the petitioner and another adult male engaged in consensual sex. Both were arrested and charged with deviate

g.

b.

c.

sexual intercourse. State law prohibits person of the same sex from engaging in deviate sexual intercourse, and defines the crime as any contact w/ genitals of one and mouth of another, or penetration of genitals or anus of one w/ object Holding: Individuals are entitled to constitutional protection, under the DP Cl, in their personal lives for sexual privacy. States cannot minimize or control their destiny by criminalizing private sexual conduct Rule: The right to privacy is the right of the individual, married or single, to be free from unwarranted governmental intrusion into matters so fundamentally affecting a person as the decision whether to bear or beget a child.\

F.

Voluntariness 1. Not responsible under the law for physical involuntariness 2. The law cannot deter involuntary movement 3. To avoid physical involuntariness must show compulsion, necessity or duress 4. Burden of proof is on prosecution: must prove all elements of the crime a. If you make a movement by duress, still voluntary acted, might not be mens rea 5. MPC involuntary acts a. Sleep Walking b. Reflexes or Convulsions c. Actions under hypnosis d. Automatism i. Actions not a product of actors effort or determination (either habitual or conscious) 6. Anticipating Involuntariness a. People v. Decina: D held responsible for an accident caused as a result of a seizure. Involuntary Act yet he knew he was susceptible to seizures and took a deliberate chance 7. Law in Wisconsin a. WI stat. 929.43(1)- intoxication not a defense unless the condition is involuntary. EG, prescription drugs, take vol. but have a reaction, so not liable cuz the condition is not voluntary b. You CAN be convicted of an intentional crime if the D knows what they are getting themselves into trouble. The burden is on him to avoid situations that cause the ill effects of automatism i. Example: If D knows drinking will induce behavior of this sort then he could still be convicted of a voluntary crime ii. People v. Decina 8. People v. Grant a. Facts: A man struck a police officer at a tavern fight and then when he was in jail he had a seizure and the psychiatrist said that he has psychomotor epilepsy and he has made a number of violent attacks on people and does not remember them. Defendant says that his mind went blank at the tavern and says that he cant remember anything from 3 days after as well. Doctor also says that defendant has grand mal seizures b. D not criminally responsible for conduct if at the time of the conduct, as a result of mental disease, he lacks substantial capacity to either appreciate the criminality of his conduct. He was convicted because he was aware he was susceptible to automatism if he was intoxicated. 9. People v. Newton a. Facts: Newton (D) was convicted of manslaughter for the murder of a police officer who was shot in a struggle with D. D testified that he was unconscious during the interval, however the trial judge failed to instruct the jury on the subject of unconsciousness as a defense to a charge of criminal homicide b. Rule: Prejudicial error exists where a judge refuses a requested jury instruction where evidence of involuntary unconsciousness has been produced in a homicide prosecution and would act as a complete defense if found to have existed

The trial court erred in failing to instruct the jury on the subject of unconsciousness, as involuntary unconsciousness is a complete defense to a charge of criminal homicide and evidence of such involuntary confidence has been produced 10. People v. Newton a. Facts: D not convicted for possession of firearm. The JKF airport was merely an interruption of flight, NOT attributable to a voluntary act by D 11. Martin v. State a. Facts: Appellant Martin, after being arrested in his home, was taken onto the highway by the arresting officers, where he allegedly manifested a drunken condition. Appellant was subsequently convicted under the Alabama public drunkenness statute. b. A person in not guilty of an offense where liability is based on involuntary conduct c. Held: D was involuntarily and forcibly brought to a public place, thus no conviction for being drunk in a public place G. The Prohibition of Status Crimes 1. Must have an act (or omission) for there to be a crime 2. Must prosecute people for what they do now who they are 3. Robinson v. California (condition of being addict is not crime) a. Robinson was convicted under a California statute that made it an offense for a person to be addicted to the use of narcotics. The Supreme Court struck down the statute on Eighth and Fourteenth Amendment grounds. Essentially, the Court held that, although a legislature may use criminal sanctions against specific acts associated with narcotics addiction, e.g., the unauthorized manufacture, sale, purchase, or possession of narcotics, it could not criminalize the status of being an addict, which the Court analogized to other illnesses. 4. Powell v. Texas (drunk in public place is action not status) a. Powell was charged with violating a Texas statute that prohibited drunkenness in a public place. Powell argued that he was a chronic alcoholic and was thus unable to prevent appearing drunk in public and sought relief under the reasoning of Robinson. The Court upheld his conviction, distinguishing the case from Robinson on the ground that Powell was being punished for the act of public drunkenness and not for his status as a chronic alcoholic 5. Pottinger v. City of Miami: The harmless conduct for which they are arrested is inseparable from their involuntary condition of being homeless 6. Johnson v. State-Delivery of a controlled substance to fetus, definition of person. (must include viable fetus under definition of child in statute) a. Conviction would undermine the public policy goal to keep families in tact H. Legality and Specificity 1. Vagueness: Two problems a. Failure to provide notice to citizens that they are doing something wrong b. Encourages arbitrary enforcement of the law 2. Rogers v. Tennessee a. Rogers stabbed Bowdery in the chest, who died 15 months later from complications of this injury.Rogers was convicted o1.05(f second degree murder, and appealed, arguing that there was a common law rule where the victim would have to die within a year and a day of the incident for Rogers to have been guilty (not in the statute though, only common law rule). The reason behind the common law rule was no longer applicable because of medical science's ability to tell what caused the death. So the court had abolished this rule. Rogers argued that this violated the Ex Post Facto clause of state & fed constitutions. The court disagreed, saying that the rule is outdated, and Rogers was put on reasonable notice that the rule no longer applied, especially since so

c.

3.

4.

5.

6.

7.

8.

many jurisdictions had abolished it. Additionally, this was not a legislative change, but a judicial one Prospectively a. Ensure fair notice to Ds so they can conform their conduct to the law b. Prevent ad hominem or discriminatory criminalization (punishing conduct on basis of who committed it, rather than because of tis wrongfulness or harmfulness) Kheeler Case a. Man beats up his wife who was 8 mos pregnant with another's man's baby; he had intended to kill the baby. Question was whether the murder statute applied was the unborn baby a human life? b. The CL idea was that a human life started at birth. The Court held that it cannot change that to also include viable fetuses (where they can live on their own if they were to be prematurely born). Court noted that fair warning was a very important constitutionally protected right - due process. was not given adequate notice in this case, and so couldnt be held guilty of murder. Two proponents: a. Legality of criminal lawmaking required by statute i. Court says that only legislature can make the law, not the courts, they just interpret it ii. No common law or judge made laws iii. Legislature makes laws b. Prospectively of criminal lawmaking required by 14 th amendment (no fair notice) i. No punishment unless theres a statute that defines the crime; fair warning to people that theres a crime because want to: a. Give people a chance to conform to the law b. Dont want arbitrary lawmaking Chicago v. Morales a. D and others were charged under a new ordinance passed by the City of Chicago prohibiting gang members from loitering with one another in public places. b. The elements of the crime i. First, a police officer must reasonably believe that at least one of the two or more persons present in a public place is a gang member. ii. Second, these persons must be loitering by remaining in one place with no apparent purpose. iii. Third, the officer must order these persons to disperse. iv. Finally, the order must be disobeyed. c. D challenged the ordinance by stating that it broadly covered a significant amount of additional activity beyond what should be interpreted as loitering and was, therefore, unconstitutionally vague. d. IL Supreme Court found it was too vague since it did not provide specific limits on the discretion of police officers to determine what conduct constituted loitering. e. RULE: A statute providing penalties for criminal conduct is unconstitutionally vague if it fails to give sufficient notice regarding the type of conduct prohibited. Papachristou v. Jacksonville a. Struck down a vagrancy law that punished many classes of person, including Rogues and vagabonds,common gamblerscommon drunkards, common night walkers, thieves, pilferers or pick pockets, traders in stolen property, lewd wanton and lascivious persons. Ex Post Facto Clause a. Ex post facto law: retroactively changes the legal consequences of acts committed that existed prior to the enactment of the law; held to be unconstitutional b. Cannot be punished unless conduct was defined as a crime before it took place

III.

c. Cannot use the law that makes a crime greater than it was when committed d. Cannot use the law that makes the punishment greater e. Cannot alter evidence to convict later 9. Due Process a. Legal concept that ensures the government will respect all of a person's legal rights instead of just some or most of those legal rights, when the government deprives a person of life, liberty, or property. b. Due process has also been interpreted as placing limitations on laws and legal proceedings in order to guarantee fundamental fairness, justice, and liberty c. Lawrence v. Texas i. Outlawed sodomy between people of the same sex ii. Used cases to illustrate idea of privacy in the constitution iii. The state could not demean their existence by making private sexual conduct a crime d. Bowers v. Hardwick: Made it a crime for anyone to engage in sodomy, this case is abolished b/c of constitutional privacy, it is a fundamental right to engage in sodomy and if it were used as precedent, law would still single out gay people for doing it and not the strait people which allow discrimination and under Equal Protection Right-cannot punish gay and not married people cause Due Process gives them a right to liberty. The Guilty Mind A. Mens Rea-Culpability and the Guilty Mind-Person is not guilty of an offense unless her performs a voluntary act (or omits an act that is his legal duty to perform) that causes social harm (the actus reus), with a mens rea (guilty mind) 1. Culpability (Moral Culpability): In the early development of the doctrine, many common law offenses failed to specify any mens rea. Mens rea was defined broadly in terms of moral blameworthiness or culpability. Thus, at common law and in jurisdictions that still define the doctrine broadly, it was and is sufficient to prove that the defendant acted with a general culpable state of mind, without the need to demonstrate a specific state of mind such as intentionally, knowingly, or recklessly 2. Elemental (Mental State): Much more prevalent today is a narrow definition of mens rea which refers to the particular mental state set out in the definition of an offense. In this sense, the specific mens rea is an element of the crime. Note that a person can be culpable in that he was morally blameworthy yet lack the requisite elemental mens rea. B. Rationale 1. Deterrence a. Punishment of one who lacks a culpable state of mind will be ineffective. This type of person is harmless and not in need of reformation 2. Retribution a. The implication of a guilty verdict is that the convicted wronged the whole community. b. This conviction condemns and stigmatizes the actor. Such stigma should not attach, and liberty not be denied, to one who has acted without a culpable state of mind C. General or Specific Intent: The common law distinguished between general intent and specific intent crimes. Today, most criminal statutes expressly include a mens rea term, or a particular state of mind is judicially implied. 1. Specific: Generally speaking, a specific intent offense is one in which the definition of the crime: a. includes an intent or purpose to do some future act, or to achieve some further consequence (i.e., a special motive for the conduct), beyond the conduct or result that constitutes the actus reus of the offense, e.g., breaking and entering of the dwelling of another in the nighttime with intent to commit a felony; or b. Provides that the defendant must be aware of a statutory attendant circumstance, e.greceiving stolen property with knowledge that it is stolen. c. Example: Intentional killing of a human being-intentional

One would not be guilty of murder if acting wickedly without intent Burglary is a specific intent Special purpose for committing actus reus i. Hindering prosecution: providing false information to law enforcement; w/purpose to hinder apprehension of D (mind-purpose) g. Awareness of special circumstances: i. Receiving stolen property: receiving property knowing or believing it is stolen h. Often purpose and knowledge; not reckless or negligent 2. General: An offense that does not contain one of the above features is termed general intent, e.g., battery, often defined statutorily as intentional application of unlawful force upon another. This is a general-intent crime, for the simple reason that the definition does not contain any specific intent beyond that which relates to the actus reus itself. The only mental state required in its definition is the intent to apply unlawful force upon another, the actus reus of the crime a. Not necessary to show that D committed offense intentionally, knowingly or with any other particular state of mind b. Sufficient to show that D acted in bad character, immorality or wickedness c. General focus on conduct d. Doesnt matter why you are committing crime i. Knowingly destroying a fire alarm e. general intent + circumstance or result that must occur that is under the purposely or knowledgably categorysee 943.10 (WI) D. Common Law Categories 1. Intentionally-A person intentionally causes the social harm of an offense if: (1) it is his desire (i.e., his conscious object) to cause the social harm; or (2) he acts with knowledge that the social harm is virtually certain to occur as a result of his conduct. a. The doctrine of transferred intent attributes liability to a defendant who, intending to kill (or injure) one person, accidentally kills (or injures) another person instead. The law transfers the defendants state of mind regarding the intended victim to the unintended one 2. Knowingly- Sometimes, knowledge of a material fact an attendant circumstance is a required element of an offense. A person has knowledge of a material fact if he is aware of the fact or he correctly believes that it exists. Most jurisdictions also permit a finding of knowledge of an attendant circumstance when the defendant is said to be guilty of wilful blindness or deliberate ignorance, i.e., if the defendant is aware of a high probability of the existence of the fact in question, and he deliberately fails to investigate in order to avoid confirmation of the fact. An instruction in this regard is sometimes called an ostrich instruction. 3. Willfully-Wilful has been held in different jurisdictions to be synonymous with other terms, e.g., intentional, an act done with a bad purpose, an evil motive, or a purpose to disobey the law. 4. Negligence-Criminal negligence (as opposed to civil negligence) ordinarily requires a showing of a gross deviation from the standard of reasonable care. A person is criminally negligent if he takes a substantial, unjustifiable risk of causing the social harm that constitutes the offense charged a. Three factors come into play when determining whether a reasonable person would have acted as the defendant did: i. the gravity of harm that foreseeably would result from the defendants conduct ii. the probability of such harm occurring; and iii. the burden to the defendant of desisting from the risky conduct 5. Recklessness-A finding of recklessness requires proof that the defendant disregarded a substantial and unjustifiable risk of which he was aware. 6. Difference Between Negligence and Recklessness- The line between criminal negligence and recklessness is not drawn on the basis of the extent of the defendants

d. e. f.

deviation from the standard of reasonable care the deviation is gross in both cases but rather is founded on the defendants state of mind. Criminal negligence involves an objective standard the defendant, as a reasonable person, should have been aware of the substantial and unjustifiable risk he was taking); recklessness implicates subjective fault, in that the defendant was in fact aware of the substantial and unjustifiable risk he was taking but disregarded the risk. 7. Malice- A person acts with malice if he intentionally or recklessly causes the social harm prohibited by the offense. E. Statutory Interpretation: 1. It is sometimes necessary to determine the precise elements that the mens rea term is intended to modify. For example, in United States v. X-Citement Video, Inc the defendant was convicted of violating a federal statute that made it a felony to knowingly transport, receive, or distribute in interstate or foreign commerce any visual depiction involv[ing] the use of a minor engaging in sexually explicit conduct. Although the defendant admitting to trading in sexually explicit materials, he claimed that he was unaware that such materials depicted a minor. The issue before the Supreme Court was whether the term knowingly modified the attendant circumstance element (relating to the age of the person depicted in the video) in addition to the obvious modification of the conduct elements (transport, receive, or distribute). 2. The Supreme Court determined that the legislature intended to require knowledge of the age of the person in the video since distribution of sexually explicit, but non-obscene, videos of adults was lawful. It was therefore the knowledge that the video depicted child pornography that was criminal. F. Model Penal Code Categories of Culpability (Mens Rea)-mental state exists for every element of an offense (unless offense is a mere violation or is strict liability) 1. Model Penal Code 2.02(1) provides that, except in the case of offenses characterized as violations, a person may not be convicted of an offense unless he acted purposely, knowingly, recklessly or negligently, as the law may require, with respect to each material element of the offense. The Code requires the prosecution to prove that the defendant committed the actus reus of the offensein fact, each ingredient of the offensewith a culpable state of mind, as set out in the specific statute. a. eschews the culpability meaning of mens rea; b. discards the common law distinction between general intent and specific intent; c. limits mens rea to four terms: purposely; knowingly; recklessly; and negligently; d. requires application of mens rea to every material element of a crime, including affirmative defenses. 2. Element of an Offense: Means such conduct, or attendant circumstances or such a result of conduct as a. Included in the description of the forbidden conduct in the definition of the offense b. Establishes the required kind of culpability or c. Negates an excuse or justification for such conduct d. Negates a defense under the statute of limitations or e. Establishes jurisdiction or venue 3. Material Element of an offense: Means an element that does not relate exclusively to the statute of limitations, jurisdiction venue to any other matter similarly unconnected with (i) the harm or evil incident to conduct, sought to be prevented by the law defining the offense, or (ii) the existence of a justification or excuse for such conduct 4. MPC 2.02(4). Prescribed culpability requirement applies to all material elements. When the law defining an offense prescribes the kind of culpability that is sufficient for the commission of an offense, without distinguishing among the material elements thereof, such provision shall apply to all the material elements of the offense, unless a contrary purpose plainly appears

5. Purposely (D aims and fires gun at V) a. In the context of a result or conduct, a person acts purposely if it is his conscious object to engage in conduct of that nature or to cause such a result. b. A person acts purposely with respect to attendant circumstances if he is aware of the existence of such circumstances or he believes or hopes that they exist. 6. Knowingly (D shoots into crowd, kills V) a. A result is knowingly caused if the defendant is aware that it is practically certain that his conduct will cause such a result. b. With attendant circumstances and conduct elements, one acts knowingly if he is aware that his conduct is of that nature or that such [attendant] circumstances exist. c. Furthermore, the Code states that knowledge is established, if a person is aware of a high probability of . . . [the attendant circumstances] existence, unless he actually believes that it does not exist. i. Requirement of Knowledge Satisfied by Knowledge of High Probability ii. When knowledge of the existence of a particular fact is an element of an offense, such knowledge is established if a person is aware of a high probability of its existence, unless he actually believes that it does not exist. d. Willful Blindness=knowingly i. Cant just decide to be oblivious to an obvious fact of some kind of crime occurring, aware of high probability a material fact exists. 7. Recklessly D drunks and fires randomly at windows killing V inside. a. The Code provides that a person acts recklessly if he consciously disregards a substantial and unjustified risk that the material element exists or will result from his conduct. b. A risk is substantial and unjustifiable if considering the nature and purpose of the defendants conduct and the circumstances known to him, its disregard involves a gross deviation from the standard of conduct that a law-abiding person would observe in the actors situation. c. Substantial and Unjustifiable=gross deviation from reasonable standard of care d. Intoxication not a defense 8. Negligence (D pushes gun off table to make room, gun goes off, killing V) a. A persons conduct is negligent if the defendant should be aware of a substantial and unjustifiable risk that the material element exists or will result from his conduct. b. The definition of substantial and unjustifiable is the same as that provided for in the definition of recklessness, except that the term reasonable person is substituted for law-abiding person. 9. If statute is silent on a culpability requirement, default to recklessness 10. Only strict liability crime is rape of minor 11. Negligence and Recklessness Difference: As in common law, negligence and recklessness, therefore, require the same degree of risk-taking: substantial and unjustifiable, and the difference between them lies in the fact that the reckless defendant consciously disregards the risk, whereas the negligent defendants risk-taking is inadvertent. 12. Statutory Interpretation: A single mens rea term of whatever specific type modifies each actus reus element of the offense, absent a plainly contrary purpose of the legislature. a. If no mens rea element exists plug in recklessness-default rule b. If there is a mens rea term attached to a material element: i. Is application of term clear?

ii.

Category

Is application ambigious? Attach ambiguous mens rea term to all elements of crime in statute iii. Watch for collateral objectives (pure mental elements) Model Penal Code Example

Purposely

Knowingly

Ds conscious object is to engage in conduct of the nature to cause such a result D is aware of attendant circumstances or hopes they exist D is aware that his conduct is practically certain to produce result D is aware that such circumstances exist D consciously disregards a substantial and unjustifiable risk that the material element exists or will result from the conduct Gross deviation from reasonable standard of conduct Reckless actor consciously disregards the risk Intoxication is NOT a defense if D would have been aware of the risk when sober D should be aware of a substantial and unjustifiable risk that the material elements exist or will result from his conduct Ds risk taking is inadvertent

D aims gun and V and kills V D pulls trigger in a large group of people and kills one D gets drunk and high and pulls trigger in a large group of people

Recklessly

Negligently

D accidentally knocks a gun off of a table and it goes off

G. WI Code Categories of Culpability 1. Criminal Intent: Highest level of culpability 939.23 a. Intent in a statue is indicated by intentionally or some form of the verbs know or believe b. Conduct-purpose to conduct c. Result-Purpose to cause the result OR aware that result is practically certain d. Attendant Circumstances-actor must have knowledge of attendant circumstances that are set forth after the word intentionally e. With intent to-usually classified as a collateral objective f. Know i. Know requires only that the actor believes that the specified fact exists ii. Other ways in which knowledge becomes a mens rea a. Facts following the term intentionally in the statute b. Possession statutes g. Significance of section as indicator of legislature purpose that criminal intent be an element of crimes. 2. Criminal Negligence (Lowest level of culpability) 939.25 (objective test) a. Ordinary negligence to a high degree-conduct that actor should realize creates a substantial and unreasonable risk of death or great bodily harm to another. b. Indicated by negligent or negligently c. Distinguishing ordinary negligence and criminal negligence i. Gravity of the harm risked ii. Substantiality of the risk iii. Use of dangerous instrumentality 3. Criminal Recklessness-939.24 (subjective mental state) a. Actor creates unreasonable/substantial risk of death or great bodily harm and actor is subjectively aware of that risk to occur, applies to unborn children and mother of child b. Indicated by reckless or recklessly c. Voluntary produced by intoxicated/drugged condition is not a defense d. Conduct: Creates an unreasonable and unjustified risk of death or bodily injury and is subjectively aware of this risk

e. Result: Highly likely to occur 4. Depraved Mind/Aggravated Recklessness-940.02 a. Reckless behavior under circumstances which show utter disregard for human life b. Utter disregard is an objective standard c. Usually involves deadly instrument d. Used in 1st Degree Reckless homicide, reckless injury, recklessly endangering safety only 5. No default to recklessness 6. Assume strict liability if no mental element set forth in statute 7. WI statutory interpretation a. Differences of WI and MPC i. Not a requirement that every element have a mental state ii. No default to recklessness iii. Criminal negligence used more in WI than MPC iv. WI has crimes that are in whole or in part strict liability a. MPC has one strict liability crime-rape of minor under 10 years Category Wisconsin Criminal Intent Intentionally/know/believe used in the statute to denote this mental state (highest level of culpability) D has purpose to d conduct D has purpose to cause result or is aware that result is practically certain (as opposed to probable) D must have knowledge of attendant circumstances that are set forth after the word intentionally Criminal Recklessness D creates unreasonable and substantial risk of death or great bodily injury and the (Subjective element) actor is subjectively aware of that risk Result highly likely to occur Voluntarily produced intoxicated/drugged condition is NOT a defense to this mental state Criminal Negligence Ordinary negligence to a high degreeconduct the actor should realize creates a (objective element) substantial and unreasonable risk of death or great bodily harm to another Depraved Mind/ Aggravated Reckless behavior under circumstances which show utter disregard for human life Recklessness utter disregard is an objective standard Only used in 3 statutes :1st degree reckless homicide, reckless injury, and recklessly endangering safety Typically involves use of a deadly instrumentality 8. H. Strict Liability 1. Definition: Strict liability offenses are those that lack a mens rea requirement regarding one or more elements of the actus reus. For such statutorily enumerated offenses, the mere proof of the actus reus is sufficient for a conviction, regardless of the defendants state of mind at the time of commission. 2. Types a. Substantive Strict Liability- Liability without moral fault b. Pure strict Liability- Liability without any culpable mental state with respect to any objective element i. They are rare; because with exceptions to possession, omission and causation, most conduct terminology implies some awareness of what one is doing c. Impure Strict Liability -Liability without any culpable mental state with respect to at least one such element. 3. Policy Considerations: Between an innocent plaintiff and an innocent defendant, the defendant should have to bear the loss

4. Public Welfare: Strict liability statutes often address so-called public welfare offenses. Such statutes are aimed at conduct that, although not morally wrongful, could gravely affect the health, safety, or welfare of a significant portion of the public. a. Examples include statutes that prohibit the manufacture or sale of impure food or drugs to the public, anti-pollution environmental laws, as well as traffic and motor-vehicle regulations. b. Strict liability statutes also regulate other types of conduct against individuals, such as the offense of statutory rape which is aimed at protecting underage females who may be too immature to make knowing decisions about sexual activity 5. Presumption against Strict Liability: While strict liability statutes are not per se unconstitutional, at least under due process grounds, the Court has indicated that there is a presumption against strict liability absent a contrary legislative purpose. Thus, most courts will interpret a federal or state statute, otherwise silent in regard to mens rea, as containing an implicit requirement of some level of moral culpability. 6. MPC: The Model Penal Code does not recognize strict liability, except with respect to offenses graded as violations. For all other offenses, section 2.02 requires the prosecution to prove some form of culpability regarding each material element. (only crime it accepts is sex w/a minor) a. When the state imposes even impure strict liability, the criminal offense may be punished only as a violation b. A violation from a crime, is defined the former as an offense punishable only by a fine or find and forfeiture or other civil penalty that shall not give rise to any disability or legal disadvantage based on conviction of a criminal offense. c. No conviction may be obtained unless prosecution proves some form of culpability regarding each material element of an offense, bold assault upon strict liability 7. People v. Dillard a. The carrying of a loaded weapon in a public place is so destructive of the social order or so extremely difficult if not impossible to prove, that the doing of the act constitutes a crime, regardless of knowledge or criminal intent i. Urgency Statute/Strict Liability: Involving dangerous instrumentalities that threaten public peace, health, safety or is destructive to social order ii. Public Welfare Offense: Often involve injury to a great number of people which explains the disregard for moral guilty and the penalty is often minor a. Manufacture of impure food or drugs to the public, antipollution, not selling alcohol to minors, traffic violations b. U.S. v. Park: President of Acme charged with shipping contaminated food. Held with a duty to prevent harm. iii. Burden shifts to Defendant: Like Grant and Decina the Ds should not be drinking, driving or carrying loaded guns around iv. If he didnt even know he had a gun on him could be acquitted, since he knows he has it, constructive possession, morally responsible/blameful for carrying it around. 8. Disfavored by Courts (example of impute strict liability-did not have to prove that property was abandoned) a. Morissette v. United States i. Junk dealer sold bomb casings for profit that he found on USAF land a. Court reads a culpability requirement into the offense b. All elements of the offense include the knowledge requirement c. So not guilty because did not know they still belonged to USAF 9. Smith v. California

Here court held that the first amendment required that any law condemning possession of obscene material must include culpability with respect to the obscenity of the material i. Strict liability would penalize booksellers ii. If no culpability required-regulation for the distribution of obscene material would be ineffective 10. Corporate Crime and Vicarious Liability a. Even where statute construed to legitimacy impose strict liability as a matter of law, legal system can covertly require some level of blameworthiness b. White collar Crime: As a way to hold people accountable who weve never seen as criminal before; way to deal with that is SL c. Criminal law categories i. Criminal law is designed to deal with individuals ii. Criminal law is designed to deal with working-class deviance. a. Corporate crime does not fit neatly into either category d. Two ways to prosecute Corporations i. Strict Liability a. We dont care whether they knew what they did was wrong, you just cant do it b. Do this because it is difficult to find ways of prosecuting a corporation finding intent and negligence in an organization. ii. Try to take regular criminal law and apply it to corporations a. This is VERY hard to do b. US v. Park-President of Acme charged with shipping contaminated food. Held with a duty to prevent harm c. U.S. v. Dotterwish-This was a matter of public health. Court relied on Balint and held that the CEO liable for selling misbranded drugs even though he did not know this was happening d. Place the burden on those that can inform themselves and who are in responsible relation to conditions affecting purity. e. For public good, knowledge element not part of law 11. Ignorance is not an Excuse (cause you dont think it is constitutional, no defense) a. Sometimes 12. Lambert v. California a. Defendant Lambert was convicted for violating a California ordinance that requires any convicted person who remains in California for the specified period to register with the State. Defendant offered proof that she had no actual knowledge of the ordinance, however the proof was refused. b. In order for a conviction to stand for failure to register under this ordinance, a defendant must have actual knowledge of the duty to register or proof of the probability of such knowledge c. Imposition of this registration law on an individual who has no actual knowledge of a duty to register and where no showing is made of the probability of such knowledge is a violation of due process d. While ignorance of the law is generally not a viable defense, Due Process places some limits on the exercise of that doctrine. Notice is a requirement imposed by Due Process in situations where a penalty may ensue from an individuals failure to act e. In the case of registration laws, failure to register is conduct that is wholly passive and therefore should be evaluated differently than commission of acts or omissions under circumstances that should alert the non-actor or a duty to act. This registration law provides no notice and no opportunity, once the defendant becomes aware of the act, to comply with law. f. The majority opinion in this case carves out an exception to the general rule that ignorance of law is not a viable defense

a.

I.

J.

13. US v. X-Citement Video a. Says that statute should contain explicit mental state term, creating criminal liability for any person who knowingly transports or ships any visual depiction of a minor b. Presumption in favor of a scienter element should apply to each of elements which criminalize otherwise innocent conduct, and inferred that knowingly applied to the fact a minor was used in the film. 14. U.S. v. Hutzell a. Court held that it was only required that D knew of the facts constituting his offense, NOT that he knew it was illegal b. Lambert principle applies only to prohibitions on activities that are not per se blameworthy 15. Giorgetti v. State: Reversed conviction of sex offender registration, jury was instructed that state did not have to prove elements of intent 16. Chicone v. State: Felonies presumed to require culpability absent imposition of strict liability in the state definition. Categories of Culpability 1. Regina v. Faulkner a. A man was aboard a ship and he was trying to steal some rum and the rum fell out and the man had lit a match and the whole ship burned. The mans attorney said that he should not be held culpable for the fire, because he did not intend to set fire to the ship. The prosecution argued that the man was committing a felony at the time and that this eliminated the need to prove intent. Jury was instructed that even if the man did not intend to burn the ship, if he was attempting to steal the rum and started the fire in the process, he should be found guilty of the arson as well. b. Having the intent to commit a specific crime does not serve to establish the intent to commit a second crime that happens by accident 2. US. Jewel a. Appellant contends that he did not have the requisite mental state when he drove a car into the United States that contained concealed drugs. Appellant argued that he had no actual knowledge of the drugs.(he was paid by stranger to drive a car over boarder with a secret compartment-willful blindness or ignorance is same as knowledge) b. Willful blindness is equivalent to knowledge c. One may deliberately shut his eyes to avoid knowing what would otherwise by obvious to view. In such cases, the person acts at his peril and is treated as having knowledge of the facts as ultimately discovered to be. d. To act knowingly is not necessarily to act only with positive knowledge, but also to act with an awareness of the high probability of the existence of the fact in question. Mistake-Mistake of Fact 1. Common Law Rules: Many states follow the Model Penal Code in requiring proof of mens rea for every element of the offense. Nevertheless, the common laws two approaches to mistakesdepending on whether the offense charged is characterized as general-intent or specific-intenthas endured. a. If the crime is one of strict liability, a mistake of fact is irrelevant. Otherwise, the first step in analyzing a mistake-of-fact claim in a jurisdiction that follows common law doctrine is to determine whether the nature of the crime of which the defendant has been charge is specific-intent or general-intent. 2. Negates Mens Rea a. The person was factually mistaken and thus acted in a morally blameless manner-not culpable i. General Intent

Negates mens rea in the elemental sense: the mistake of fact D does not possess the specific state of mind requisite in the definition of the crime. i. Specific Intent 3. Specific-Intent Offenses A defendant is not guilty of an offense if his mistake of fact negates the specific-intent portion of the crime, i.e., if he lacks the intent designated in the definition of the offense, e.g., knowingly, negligently, recklessly. a. Elemental Approach b. Does NOT matter whether mistake was reasonable or unreasonable, as long as it negates the specific intent element c. Example: D attempts to have sex with V. Because of a language barrier, D, incorrectly believes that V was a consenting prostitute. D is arrest as intercourse occurs and is charged with the specific intent of assault with the intent to commit rape. i. Ds mistake belief that V consented is pertinent to the specific intent portion of the crime, intended to rape 4. General-Intent Offenses a.

b.

Ordinary Approach: Reasonableness The ordinary rule is that a person is not guilty of a general-intent crime if his mistake of fact was reasonable, but he is guilty if his mistake was unreasonable.(OBJECTIVE) b. Moral-Wrong Doctrine On occasion, courts apply the moral wrong doctrine, under which one can make a reasonable mistake regarding an attendant circumstance and yet manifest a bad character or otherwise demonstrate worthiness of punishment. The rule is generally that there is no exculpation for mistakes where, if the facts had been as the defendant believed them to be, his conduct would still be immoral. i. Example: Factor in underage sex cases: even if she looked 18, and told you she was 18, still no defense becaue it was morally wrong c. Legal-Wrong Doctrine A less extreme alternative to the moral-wrong doctrine is the legal-wrong doctrine. That rule provides for no exculpation for mistakes where, if the facts were as the defendant thought them to be, his conduct would still be illegal. Often this means that a defendant possessed the mens rea for committing a lesser offense, but the actus reus was associated with a higher offense. Under this doctrine, the defendant is guilty of the higher offense in such circumstances. i. Example: Actor thinks that he has 500g, but actually has 2,000 g of cocaine ii. Under CL we dont care iii. Under WI, would only be guilty of possessing 500g 5. Strict Liability: Under CL, No mistake for these offense a. Example: D drives above the speed limit because speedometer is inaccurate, he will be convicted of a strict-liability driving offense, even if the speedometers fault calibration was unknown or unforeseeable to him b. Statutory Rape: No mistake regarding age of victim 6. MPC-Section 2.04(1) provides that a mistake is a defense if it negates the mental state required to establish any element of the offense (SUBJECTIVE) a. If no mens rea is stated, then D is guilty if he acts recklessly knowingly or purposely to each element, regardless of mistake b. If D was negligent regarding the risk of which he was mistaken, D not guilty i. Unless it was a negligent crime c. Exception: The defense of mistake-of-fact is not available if the defendant would be guilty of another offense, had the circumstances been as he supposed. In such cases, contrary to the common law, the Code only permits punishment at the level of the lesser offense. [MPC 2.04(2)] (lower the degree of the offense to that which he believed he was committing)

MPC Purposeful Knowing Reckless Negligent Strict Liability

Example: Carl Copp is an undercover police officer posing as a school teacher trying to bust a drug dealer. Dave Drugg thinks Carl is meddling in his business and tries to scare him by threatening to punch him. Carl is convicted of assault to a police officer. But he is given the punishment as if he had assaulted a school teacher. 7. Wisconsin (OBJECTIVE) a. No mistake defense to negligence b. WI 939.43-An honest error, whether fact or law other than criminal law, is a defense if it negates the existence of a state of mind essential to the crime c. If no mental state given, does not default to recklessness, specific intent d. Mistake as to age of minor or existence/constitutionality of the offense is not a defense e. WI 939.43honest error, whether of fact or law other than criminal law is a defense if negates the state of mind essential to the crime f. WI 939.42- voluntary intoxication can be a defense if knocks out mental element(except criminal recklessness) CL Type of Mistake Specific Intent Any Mistake (reckless, negligent, faultless) Any mistake General Negligent/Faultless (reasonable) mistake General Intent Faultless (reasonable) mistake N/A

d.

K. Mistake-Mistake of Law 1. General: Under both the common law and Model Penal Code, ignorance of the law excuses no one. Nevertheless, a number of doctrines apply when a defendant is ignorant or mistaken about the law. 2. Reasonable-Reliance Doctrine (Entrapment by Estoppel)-Under both the common law and Model Penal Code, a person is excused for committing a criminal offense if he reasonably relies on an official statement of the law, later determined to be erroneous, obtained from a person or public body with responsibility for the interpretation, administration, or enforcement of the law defining the offense. a. Official Statement For a statement of the law to be official, it must be contained in: i. (1) a statute later declared to be invalid; ii. (2) a judicial decision of the highest court in the jurisdiction, later determined to be erroneous; or iii. (3) an official, but erroneous, interpretation of the law, secured from a public officer in charge of its interpretation, administration, or enforcement, such as the Attorney General of the state or, in the case of federal law, of the United States. b. Even if a person obtains an interpretation of the law from a proper source, that interpretation must come in an official manner, not an offhand or informal manner. For example, a person may rely on an official opinion letter from the state Attorney General, formally interpreting the statute in question.

c.

3. Exemptions to the Reasonable Reliance Doctrine a. Reliance on Ones Own Interpretation of the Law A person is not excused for committing a crime if he relies on his own erroneous reading of the law, even if a reasonable person even a reasonable lawtrained person would have similarly misunderstood the law.

b.

c.

Advice of Prosecutor Although there is very little case law on the matter, there is some support for the proposition that a person may not reasonably rely on an interpretation of a law provided by a local prosecuting attorney. Advice of Private Counsel Reliance on erroneous advice provided by a private attorney is not a defense to a crime.

4. Fair Notice and the Lambert Principle a. Common Law At common law, everyone is presumed to know the law. i. However, in Lambert v. California [355 U.S. 225 (1957)], the Court overturned the petitioners conviction for failing to register with the city of Los Angeles as a prior convicted felon, as required pursuant to a strict liability ordinance of which he was unaware; the Court reversed on lack of fair notice due process grounds. ii. The Supreme Court held in Lambert that, under very limited circumstances, a person who is unaware of a duly enacted and published criminal statute may successfully assert a constitutional defense in a prosecution of that offense. iii. Key to the courts decision in Lambert was the passive nature of the offense. Namely, (1) it punished an omission (failure to register); (2) the duty to act was imposed on the basis of a status (presence in Los Angeles), rather than on the basis of an activity; and (3) the offense was malum prohibitum. As a result of these factors, there was nothing to alert a reasonable person to the need to inquire into the law b. Model Code The Model Penal Codes fair notice exception [MPC 2.04(3)(a)] applies where: i. (1) a defendant does not believe that his conduct is illegal, and ii. (2) the statute defining the offense is not known to him; and was not published or otherwise reasonably made available to him before he violated the law. 5. 18.04 Ignorance or Mistake that Negates Mens Rea a. Common Law i. Different Law Approach A defendants lack of knowledge of, or misunderstanding regarding the meaning or application of, another law usually, it will be a nonpenal law will negate the mens rea element in the definition of the criminal offense. When a defendant seeks to avoid conviction for a criminal offense by asserting a different-law mistake, on the ground that the different-law mistake negates his mens rea, the first matter for determination is whether the offense charged is one of specific-intent, general-intent, or strict-liability. ii. Example: Getting divorced in DE and remarried in MD, but MD still recognizes 1st marriage, may not be guilty of bigamy getting married w intent to marry 2 plus women (mistake about divorce law) a. Specific-Intent Offenses A different-law mistake, whether reasonable or unreasonable, is a defense in the prosecution of a specific-intent offense, if the mistake negates the specific intent in the prosecuted offense. b. General-Intent Offenses Although there is very little case law on point, a different-law mistake, whether reasonable or unreasonable, apparently is not a defense to a general-intent crime. c. Strict-Liability Offenses A different-law mistake is never a defense to a strict-liability offense.

Model Penal Code MPC 2.04(1) provides that mistake or ignorance of the law is a defense if it negates a material element of the offense. Application of this defense generally surfaces in cases of a different-law mistake. 6. Wisconsin a. Honest mistake of law, but not criminal law, is a defense if it negates the mens rea requirement b. Mistake must be reasonable c. D asks DA in open manner and relies on it, thats a defense IV. Particular Mental States Only Negated by Certain Mistakes b. V. Category Purposely Knowingly Recklessly Negligently Strict Liability MPC Any mistake (reckless, negligent, or faultless) Any mistake (reckless, negligent, or faultless) Negligent or faultless mistake Faultless mistake N/A Common Law General Intent Any reasonable mistake of fact Any reasonable mistake of fact Any reasonable mistake of fact Reasonable mistake NO mistake Common Law Specific Intent Any mistake of fact reasonable or unreasonable Any mistake of fact reasonable or unreasonable Any mistake of fact reasonable or unreasonable Reasonable mistake NO mistake

A. Cases 1. Regina v. Prince a. Defendant was convicted under a statute making it unlawful to take any unmarried girl under the age of sixteen out of the possession and without the consent of her parents. Although the girl taken by Defendant was actually 14 years old, she had told the Defendant, and the defendant reasonably believed that the girl was 18. b. Where a statute is silent as to the mens rea required to make the act a crime, the court is not bound to read a requirement of mens rea into the statute. c. Mistake of fact does not stand as a defense to a crime where the statute making the act a crime contains no requirement of knowledge of that fact to begin with. d. In this case the forbidden act is wrong in itself and the legislature has enacted that if anyone does this act, he does so at his own risk. e. This case introduces the mistake of fact defense in the context of a criminal act for which the statutory language making the act a crime has no requirement of mens rea. 2. People v. Ryan a. Statute says its a felony to "knowingly and unlawfully possess . . . Six hundred twenty-five milligrams of a hallucinogen." was convicted of possessing mushrooms which had this hallucinogenic properties of more than the statutory limit. argued he did not know that the mushrooms would have so much. No evidence was offered on how much hallucinogen would typically be in that amount of mushrooms. But court convicted him because they said the statute implied "knowingly" to refer only to possession of it, not to the actual weight b. Court first looks at legislative intent - the statutory language i. It says "knowingly and unlawfully possesses 625 mg of a hallucinogen ii. It must also mean the weight, b/c it stated in the same sentence iii. Therefore, there is a mens rea element associated with the weight of the drug c. Also, looking at the legislature's culpability requirements: i. Says that when the statute says a culpable mental state is to be applied to an element of the offense, it is presumed to apply to every element of the offense, unless clearly stated otherwise d. Rule: Unless otherwise stated in the statute, the mental element of "knowingly" is to be applied to all elements of the crime

3. People v. Bray a. was convicted of being an accessory after the fact and was sentenced to probation. He moved to CA and he wanted to become a guard. He bought a gun and his place got searched and the gun was found and he was charged with two counts of possession of a firearm by a felon. There is a lot of evidence that he did not know he was a felonvarious applications filled out. b. In order to convict , prosecution must prove a prior felony conviction and ownership of a firearm capable of being concealed c. Bray says that the prosecution must show that he knew he was a felon d. He also says that he should be able to prove that he was mistaken as to whether he was a felon e. Courts tend to dislike statutes that impose strict liability f. Prosecution says that it is not required to prove that the defendant knew his status g. Knowledge of a felony conviction is important when there is doubt as to whether the individual is a felon h. Kansas experts said it was hard to tell whether his crime was a felony i. Without knowledge that he was a felon, he could not be aware of the facts that would bring his conduct within that prohibited by the statute j. RULE: Under these circumstances: Where D was not aware his conduct was a felony, he negated a mental element of the crime and was not convicted. 4. Regina v. Smith a. Smith told police he honestly believed the property he destroyed was his own. Court dismissed charge of damaging property of another on the ground that prosecution had not proven Smith did so intentionally, did not meet all elements 5. United States v. Barker a. What he did now know did not affect the elements of the crime b. Baker commits all the elements of the crime, he just does not know its an offense; CRIMINAL LAW DOES NTO REQUIRE THAT D KNEW HIS ACT WAS ILLEGAL 6. Hopkins v. State a. State attorney advised him that signs were legal prior to putting them up. Advice of counsel, followed even in good faith is not excuse for violating the law. (WI does not follow case) 7. Cheek v. United States a. Trial Court did not instruct jury as C requires, but the SC held that is should have done so: That he was not guilty of the offense if he believed, even unreasonably, that he was not legally required to report his wages 8. Commonwealth v. Twitchell a. Good faith mistake of law defense available where rely (mistakenly) on official statement of law and try to comply while in exercise of religious belief (very narrow application) 9. Long v. State a. Ignorance and consultation with personal attorney not available in every state, but here D actively sought legal advice and followed thru with that legal adviceb. Placed burden on D to show reasonable reliance on his attorneys advice at his new trial 10. Montana v. Egelhoff a. Appellant was convicted of premeditated murder. Appellant argued that he should have been allowed to present evidence of voluntary intoxication to show that he did not commit premeditated murder b. Rule: Allowing evidence of voluntary intoxication is not a traditional well founded principle of American law that would constitute a violation of the Defendants 14th Amendment rights under the Constitution of the United States

c.

d.

Held: Although voluntary intoxication cannot negate a mental state, it can be shown to the jury to help them assess whether a defendant acted in premeditation or if the murder was done only in the heat of passion. Although a rule allowing a jury to consider evidence of a defendants voluntary intoxication where relevant to mens rea has gained considerable acceptance, it is of too recent vintage and not fundamental to constitute the prohibition of allowing evidence of voluntary intoxication is not a violation of a defendants 14th Amendment rights.

VI.

Causation A. CAUSATION pg. 287 B. Elements of a crime 1. Circumstance Results Conduct a. Every statute falls into the above categories. b. Conduct all characteristics are different circumstance elements c. Results caused by the conduct d. Ex. setting up fruit stand on 94 i. Circumstance setting up on public highway ii. Criminal act defines the offense that prosecution has to prove iii. Causation of crime is usually part of conduct part of crime iv. Result may be the particular act v. 940.01 First degree intentional homicide. vi. Harm is indisputable fact vii. Cause harm sometimes luck and no bad results if unlucky v. bad results viii. At what level should person be held responsible for causing the harm e. But-for cauation i. But-for which the harm would not have occurred. ii. Factual causation iii. Necessary but not sufficient causation iv. Question if actor had not done (x) would harm occur? v. Tied into Proximate cause a. Foreseeability, another causal conduct, intervening events b. Proximate cause hold them responsible for normative judgment; based on standards or norms; intuitive judgment is causation too remote? c. When is cause too remote? d. Foreseeability 1. Requires connection between culpable mental state and result 2. Causal chain many different causal factors what is most influential? e. Intervening events 1. An intervening event that broke the causal chain 2. Canteen example (END of class notes) C. Common Law 1. causation in general a. actus reus element b. link b/w voluntary act (or omission) and social harm c. must be proved beyond reasonable doubt d. applies only to result crimes specifically homicides 2. Actual cause a. But for test broad standard that asks, but for the voluntary act or omission by the D, would the harm to the P have occurred when it did? b. Distinguished from conditions which are necessary for harm to occur c. Actual cause does not imply liability; must have mens rea and proximate cause

3. Violent Acts-Common Law was originally concerned primarily with suppressing conspicuous violence a. TF, necessary conditions for a harmful results were only perceived as criminal if they appeared violent-expanded to be dangerous 4. Foreseeability a. Connection between the actors culpable mental state and the result b. If D is charged with recklessly causing injury, the injury resulting from defendants reckless action bust be one she foresaw in order for us to conclude she caused it c. Negligence actions causes harm only if it leads to harm that is reasonably foreseeable. (proximate cause) 5. Multiple Actual Causes a. Accelerating a result (p. 184 dressler) i. D1 and D2 intentionally shoot V in the stomach V dies ii. Wound from D1 accelerated death from 1h to 5min iii. Both actors are actual causes of the death iv. Both may be prosecuted for murder v. But for the voluntary act of D1, the harm would not have occurred when it did b. Concurrent Sufficient Causes (p 185) i. Either wound would have killed V instantly ii. Each act alone was sufficient to cause the result that occurred when it did iii. But for the act would harm have occurred when and as it did c. Obstructed Cause (accelerating a result) i. D1 shoots V in the stomach ii. Simultaneously and independently, D2 shoots V 3 times in the head, killing him instantly iii. D1 attempted to take Vs life; he was thwarted in this goal b/c D2 was a more effective killer iv. If both acts hasten the social harm, both will probably be convicted (e.g., two hunters simultaneously shooting another hunter) d. Proximate Cause (narrower than but for test) i. Direct Cause (majority of cases) a. An act that is the direct cause of social harm is a proximate cause of it b. No intervening factors are seen as the causes of the social harm (aneurism example punch and hit head, difference between proximate and actual) 6. Intervening Cause a. An independent force that operates in producing social harm, but which only comes into play after the Ds voluntary act or omission has been committed b. Circumstances under which an intervening cause will absolve the D of liability: i. Superseding Cause (de minimis contribution to social harm) Ds conduct is substantial when compared to that of the intervening cause (D thus not liable)the law will treat substantial intervening cause as the proximate cause of the harm ii. Foreseeability of the intervening cause a. Test: Was intervening cause reasonably foreseeable? b. Responsive (dependent) Intervening Causes: An act occurs in reaction or response to the Ds prior wrongful actGenerally, a responsive intervening cause does not relieve the initial wrongdoer of liability, unless the response was highly abnormal or bizarre

c.

d.

e.

f.

actor will be liable where victim attempts to extricate himself of another from a dangerous situation cause by D, even if Victim is contributory negligent 2. negligent medical treatment will generally not relieve liability 3. grossly negligent medical treatment is sufficiently abnormal to supersede actors wrongful acts and responsibility 4. Ex: if D injures someone and negligent medical treatment is given, D may be punished for the death; however if the treatment is grossly negligent or reckless, the D may not be held liable Coincidental (independent) intervening causes: An act that does not occur in response to initial wrongdoers conductGenerally, a coincidental intervening cause relieves the D of liability unless the intervention was foreseeable 1. D merely places the victim in a position where the intervening cause could independently act upon him 2. Free, deliberate and informed human intervention will more likely relieve actor of responsibility than if intervention is not entirely free 3. an omission will rarely, if ever, serve as superseding intervening cause (vs failure to wear seatbelt where D drives negligently) 4. EX: if injured person is hit by a car on the way to the hospital Intended-Consequence doctrine 1. where a voluntary act intended to bring about what in fact happens, and in the manner in which it happens, the cause of social harm is traced backwards until an intentional wrongdoer is involvedDespite the intervening acts, the D is found to be the proximate cause of the harm 2. you got what you wanted 3. Different Manner: MPC-Look to whether the actual result involved the same kind of injury or harm as that designed or contemplated and is not too remote or accidental in its occurrence to have a just bearing on the actors liability or on the gravity of his offense 4. Different Injury-Still counted as the legal cause if the injury or harm designed or contemplated would have been more serious or more extensive than that caused a. Transfer of intent b/w offense, intend murder, commit assault; guilty of assault even though it was not intended b. Same in Common Law c. Revere: Cannot be guilty of murder if you only intended battery; lack the requisite mens rea, not a causation issue Temporal Intervals-Lengthy intervals b/w cause and result raise problems 1. More plausible that but-for Ds actions the victim may suffer some other misfortune 2. More plausible it becomes that an undetected factor caused the result Mistreatment and Failure to Retreat

1.

1. 2.

TAKE YOUR VICTIMS as you find them Negligent Treatment: Simply medical negligent actions is not enough, but grossly negligent medical behavior is enough to break the chain of causation

g. Suicide 1. If its a foreseeable risk and a natural and probably result of action (rape); you are responsible. h. Transfer of Intent (MPC and WI work) 1. CL-Disposes of discrepancies b/2 the actual and intended victim through transfer of intent 2. MPC-An actual result will be treated as having been caused by my act, even w/o the requisite mode of culpability, if it differs from the intended result only in the respect that a difference person or property is injured or affected i. Apparent-safety doctrine 1. causal connection between Ds harmful conduct and the end result is severed 2. when a Ds active force has come to rest in a position of apparent safety, the court will follow it no longer 3. the D will not be responsible for the harm the victim places himself into after the D no longer poses a threat to the victim 4. EX: man threatens wife, she leaves and walks within 200 yards of Dads house, then decides to sleep outside and freezes to deathhusband no longer a threat to safety and will likely be absolved of any liability 7. Duties-Limits the causal responsibility of omitters to those who have some duty to act, resulting forms statute, status, contract or undertaking a. Thus, where passive conduct is a necessary condition for a result, it must be combined with a duty to constitute a cause D. Model Penal Code (p 195) 1. Actual cause a. Basically the same as common law b. but for test applied c. substantial factor test not applied in cases of concurrent sufficient causes, only but for 2. Proximate Cause a. MPC treats but for cause as the exclusive means of causation b. Question is whether actor cause the prohibited result with the level of culpability required by the definition of the offense (i.e., purpose, knowledge, recklessness, or negligence)Does not matter if D was the proximate cause just whether D caused the result with the appropriate mens rea c. Causation is not established unless the actual result (including the way in which it occurred) is a probably consequence of the actors conduct (foreseeability issue) d. *Apply statute and requisite mens rea to facts of case* breakdown elements and go through each E. Wisconsin 1. Substantial Factor Test a. Used to determine proximate causation in WI b. Act of D must be substantial factor in causing resultthere can be more than one substantial factor c. * Hint for EXAM: Consider that WI may adopt MPC or CL provisions, so include them in your analysis!*

Causes needed

Intervening factors

Medical mistreatment

Common law But for causew/o Ds conduct, result wouldnt have occurred, Ds conduct must be substantial factor in producing result Proximate causeno event of causal significance intervened b/w Ds conduct and the result D not responsible for unforeseen (to reasonable person in Ds situation) independent intervening causes General negligence not intervening Gross negligence is intervening

MPC But for causeactual result and the way it occurred is a probable consequence of Ds conduct, must be foreseeable to D

WI But for causeDs act must be substantial factor in causing result (can have 1+ substantial factors

May adopt this provision

May adopt this provision

F. G. But For Causation 1. Regina v. Martin Dyos a. Dyos (defendant) was a member of a group of seven young men who participated in a fight with another group of five youths following a dance at a community center. During the fight, one of the group of five, RM, was hit in the forehead by a brick and received another blow behind his right ear. Dyos admitted to the police that he had hit RM with the brick but there was no indication as to the source of the second wound. When RM subsequently died from his injuries, Dyos was charged with affray, murder and grievous bodily harm. At Dyos trial, a pathologist testified that both wounds were potentially fatal, that RM might have survived from the first wound, but that there was no certainty as to which wound occurred first. At the close of the prosecutions (plaintiffs) case, the court granted Dyos motion to dismiss the murder charge on the grounds that, first, the injury behind RMs ear might have been the cause of death, and second, there was no evidence that Dyos blow with the brick was beyond a reasonable doubt the cause of death. b. Concurrent Sufficient Causes: Brick in the head brawl: no but for causation can be proven b/c cant be sure from which the impact occurred c. Substantial Factor Test: Ds conduct must be a substantial factor in producing death H. Violent Acts-Cause v. Condition 1. Hubbard v. Commonwealth a. Condition: Normal events (V has a heart condition) b. Cause: Abnormal, unlawful act by D; and natural and probable consequences of Ds actions c. Not guilty for guards hearth attack in jail brawl b/c there was a preexisting condition in V, and D didnt even touch V. d. Was not a DIRECT OR PROXIMATE CAUSE (death was not due to his blow and death was not a probably and natural cause of his indirect, unlawful act) I. Proximate Cause-Foreseeability 1. Commonwealth v. Rhoades a. Requires a connection b/2 the actors culpable mental state and the resultProximate Cause b. Firefighter dies but this was not foreseeable, cannot have criminal liability on just a but-for cause J. Intervening Causes

VII.

1. Commonwealth v. Root a. The tort liability concept of proximate cause has no proper place in prosecutions for criminal homicide and a more direct causal connection is required for conviction. b. Ds reckless conduct was not a sufficiently direct cause of the competing drivers death to make him criminally liable. c. Did not force decedent to swerve into traffic d. Decedent was negligent and brought about own death 2. Russian Roulette and that the deceased had shot himself in the head? Made his chances of death more likely; is responsible 3. State v. Wassil: D brought drugs for himself and V, V died b/c of overdoes-injecting drugs not an interning event relieving D of liability when D provided the drugs a. Him giving him the drugs was a proximate cause and that him injecting the drugs himself was not an efficient intervening cause of death 4. Defendant as an intervening Actor: a. Meant to kill, intoxicated him (not dead); thought he was and threw in river (drowned); still liable even though no mens rea for second K. Negligent/Grossly Negligent Mistreatment 1. United States v. Hamilton a. V pulls out tubes in hospital, there b/c of Ds actions, V dies i. If a person strikes another and inflicts a blow that may not be mortal in and of itself but starts a chain of causation that leads to Vs death, then D guilty, even if V contributes to his own death or hastens it by failing to take proper treatment 2. Blaue: B stabbed V, penetrating her lung. She refused a blood transfusion b/c Jehovahs Witness. And died; held guilty a. Take victim as you find them 3. Baylor v. United States a. D struck his wife and waited two hours for ambulance and diagnoses. She suffered lung problems and got pneumonia and died two weeks later. It held that negligence of doctor did not negate liability because he could have reasonably foreseen that there was a possibility that she could receive negligent treatment. She suffered a dangerous wound that would have killed her without treatment. 4. People v. Stewart a. D stabbed victim in stomach and surgeon closed wound, noticed unrelated hernia and proceeded to fix that too and anesthesiologist exhibited gross negligence; he later died of complications and Ds charges were dropped. L. Suicide 1. Stephenson v. State a. When suicide follows a wound inflicted by D, his act is homicidal, if deceased was rendered irresponsible by the wound and as a natural and probable result of it (abducted, raped her and she killed herself) 2. State v. Bier: a. Handed drunk wife a gun and challenged her; negligent homicide conviction, Ds very intoxicated wife, shot herself during argument. D created a foreseeable risk. M. Omission: Duties 1. People v. Beardsley a. The fact that Burns was in his house created no such legal duty as exists in law and is due from a husband towards his wife. Burns was past 30 years old, married twice, accustomed to visiting saloons, used intoxicants and was there of her own will. Moral obligation, no legal duty. Presumptions A. Presumptions

1. Common Law Mandatory Presumptions If a jury is instructed that it must presume Fact B upon proof of basic Fact A, the presumption is a mandatory presumption a. Rebuttable Mandatory Presumptions A mandatory rebuttable presumption requires a finding of the presumed fact upon proof of the basic fact, unless that finding is rebutted by the opposing party. The procedural effect of a mandatory rebuttable presumption is to shift to the defendant the burden of persuasion regarding the presumed fact, upon proof by the prosecution of the basic fact. Rebuttable mandatory presumptions are unconstitutional when the presumed fact is an element of the crime charged. b. Irrebuttable (Conclusive) Presumptions An irrebuttable or conclusive presumption requires the jury to find the presumed fact upon proof of the basic fact, even if the opposing party introduces rebutting evidence. (NB: True irrebuttable presumptions are rare.) A mandatory irrebuttable presumption pertaining to an element of an offense is unconstitutional for the same reasons that mandatory rebuttable ones are impermissible. 2. Common Law Permissive Presumptions (Inferences) A permissive presumption or inference is one in which the fact-finder may, but need not, find the existence of the presumed fact upon proof of the basic fact. As such, it does not formally shift the burden of proof from one party to another, as a true mandatory presumption does. While not unconstitutional per se, to be constitutionally permissible, there must be a rational connection between the basic fact and the presumed (inferred) fact; i.e., the presumed fact more likely than not flows from the basic fact. 3. Model Penal Code The Model Penal Code [ 1.12(5)(b)] does not recognize mandatory presumptions but permits permissive presumptions regarding elements of an offense. 4. Burden of Production a. Prosecution Burden of Production Prior to trial the prosecution must file a document with the court that indicates the crime or crimes it believes that the defendant has committed. This document provides the accused with notice of the essential elements of the offense(s) charged, and the basic facts that the prosecutor intends to prove at trial to support his allegation that the defendant committed the crime(s). The prosecutor must produce enough evidence that a rational trier-of-fact may fairly determine that the elements of the crime have been proved beyond a reasonable doubt. i. If the judge concludes that the prosecutor failed to satisfy the burden of production regarding any element of the offense charged, the defendant is entitled to a directed verdict of acquittal at the conclusion of the prosecutors case-in-chief or at the end of the trial. If the prosecutor failed to introduce enough evidence to support a jury finding beyond a reasonable doubt that the defendant committed the crime, there is no reason for it to deliberate on the matter. b. Defendants Burden of Production The defendant is sometimes required to provide advance notice to the prosecution of defenses he intends to assert at trial. The amount of evidence required to satisfy the burden of production on affirmative defenses varies by jurisdictions. In some jurisdictions the defendant meets his burden of production (and, thus, is entitled to an instruction to the jury on the defense) if he produces more than a scintilla of evidence regarding an affirmative defense; in other jurisdictions the defendant must introduce enough evidence to raise a reasonable doubt on the issue of the defense claimed. i. If the defendant fails to meet his burden of production regarding an affirmative defense, the judge will not instruct the jury on the law pertaining to the defense, and the defendant is not entitled to have the issue considered by the jury in its deliberations. 5. Burden of Persuasion Once a party satisfies his burden of production pertaining to an issue, that matter is properly before the jury as fact-finder, i.e., it will decide whose factual claims are more persuasive.

6.

7.

8. 9.

10. 11.

Prosecutions Burden of Persuasion (the Winship doctrine) Pursuant to the due process clause, a person charged with a crime is presumed innocent and, to enforce this presumption, the Supreme Court held in In re Winship that the prosecution must persuade the fact-finder beyond a reasonable doubt of every fact necessary to constitute the crime charged. This rule has come to be known as the Winship doctrine. i. If the prosecution fails to meet its burden of persuasion, the defendant must be acquitted. Procedurally, the acquittal may occur in either of two ways. First, after the prosecution completes its presentation of evidence or immediately before the case is due to be submitted to the jury, upon motion of the defendant, the trial court must direct a verdict of acquittal if the evidence, viewed in the manner most favorable to the prosecution, can support no reasonable verdict other than acquittal. Alternatively, if the judge believes that reasonable minds can differ and, therefore, permits the case to go to the jury, the jury must acquit if it possesses a reasonable doubt regarding one or more elements of the offense charged. b. Defendants Burden of Persuasion Jurisdictions differ in their allocation of the burden of persuasion regarding affirmative defenses. Some states require the prosecution to disprove beyond a reasonable doubt some or all defenses, once the defendant has met his burden of production. In states that allocate to the defendant the burden of persuasion regarding defenses, it is typical to require the defendant to prove the validity of the claimed defense by the less strict preponderance-of-the-evidence standard. i. If a defendant presents sufficient evidence to meet his burden of production regarding a defense to the crime charged, the jury must be permitted to evaluate the defense claimed. When the defendant also has the burden of persuasion, a jury should reject the claimed defense if he fails to satisfy the stated burden. c. If the prosecution has the burden of disproving a defense, the jury must acquit the defendant if the prosecution fails to persuade the jury beyond a reasonable doubt of the defenses non-existence. Determining if a Fact Relates to an Element or an Affirmative Defense A prosecutor must prove every element of an offense beyond a reasonable doubt. The legislature may allocate to the defendant the burden of persuasion regarding facts not formally identified as elements of the offense charged. Thus, generally speaking, a legislature may allocate to the defendant the burden of persuasion regarding facts that relate to an affirmative defense. However, a court interpretation of the statute may be required to determine whether a particular fact relates to an element of an offense or to an affirmative defense. Model Penal Code Except for defenses that the Code expressly requires the defendant to prove with the standard of proof being a preponderance of the evidence the prosecution must prove every element of an offense beyond a reasonable doubt, [MPC 1.12(1)] including conduct that negates an excuse or justification for the action. [MPC 1.13(9)(c)] That is, the Model Penal Code allocates to the prosecution the duty to disprove defenses, assuming that the defendant has satisfied his burden of production. Winshop: Requires Prosecution to prove every element BARD; Due Process protects accused against conviction without that Mulaney: Statute was wrong b/c did not require prosecution to prove all elements a. P has duty to prove BARD elements of a killing, but presumption included that relieved P from proving all elements, which violated Winship Patterson: P still had to prove all elements of crime, just moved what was wrong in Mulaney to affirmative defense of EED, which D then had to prove Mandatory Presumptions are always what pose the problem a. It takes away prosecutors need to prove something-unconstitutional b. If make it an affirmative defense, not unconstitutional a.

VIII.

12. Book Notes: a. Burden of Proof has tactical importance in criminal cases because it determines which sides gets the benefit of the doubt in disputed questions i. Important policy issues because it allocates the risk of error ii. Normal rule that prosecution must prove its case beyond a reasonable doubt implies a social or political or moral judgment that we are far more willing to tolerate erroneous acquittals than erroneous convictions iii. Rules gives accused are entitled to presumption of innocence. b. Burden of Production-duty to introduce at least some prima facie evidence in order to compel a fact finder to at least consider a claim c. Burden of Persuasion-The duty to persuade the fact finder that the totality of evidence presented warrants accepting or rejecting that claim d. Prosecution is required to prove that the D had committed a criminal act, and the D was required to prove circumstances of justification, excuse and alleviation e. Homicide A. Common Law and Statutory Homicide At very early common law, homicide was defined as the killing of a human being by a human being. This definition included suicide. However, modern law defines homicide as the killing of a human being by another human being. Suicide, therefore, is no longer a form of homicide in most statutes. Homicide is divided into two crimes murder and manslaughter. B. Human Being The common law and majority approaches define the beginning of life as birth for purposes of interpreting the criminal homicide law. A minority of states now treat a viable or, at times, even nonviable fetus as a human being under the homicide statute. 1. Regarding the end of human life, a majority of states, either by statute or judicial decision, have incorporated brain death in their definition of death. C. Murder The common law definition of murder is the killing of a human being by another human being with malice aforethought. D. Manslaughter Manslaughter is an unlawful killing of a human being by another human being without malice aforethought. E. Malice As the term has developed, a person kills another acts with the requisite malice if he possesses any one of four states of mind: 1. 2. 3. 4. the intention to kill a human being; the intention to inflict grievous bodily injury on another; an extremely reckless disregard for the value of human life; or the intention to commit a felony during the commission or attempted commission of which a death results.

F.

Model Penal Code A person is guilty of criminal homicide under the Model Code if he unjustifiably and inexcusably takes the life of another human being [MPC 210.0(1)] purposely, knowingly, recklessly, or negligently. [MPC 210.1(1)] The Code recognizes three forms of criminal homicide: murder, manslaughter, and (unlike the common law) negligent homicide. 1.

IX.

Murder A. Degrees of Murder At common law, there were no degrees of murder, and murder was a capital offense. Reform of the common law has resulted in the division of murder into degrees, with only murder in the first degree being a capital offense. 1. First Degree-Intentional killing of another human being with premeditation and malice aforethought (not out of passion) a. Deliberation i. Must show that D gave thought before acting ii. Reached a definite decision to kill-weighed consequences

iii. Intoxication as negating deliberation-If lost ability to deliberate or premeditate-defense to 1st degree; but not to murder; still be second-degree) b. Criteria i. Some appreciable time b/w formation of intent to kill and execution ii. Earlier hostility b/2 accused and victim iii. Self-interested motive iv. Manner and circumstances of killing (interruptions) v. Accused behavior before the killing (stalking) vi. Origin of the murder weapon (did he bring it) 2. Second Degree-Intentional Murder-Intent to kill establishes malice a. No premeditation b. Intent to Inflict Grievous Bodily Injury Malice aforethought is implied if a person intends to cause grievous bodily injury to another, but death results. In states that grade murder by degree, this form of malice nearly always constitutes second-degree murder. c. Extreme Recklessness (Depraved Heart Murder) Malice aforethought is implied if a persons conduct manifests an extreme indifference to the value of human life. In states that separate murder into degrees, this type of murder almost always constitutes second-degree murder. d. Mitigated Factors: Provocation, self-defense, coercion, reasonable belief of preventing imminent public disaster B. The Model Penal Code rejects the degrees-of-murder approach. C. Common Law: The common law definition of murder is the killing of a human being by another human being with malice aforethought. 1. As the term has developed, a person kills another acts with the requisite malice if he possesses any one of four states of mind: a. b. c. d. the intention to kill a human being; the intention to inflict grievous bodily injury on another; an extremely reckless disregard for the value of human life; or the intention to commit a felony during the commission or attempted commission of which a death results.

2. Intent to Kill a. Deliberate and Premeditated Typically, a murder involving the specific intent to kill is first-degree murder in jurisdictions that grade the offense by degrees if the homicide was also deliberate and premeditated. Wilful, Deliberate, Premeditated Nearly all states that grade murder by degrees provide that a wilful, deliberate, premeditated killing is murder in the first degree. Intent to Inflict Grievous Bodily Injury Malice aforethought is implied if a person intends to cause grievous bodily injury to another, but death results. In states that grade murder by degree, this form of malice nearly always constitutes second-degree murder. Extreme Recklessness (Depraved Heart Murder) Malice aforethought is implied if a persons conduct manifests an extreme indifference to the value of human life. In states that separate murder into degrees, this type of murder almost always constitutes second-degree murder. i. Conscious disregard of a substantial and unjustifiable risk to human life

b.

c.

d.

e.

f.

Can get raised to be murder if have exceptional level of recklessness a. When an action, unlawful in itself, is done w/deliberation; and w/intention of mishchief or great bodily harm, or of mischief indiscriminately and death ensues, against or beside the original intention of the party, it will be murder b. D acts solely from general malicious recklessness, disregarding any and all consequences, sufficient that the manifested a reckless, murderous disposition Year and a day rule: Originally provided that D may not be prosecuted for criminal homicide unless V dies within a year and a day of the act inflicting the fatal injury (today, rules not accepted in most jurisdictions) Deadly Weapon Rule i. D uses a deadly weapon directed at a vital part of the human body, intention to kill may properly be inferred

ii.

Model Penal Code A homicide is murder if the defendant intentionally takes a life, or if he acts with extreme recklessness (i.e., depraved heart murder). 1. D Purposely or knowingly takes a life w/o mitigating circumstances a. Actor must subjectively intended death or know death was almost certain to result from his action 2. Extreme recklessness a. Showing extreme indifference for human life b. Depraved heart murder E. WI 1. 1st degree intentional homicide a. Causes death of another with intent to kill that person or another i. Mental purpose to take the life of another human being ii. Or was practically certain his conduct would cause death a. Directly or indirectly from the facts b. Consider statements or conduct iii. w/o adequate provocation a. Something D reasonably believed the intended V had done b. Complete loss of self-control in an ordinary person c. Mitigated to 2nd degree if has adequate provocation iv. Intent can be formed in an instant, judged by the objective standard a. What a person of ordinary intelligence and prudence would have believed in the position of D under the circumstances existing at the time of the alleged offense 2. 2nd degree intentional homicide a. Causes the death of another w/the intent to kill that person or another b. No showing that there was not adequate provocation c. 1st degree mitigated to this if one of the following is present i. Adequate provocation ii. Unnecessary defensive force iii. Prevention of felony iv. Unreasonable belief force necessary to stop felony v. Coercion, necessity 3. 1st degree reckless homicide a. Recklessly causes the death of another under circumstances showing utter disregard for human life b. Causes the death of another under any of the following circumstances i. Makes, distributes, delivers illegal drugs that another uses and dies ii. Delivery doesnt have to be direct 4. 2nd degree reckless homicide a. Recklessly causes the death of another D.

F.

X.

Intentional Murder 1. Francis v. Franklin a. P escaped from custody with an officers pistol. Fled to a nearby house, demanded car keys, and man shut the door in his face and he shot twice killing the man. Jury instructions directed jury to presume an essential element. Unconstitutional under the Due Process Clause G. Premeditated Murder (First Degree) 1. United States v. Watson: Two significant pauses, when appellant immobilized officer and when the officer repeated his plea for his life. This afforded appellant time to premeditate and deliberate. 2. State v. Bingham-D proved to have strangled the victim, mentally retarded woman, to death. both time and fact of deliberation are required, and that a reasonably jury cannot infer deliberation simply from the length of time 3. Criteria of Premeditation a. Hostility between the accused the victim b. Self-interested motive c. Accuseds behavior before killing d. The manner and circumstances of the killing e. Origin of murder weapon. Manslaughter A. Common Law: Forms of Manslaughter Traditionally, three types of unlawful killings constitute manslaughter: 1. (1) an intentional killing committed in sudden heat of passion as the result of adequate provocation (voluntary manslaughter); 2. (2) an unintentional killing resulting from the commission of a lawful act done in an unlawful manner (involuntary manslaughter). This is akin to criminally negligent homicide. 3. (3) an unintentional killing that occurs during the commission or attempted commission of an unlawful act (involuntary manslaughter). This type of manslaughter is sometimes dubbed unlawful-act manslaughter, or if the killing occurred during the commission of a non-felony, misdemeanor-manslaughter. B. Provocation (Sudden Heat of Passion) 1. Elements of the Mitigating Factor Under common law principles, an intentional homicide committed in sudden heat of passion as the result of adequate provocation mitigates the offense to voluntary manslaughter. The common law defense contains four elements: a. (1) The defendant must have acted in heat of passion at the moment of the homicide. Passion has been interpreted to include any violent or intense emotion such as fear, jealousy, and desperation. (SUBJECTIVE) i. A person of exceptional restraint or maturity is held to a higher standard a. Police officer ii. No previous intention to kill iii. Heat of passion b. (2) The passion must have been the result of adequate provocation (reasonable person would have lose his self-control) Under the modern approach, it is up to the jury to determine what constitutes adequate provocation. Juries in such cases are typically instructed to apply an objective reasonable-person standard. i. Physical Attack ii. Mutual Combat iii. Threat of Physical Attack iv. Unlawful arrest v. Witnessing adultery c. The defendant must not have had a reasonable opportunity to cool off. i. Law presumes that provocation dissipates over time

d.

ii. Must be sudden anger, not cumulative effect There must be a causal link between the provocation, the passion, and the homicide.

C. Words as Adequate Provocation Surviving from the common law in most non-Model Penal Code jurisdictions is the rule that words alone do not constitute adequate provocation. However, a few courts allow the defense to be raised in the case of informational, but not insulting, words. Other courts have held open the possibility that insulting words may qualify in extreme circumstances. 1. Informational: Husbands wife reports her neighbor raped her; he goes next door and kills the rapist. Adequate provocation 2. MPC: The words alone rule does not apply in jurisdictions following the Model Penal Code. D. Unlawful-Act (Misdemeanor-Manslaughter) An accidental homicide that occurs during the commission of an unlawful act not amounting to a felony (or, at least, not amounting to felony that would trigger the felony-murder rule) constitutes involuntary manslaughter. This may be termed misdemeanor-manslaughter or unlawful-act manslaughter. 1. The scope of the doctrine varies widely by jurisdiction. Some courts limit its applicability to inherently dangerous misdemeanors while others apply the doctrine to all misdemeanors. E. Partial Justification 1. The provocation would have to come from the victim 2. Words themselves would never be adequate provocation 3. The victims defensive force against the killers initiating force could never be a provocation F. Partial Excuse 1. Killer must act immediately after provocation 2. Victims need not have been the cause of the provocation 3. Victims defensive force against killers initiations force could be a provocation 4. The killer may have been wholly, though understandably mistaken ascribing bad conduct to the victim G. Model Penal Code 1. In General A person is guilty of manslaughter if he: a. (1) recklessly kills another; or b. (2) kills another person under circumstances that would ordinarily constitute murder, but which homicide is committed as the result of extreme mental or emotional disturbance for which there is a reasonable explanation or excuse. 2. The reasonableness is determined from the standpoint of a person in the actors circumstances, as he believes them to be a. Identify actors personal characteristics (youth) b. Includes emotion of fear 3. The Code does not recognize any form of criminal homicide based on the unlawful-act (misdemeanor-manslaughter) rule. [MPC 6.06(2).] 4. Reckless and Criminally Negligent Homicide A person who kills another recklessly is guilty of manslaughter. In a sharp departure from the common law, the Code precludes liability for manslaughter based on criminal negligence. A criminally negligent homicide involuntary manslaughter at common law constitutes the lesser offense of negligent homicide under the Code. [MPC 210.4] 5. Extreme Mental or Emotional Disturbance (OBJECTIVE) A person who would be guilty of murder because he purposely or knowingly took a human life, or because he killed a person recklessly under circumstances manifesting an extreme indifference to the

value of human life, is guilty of the lesser offense of manslaughter if he killed the victim while suffering from an extreme mental or emotional disturbance (EMED) for which there is reasonable explanation or excuse. The reasonableness of the explanation or excuse regarding the EMED is determined from the viewpoint of a person in the defendants situation under the circumstances as he believes them to be. The concept of EMED is intended to incorporate two common law doctrines: (1) sudden heat of passion (but in a much expanded form); and (2) partial responsibility (diminished capacity). (D be required to prove beyond preponderance of evidence; P be required to disprove provocation beyond a reasonable doubt) a. The EMED manslaughter provision is broader than the common law provocation defense in the following ways i.

(1) a specific provocative act is not required to trigger the EMED defense; ii. (2) even if there is a provocation, it need not involve an injury, affront, or other provocative act perpetrated upon [the defendant] by the decedent; iii. (3) even if the decedent provoked the incident, it need not fall within any fixed category of provocations iv. (4) words alone can warrant a manslaughter instruction; v. (5) there is no rigid cooling-off rule. The suddenness requirement of the common law that the homicide must follow almost immediately after the provocation is absent from the EMED defense. 6. Subjective Standard-The person had to be adequately provoked (can be something somebody other than victim did) 7. WI: a. Only mitigates 1st degree to 2nd degree intentional b. Objective i. What a reasonable person ordinary prudence and control would have believed if they were in Ds situation as D would have interpreted it ii. Only reasonable if its something victim did c. Subjective: D must have actually been provoked Provocation Elements Common law 2 prongs Objective: cause a reasonable man to lose control, no opportunity to cool off Subjective: Acted in heat of passion that obscured reason, D did lose control Mitigation Other Murder to voluntary manslaughter Adultery as adequate provocation MPC Extreme emotional disturbance (EED) Subjective, viewpoint of person in Ds situation under the circumstances as D believes them to be No limit on what it is, just needs to negate the necessary mens rea Murder to manslaughter WI Adequatemust cause complete loss of self control in ordinary person under the circumstances Provocationsomething D reasonably believes intended V has done which causes D to lose self control completely at the time of causing death 1st degree homicide to 2nd degree Must be something V did D raises provocation defense, P disproves it

Doesnt have to be something V did D raises defense & P disproves it 8. Voluntary Manslaughter: intentional (different from involuntary which is unintentional); D has with knowledge or purpose killed another human being rather than having done so with gross negligence or recklessness

If murder in self-defense with unreasonable and mistaken belief; kills in face of actual threat of death or serious injury you invoked 9. People v. Walker a. D cuts Vs throat after V threatened them with a knife for not gambling with him i. Killed in excited, irrational state is manslaughter 10. Cooling Time a. Ex Parte Fraley-Court held that 10 months was sufficient time to cool ones passion; crime not mitigated b. Indirect Provocation-D killed murderer of his brother two hours after he learned of murder-might be adequate provocation c. Gradual Provocation-Cannot be accumulated; must be present anger d. Fear can be a adequate provocation 11. Adultery and other Adequate Provocation a. Rowland v. State i. and his wife were separated but on good terms and he visited her frequently. He came to visit one night and noticed another mans horse hitched ont eh fence. Upon entering the house, he caught his wife and the man in bed. They jumped out of the bed and ran past . tried to kill the man but killed his wife instead. ii. In Reed the court said that if a killed someone catching them in the act of adultery, this was not murder but manslaughter based on the common law rule that says that this situation would cause adequate provocation iii. NO difference if he shoots his wife or her lover, because there is passion still and it will still be deemed manslaughter b. Honor Defense-Gave juries an opportunity to nullify the manslaughter rule and acquit those husband-killers who claimed to kill to protect their owner c. Complete Defense-Entitled to JI on cultural background to determine mental state 12. Provocation-Modern Rules a. People v. Berry i. No adulterous act or words alone. Held there was long continued provocateur conduct UNINTENTIONAL HOMICIDE OFFENSES XI. Involuntary Manslaughter A. Death is accidential. B. Reckless Homicide 1. Common Law a. Reckless murder is normally the manslaughter category. b. A conscious disregard of any and all consequences; gross deviation from the standard of conduct of a law abiding person. c. Focuses on the actor, what is going on inside the actors, look to motivation. 2. Welansky: night club owner was charged with manslaughter by proving he created an unreasonable and substantial risk of death or great bodily harm. Massachusetts standard was that he should have known, thus they affirmed the conviction. 3. Mayes: court affirmed the conviction for murder even though he did not intend the actual result; his malicious recklessness, disregarding any and all consequences was enough. 4. Malone: affirmed 2nd degree murder conviction for playing Russian roulette. He showed an extreme indifference to human life. 5. Wisconsin a. First degree reckless homicide (940.02) i. (1) Very high degree of risk a. Requires actual knowledge of the risk ii. (2) Utter disregard of human life.

a.

b. c. d.

Second degree reckless homicide (940.06) i. Whoever recklessly causes the death of another human being. Could use provocation to mitigate from 1st degree reckless Levesque: couple accidentally started a fire in a warehouse, convicted of manslaughter. i. conviction of criminal recklessness in WI, the had actual knowledge of the fire, and knowingly left the building and did not take steps to stop the fire.

XII.

He would NOT be convicted of reckless homicide, because the standard requires actual knowledge AND conscious disregard b. Look to the actors extreme indifference to the value of human life. i. External circumstances (e.g. huge crowd of people) ii. Less emphasis on getting inside the actors head c. Eliminated intent to injure as a separate liability for murder. C. Negligent Homicide 1. Williams: couple charged with negligently failing to provide their son with medical attention. 2. Common Law a. Must be more than simple negligent, gross negligence is essential 3. Wisconsin a. Ordinary negligence to a high degree gross negligence 4. MPC D. Vehicular Homicide 1. Car related fatalities are so common in our society that many jurisdictions define a category of criminal homicide less severe than involuntary manslaughter, and sometimes negligent homicide. 2. Policy Considerations a. Invoking a general manslaughter charge exposes too large a percentage of the population to the risk of prosecution for a serious crime. b. Jurors tend to identify with careless drivers, even grossly negligent drivers. Thus, enacting a statute with a less penalty increases the likelihood that some punishment will result. 3. MPC: vehicular homicide falls under negligent homicide. 4. Watson: CA court found a drunk driver guilty of 2 nd degree murder; rejected vehicular manslaughter because they found implied malice. MANSLAUGHTER INVOLUNTARY A. Involuntary manslaughter based on criminal negligence: A person whose behavior is grossly negligent may be liable for involuntary manslaughter if his conduct results in the accidental death of another person. 1. Gross negligence required: Nearly all states hold that something more than ordinary tort negligence must be shown before D is liable for involuntary manslaughter. Most states require gross negligence. Usually, D must be shown to have disregarded a very substantial danger not just of bodily harm, but of serious bodily harm or death. a. Model Penal Code: The MPC requires that D act recklessly. (The MPC also requires that D be aware of the risk, as discussed below.) CLASS MPC must be aware of danger. WISCONSIN, 940.06., criminal negligence, 939.25, criminally reckless, 939.24. 2. All circumstances considered: The existence of gross negligence is to be measured in light of all the circumstances. The social utility of any objective D is trying to fulfill is part of the equation. Example: D kills V, a pedestrian, by driving at 50 mph in a 30 mph residential zone. Ds conduct may be grossly negligent if D was out for a pleasure spin, but not if D was rushing his critically ill wife to the hospital. 3. Inherently dangerous objects: Where D uses an object that is inherently dangerous, the courts are quicker to find him guilty of involuntary manslaughter. This is especially true where the accident involves a firearm.

6. MPC a.

4. Defendants awareness of risk: Courts are split as to whether D may be liable for manslaughter if he was unaware of the risk posed by his conduct. a. Awareness usually required: As noted, most states require D to have acted with gross negligence or recklessness. In these states, courts usually require that D have been actually aware of the danger. i. Model Penal Code agrees: The MPC, which requires recklessness for involuntary manslaughter, similarly requires actual awareness. Under the MPC, a person acts recklessly only when he consciously disregards a substantial and unjustifiable risk. 5. Victims contributory negligence: The fact that the victim was contributorily negligent is not a defense to manslaughter. (However, the victims negligence may tend to show that the accident was proximately caused by this action on the victims part, rather than by any gross negligence on Ds part.) 6. Vehicular homicide: Many states have defined the lesser crime of vehicular homicide, for cases in which death has occurred as the result of the defendants poor driving, but where the driving was not reckless or grossly negligent. (Most successful involuntary manslaughter cases also involve death by automobile.) a. Intoxication statutes: Also, some states have special statutes which make it a crime to cause death by driving while intoxicated. b. Criminally negligent homicide: Additionally, some states define the crime of criminally negligent homicide, whose penalties are typically less than the penalties for involuntary manslaughter. These statutes are not limited to vehicular deaths. (Example: The MPC defines the crime of negligent homicide, which covers cases where D behaves with gross negligence, but is not aware of the risk posed by his conduct.) B. The misdemeanor-manslaughter rule: Just as the felony-murder rule permits a murder conviction when a death occurs during the course of certain felonies, so the misdemeanormanslaughter rule permits a conviction for involuntary manslaughter when a death occurs accidentally during the commission of a misdemeanor or other unlawful act. 1. Most states apply: Most states continue to apply the misdemeanor manslaughter rule. 2. Substitute for criminal negligence: The theory behind the rule is that the unlawful act is treated as a substitute for criminal negligence (by analogy to the negligence per se doctrine in tort law). 3. Unlawful act defined: Any misdemeanor may serve as the basis for application of the misdemeanor-manslaughter doctrine. Also, some states permit the prosecution to show that D violated a local ordinance or administrative regulation. And if a particular felony does not suffice for the felony-murder rule (e.g., because it is not inherently dangerous to life), it may be used. a. Battery: The most common misdemeanor in misdemeanor manslaughter cases is battery. Example: D gets into an argument with V, and gives him a light tap on the chin with his fist. D intends only to stun V. Unbeknownst to D, V is a hemophiliac and bleeds to death. Since D has committed the misdemeanor of simple battery, and a death has resulted, he is guilty of manslaughter under the misdemeanor manslaughter rule. The same result would occur if as the result of the light tap, V fell and fatally hit his head on the sidewalk. b. Traffic violations: The violation of traffic laws is another frequent source of misdemeanor-manslaughter liability. Example: D fails to stop at a stop sign, and hits V, a pedestrian crossing at a crosswalk. V dies. Even if D does not have the gross negligence typically required for ordinary voluntary manslaughter, Ds violation of the traffic rule requiring that one stop at stop signs will be enough to make him guilty of manslaughter under the misdemeanormanslaughter rule. 4. Causation: There must be a causal relation between the violation and the death. a. Malum in se: In the case of a violation that is malum in se (dangerous in itself, such as driving at an excessive speed), the requisite causal relationship is often found so long as the violation is the cause in fact of the death, even though it was not natural and probable or even foreseeable that the death

would occur. That is, in malum in se cases, the usual requirement of proximate cause is often suspended. b. Malum prohibitum: But if Ds offense is malum prohibitum, (i.e., not dangerous in itself, but simply in violation of a public welfare regulation), most states do require a showing that the violation was the proximate cause of the death. i. Natural or foreseeable result: Some courts impose a requirement of proximate cause by holding that the death must be the natural or foreseeable consequence of the unlawful conduct. (Example: D fails to renew his drivers license, and then runs over V, a pedestrian. A court might well hold that since failure to renew a drivers license is malum prohibitum, and since Vs death was not a natural or probable consequence of Ds failure, D is not guilty under the misdemeanor-manslaughter rule.) ii. Violation irrelevant: Other courts simply do not apply the misdemeanor-manslaughter rule at all to conduct that is malum prohibitum Ds conduct must be shown to amount to actual criminal negligence, just as if there had been no violation. 5. Model Penal Code abolishes: The Model Penal Code rejects the misdemeanormanslaughter rule in its entirety. However, under the MPC, the fact that an act is unlawful may be evidence that the act was reckless (the Codes mens rea for manslaughter). C. Unintentional Homicide D. Class notes 1. aggravated recklessness v. ordinary reckless homicide a. depraived mind murder b. difference between first and second degree reckless homicide c. creation of risk, recognizing that the risk is there d. more than an unreasonable risk, a very high degree of unreasonable risk e. knowledge the actor in the creation of risk f. social utility of risk someone takes XIII. Summary of Provocation A CL PROVOCATION 1 Elements a Acted in heat of passion b Adequate provocation c Actor didnt have reasonable opportunity to cool off d Causal connection b/w first 3 2 Mitigates murder to voluntary manslaughter (not exoneration) 3 Objective a Cause a reasonable man to lose control b Fraley: 9 mos after incident, D shoots V and kills him (i) A reasonable person would have cooled off in 9 months 4 Subjective a Actor acts in heat of passion (i) Rage, fear, strong emotion obscuring reason b D did lose control 5 Need subjective and objective prong to prove it existed 6 Rowland: D walked in on wife with V adulterer, fired gun at adulterer and killed wife a Adultery = adequate provocation 7 Cumulative a Some cts rule that no cumulative provocation exists b Is cumulative effect, so cooling time wont start until last provocative event 8 Theories of mitigation to manslaughter a Partial justification (i) Excessive reaction to a plausible reason for force

(ii) Not justified, but not murder Partial excuse (i) Recognition that D was so enraged at time that his actions were not as voluntary B MPC PROVOCATION (210.3) 1 Extreme emotional disturbance a D must show that EED existed, prosecution has to persuade that didnt exist b Determined as reasonable from actors point of view c Viewpoint of a person in the actors situation under the circumstances as he believes them to be (i) Account for age, sex, physical disability, etc. d Just needs to negate mens rea, no limit on what is provocation (i) Specific provocative act not required to trigger the EED defense, need not involve an injury, affront, or other provocative act perpetrated upon D by decedent (ii) Doesnt have to be something V did e No cooling off time 2 Mitigates murder to manslaughter 3 Berry: D strangles V after she taunts him for weeks about her other lover in Israel a This constitutes extreme emotional disturbance, D should be convicted of manslaughter b Long course of provocation reached its final culmination in the apartment when V began screaming, D was in heat of passion under an uncontrollable rage C WI PROVOCATION ( 939.44) 1 Adequate = must cause complete loss of self control in ordinary person under the circumstances 2 Provocation = something which D reasonably believes intended V has done which causes D to lack self control completely at the time of causing death 3 Only mitigates 1st degree homicide to 2nd degree homicide 4 D has to raise the provocation defense, some evidence to support that circumstance, burden on prosecution to disprove the provocation XIV. Felony-Murder A. Common Law At common law, a person is guilty of murder if he kills another person during the commission or attempted commission of any felony. Nearly every state retains the felonymurder rule. B. Statutory Law Under most modern murder statutes, a death that results from the commission of an enumerated felony (usually a dangerous felony, such as arson, rape, robbery, or burglary) constitutes first-degree murder for which the maximum penalty is death or life imprisonment. If a death results from the commission of an unspecified felony, it is second-degree murder. The felony-murder rule authorizes strict liability for a death that results from commission of a felony. C. Model Penal Code The Code also provides for felony-murder by setting forth that extreme recklessness (and, thus, murder) is presumed if the homicide occurs while the defendant is engaged in, or is an accomplice in, the commission, attempted commission, or flight from one of the dangerous felonies specified in the statute. [MPC 210.2(1)(b)] 1. Limits on the Felony-Murder Rule a. Inherently-Dangerous-Felony Limitation Many states limit the rule to homicides that occur during the commission of felonies which by their nature are dangerous to human life, e.g., armed robbery. b. Independent Felony (or Merger) Limitation Most states recognize some form of independent felony or collateral felony limitation. That is, the felony-murder rule only applies if the predicate felony is independent of, or collateral to, the homicide. If the felony is not independent, then the felony merges with the homicide and cannot serve as the basis for a felony-murder conviction. For example, most jurisdictions hold that felonious assault may not serve as the basis for felony-murder c. Res Gestae Requirement A requirement of the felony-murder rule is that the homicide must occur within the res gestae [things done to commit] of the felony, which requires both: i. temporal and geographical proximity There must be a close proximity in terms of time and distance between the felony and the homicide. The res gestae period begins when the defendant has reached b

ii. XV.

the point at which he could be prosecuted for an attempt to commit the felony, and it continues at least until all of the elements of the crime are completed. Most courts provide that the res gestae of a felony continues, even after commission of the crime, until the felon reaches a place of temporary safety. a causal relationship between the felony and the homicide.

FELONY-MURDER A. Generally: Under the felony-murder rule, if D, while he is in the process of committing certain felonies, kills another (even accidentally), the killing is murder. In other words, the intent to commit any of certain felonies (unrelated to homicide) is sufficient to meet the mens rea requirement for murder. 1. Common law and today: The felony-murder rule was applied at common law, and continues to be applied by most states today. 2. Example: D, while carrying a loaded gun, decides to rob V, a pedestrian. While D is pointing his gun at V and demanding money, the gun accidentally goes off, and kills V. Even though D never intended to kill V or even shoot at him, D is guilty of murder, because the killing occurred while D was in the course of carrying out a dangerous felony. 3. WISCONSIN 939.05 940.04, felony murder in WI felony murder is a strict liability crime. B. Dangerous felonies: Nearly all courts and legislatures today restrict application of the felonymurder doctrine to certain felonies. 1. Inherently dangerous felonies: Most courts today use the inherently dangerous test only those felonies which are inherently dangerous to life and health count, for purposes of the felony-murder rule. 2. Two standards: Courts are split about how to determine whether a felony is inherently dangerous. Some courts judge dangerousness in the abstract (e.g., by asking whether larceny is in general a dangerous crime), whereas others evaluate the felony based on the facts of that particular case (so that if, say, the particular larceny in question is committed in a very dangerous manner, the felony is inherently dangerous even though most other larcenies are not physically dangerous). 3. b. Listing: In courts that judge inherent dangerousness in the abstract, here are felonies that are typically considered inherently dangerous: robbery, burglary, rape, arson, assault and kidnapping. By contrast, the various theft-related felonies are generally not considered inherently dangerous: larceny, embezzlement and false pretenses C. Causal relationship: There must be a causal relationship between the felony and the killing. First, the felony must in some sense be the but for cause of the killing. Second, the felony must be the proximate cause of the killing. 1. Natural and probable consequences: The requirement of proximate cause here is usually expressed by saying that D is only liable where the death is the natural and probable consequence of Ds conduct. 2. Robberies and gunfights: Most commonly, proximate cause questions arise in the case of robberies. a. Robber fires shot: felony-murder doctrine should apply. i. Example 1: On a city street, D points a gun at V, and says, Your money or your life. While V is reaching into his pocket for his wallet, D drops his gun. The gun strikes the pavement and goes off accidentally, killing V. Ds acts of robbery are clearly the proximate cause of Vs death, and D is guilty of murder under the felony-murder rule. Example 2: Same facts as above example. Now, assume that when the gun strikes the pavement and goes off, it kills B, a bystander 20 feet away. Ds acts are the proximate cause of Bs death, so D is guilty of murdering B under the felony-murder doctrine. b. Victim or police officer kills bystander: courts are split as to whether the robber is the proximate cause of the death. California, for instance, does not apply the felony-murder doctrine in any situation where the fatal shot comes

from the gun of a person other than the robber. In other states, the result might depend on whether the robber fired the first shot, so that if the first shot was fired by the victim and struck a bystander, the robber would not be guilty. c. Robber dies, shot by victim, police officer or other felon: Where the person who dies is one of the robbers, and the fatal shot is fired by another robber, the robbery victim or by police officers, courts are even more reluctant to apply the felony-murder doctrine. Some courts hold that the felony-murder doctrine is intended to protect only innocent persons, so it should not apply where a robber is killed. Where a robber is killed not by one of his cohorts but by the robbery victim or the police, the case for applying the felony murder rule is the weakest of all. 3. In furtherance test: In most courts, all of the co-felons are liable for a killing committed by one of them, if the killing was: (1) committed in furtherance of the felony; and (2) a natural and probable result of the felony. a. Accidental killing: Thus one felon will commonly be guilty of murder based on another felons accidental killing. Example: A and B decide to rob a convenience store together. A carries no gun. A knows that B is carrying a loaded gun, but also knows that B has never used a gun in similar robberies in the past, and that B does not believe in doing so. During the robbery, B accidentally drops the gun, and the gun goes off when it hits the floor, killing V, the convenience store operator. Because B was holding the gun in furtherance of the robbery when he dropped it, and because an accident involving a loaded gun is a somewhat natural and probable consequence of carrying the loaded gun during the felony, there is a good chance that the court will hold not only that B is guilty of felony-murder, but that A is also guilty of felony murder as an accomplice to Bs act of felony murder. b. Intentional killing: Similarly, if the killing by one co-felon is intentional rather than accidental, the other co-felons will probably still be liable under accomplice principles as long as the killing was committed in furtherance of the felony. This will normally be true even though the other co-felons can show that they did not desire or foresee the killing. But if the other co-felons can show that the killing was not committed for the purpose of furthering the felony, they may be able to escape accomplice liability. Example: A and B rob a convenience store together; as A knows, B is carrying a loaded gun, but B has never used the gun on any previous robberies and is generally opposed to violence. Unknown to either, the new owner of the store is V, an old enemy of Bs. B decides to shoot V to death during the course of the robbery, even though V is not threatening to call the police or resisting the robbery in any way. A will have a good chance of persuading the court that the killing was not in furtherance of the robbery, and thus of escaping accomplice liability for felony-murder. D. In commission of a felony: The felony-murder doctrine applies only to killings which occur in the commission of a felony. 1. Causal: There must be a causal relationship between felony and killing 2. Escape as part of felony: If the killing occurs while the felons are attempting to escape, it will probably be held to have occurred in the commission of the felony, at least if it occurred reasonably close, both in time and place, to the felony itself. 3. Killing before felony: Even if the killing occurs before the accompanying felony, the felony-murder doctrine will apply if the killing was in some way in furtherance of the felony. Example: D intends to rape V. In order to quiet her, he puts his hand over her mouth, thereby asphyxiating her. D is almost certainly liable for felony-murder, even though he killed V before he tried to rape her, and even though the final felony was only an attempted rape (since one cannot rape a corpse). E. Felony must be independent of the killing: For application of the felony murder doctrine, the felony must be independent of the killing. This prevents the felony-murder rule from turning virtually any attack that culminates in death into automatic murder.

F.

G. H.

I.

J.

K.

1. Example 1: D kills V in a heat of passion, under circumstances that would justify a conviction of voluntary manslaughter but not murder. Even though manslaughter is obviously a dangerous felony, the felony-murder rule will not apply to upgrade the manslaughter to felony-murder. The reason is that the underlying felony must be independent of the killing, a requirement not satisfied here. 2. Example 2: D intends to punch V in the jaw, but not to seriously injure him or kill him. V, while falling from the blow, hits his head on the curb and dies. Even though D was committing the dangerous felony of assault or battery, this will not be upgraded to felony-murder, because the felony was not independent of the killing. Model Penal Code approach: The Model Penal Code does not adopt the felony-murder rule per se. Instead, the MPC establishes a rebuttable presumption of recklessness...manifesting extreme indifference to the value of life where D is engaged in or an accomplice to robbery, rape, arson, burglary, kidnapping or felonious escape. Thus if an unintentional killing occurs during one of these crimes, the prosecution gets to the jury on the issue of depraved heart murder. But D is free to rebut the presumption that he acted with reckless indifference to the value of human life. The MPC provision is thus quite different from the usual felony-murder provision, by which D is automatically guilty of murder even if he can show that he was not reckless with respect to the risk of death. Rationale Common Law 1. Old: transfer of intent 2. New: strict liability, the actor is held culpable despite mental state, and will be deterred from future felonies. MPC 1. Eliminates felony murder rule 2. Maintains a presumption of recklessness when homicide occurs during certain felonies (arson) 3. D must rebut this, or presumption supports murder. Wisconsin 1. Predicate offense; essentially strict liability 2. Need NOT be foreseeable 3. Specific offense limitations within the statute; beyond this NO limits. So long as you are committing one of the listed crimes and death occurs; conviction stands. a. State v. Oimen: Convicted of felony murder State v. Martin 1. went to a party with some of his friends, and he had consumed a lot of alcohol and smoked marijuana. and his friends were kicked out of the party. says he basically was trying to make a mess of the garbage(possibly, because he was mad about being thrown out), and lit it on fire. claims he never intended the fire tospread, that he only wanted to make a mess. But the fire did spread, and someone dies. However, according tostate's experts, the fire was set by spreading kerosene between the ground floor & second floor (so it looksreally intentional). found guilty of felony murder (and arson). The court ended up reversing due to a bad juryinstruction.Holding 2. Trial Court jury instruction said:Under the felony murder law, it does not matter whether the act which caused death iscommitted recklessly or unintentionally or accidentally. The perpetrator is as guilty as hewould be if he has purposefully or knowingly committed the act which caused the death. 3. Court says the trial court failed to instruct the jury that would not be liable for the felony murder of the victim if her death was "too remote, accidental in its occurrence, or too dependant on another'svolitional act." Reversed & remanded. 4. Court discussing felony Murder: a. At CL, if the victim died during course of a felony, also guilty of murder via transferred intent. If there's an intent to commit the felony, even in the absence of an intent to kill, that intent wastransferred to the death of the victim. Now, felony murder not really viewed as transferred intent, butof strict or absolute liability. Reasoning is that it serves as a deterrent against the commission

of violent crimes. If a potential felon knows they may be culpable also as murderers, they will be lesslikely to commit the felony b. Mens rea requirement & felony murder - felony murder holds actor liable for murder even though nomens rea - no actual intent to murder, only for the felony. However, reasoning on why it's acceptedis that felonies are disproportionately dangerous. Murders happen often during the commission of afelony, so law wants to prevent such deaths by making it undesirable to commit a felony. So then theactor is liable not only for foreseeable consequences, but also, even if not foreseeable to the actor himself (objective), those consequences that the legislature apprehends c. CL vs. MPC (This jurisdiction's felony murder statute is based on the MPC) d. At CL felony murder treated as an absolute liability offense. e. Govt's burden to prove both cause-in-fact and also that the felony was a proximatecause of the victim's death. f. The death must be a probable consequence of the commission of the felony g. Probable consequence - refers to the risk of death created by the felon's acts h. The MPC creates the presumption of recklessness when a homicide occurred in the course of the commission of certain felonies. If not rebutted, that presumption would support aconviction of murder. i. MPC meaning of probable consequences - related to concept of foreseeability. Somedeaths that occur in the course of a felony are too remotely related or accidental towarrant holding the actor liable. i. Ex: too remote - the moment a bank robber steps into the bank, an employee pushes the button for a burglar alarm and is electrocuted. ii. Ex: not too remote and is foreseeable/a probable consequence (often usedinterchangeably) - robs a store, shopkeeper fires at the , but kills aninnocent bystander. would be guilty of felony murder. L. People v. Gladman i. robbed a deli, and walked through the neighborhood to a bowling alley. Police were notified of the robbery, and when the saw an officer, he hid under a parked car. When the officer approached the car, the got out and shot the officer, killing him. The issue was whether the murder happened during the escape of a felony, in order to apply the felonymurder rule. ii. Holding: Court says question for the jury if the question of whether the was in immediate flight from the felony if it is not clear. Jury should take the circumstances into consideration - interval of time between felony & the murder, whether had the fruits of the felony in possession, whether was in close pursuit by police or others, & whether had reached a place of temporary safety. No single factor is controlling, and list is not exclusive. iii. Court holds that the jury could properly find, with these facts, that the was in immediate flight - has cash & was trying to get away, he hadn't reached a place of safety yet, and police were looking for him. Doesnt matter that the officer may not have known was there or that he was the felon, all that matters is that believed it. iv. Court here doesnt lay down rigid standards to determine the duration of the felony - they want flexibility; depends on the circumstances. v. Class Notes vi. Court's standard on whether or not this was felony murder:

vii. The scope of the felony event - Concept of time and proximity of death to the felony - can't be too far away viii. Court says must have been in immediate flight from the felony - jury will decide (standard-based b. LIMITATIONS on felony murder rules i. IL v. Hickman: Where D sets in motion the chain of events leading to death, hold D responsible for the killing ii. CA v. Washington: killing committed by 3d person not attributable to robber 1. Not enough that killing is just a risk, need to show intent to robber himself iii. WI v. Oimen: if have intent for predicate offense, then felony murder is SL offense and need not be foreseeable iv. Gladman: asportation issue 1. Flight/escape from scene 15 minutes later is unclear, might now work as felony murder 2. Continuation of asportation phase, so felony still continuing c. FELONY MURDER i. CL 1. Guilty of murder if D kills during commission or attempted commission of a felony a. Usually felonies dangerous to human life (arson, robbery, rape, burglary) b. Usually does not apply if an adversary to the crime, rather than a felon, personally commits the homicidal act c. When felon kills innocent person, co-felons also responsible for the shooting d. Felons attempt to commit a forcible felony sets in motion a chain of events which were or should have been in his contemplation when the motion was initiated, felon should be held responsible for any death which results from the sequence 2. Intent to commit felony transferred to death of V 3. Strict liability offense, so may serve as deterrent against violent crime 4. Martin: need a homicidal intent along with the commission of the felony a. Ct should instruct the jury that D, whether a sole actor or an accomplice, is liable for felony murder only if the death is not too remote, accidental in its occurrence, or too dependent on another's volitional act to have a just bearing on D's culpability ii. MPC 1. Not proponent of SL offenses, so not real felony murder rule here a. Extreme recklessness can be presumed in the commission, attempt to, or flight from commission of a felony b. Murder for any death, even complete accident occurring in course of commission or flight from a felony c. Accomplices to felony as accomplices to murder iii. WI (940.03) 1. Cause the death of another while committing or attempting to commit armed robbery, armed burglary, arson, 1st deg. sex. assault, etc. a. Still uses WI proximate cause substantial factor test 2. Includes flight from felony 3. Applies to accomplices if participating in felony and when V shoots co-felons 4. Oimen: V shot a co-felon as O was driving off w/o others in the getaway car a. Strict liability offense, doesnt require any mental element except to the predicate offense b. Need not be foreseeable II. Felony murder Common law MPC WI Felon sets in motion a chain Extreme recklessness can be Cause death of another Elements of events which were or presumed in the commission while committing, should have been in his of, attempt to, or flight from attempting to commit, or

Intent

contemplation when the motion was initiated, held responsible for any death which results from the sequence Intent to commit felony transferred to death of V

commission of a felony

flight from felonies dangerous to human life Need not be foreseeable

Co-felons responsible? 3rd person causes death a.

Yes D not responsible

Intent to commit felony doesnt transfer to intent to kill, but recklessness presumed Yes

Intend to commit the crime, transfers

Yes D & co-felons responsible

VEHICULAR HOMICIDE i. Less severe than involuntary manslaughter or negligent homicide for those who accidentally kill while operating a vehicle (MPC 210.4) b. MURDEROUS ANIMALS i. Life endangered conduct only done w/awareness that conduct is contrary to laws of society ii. Sufficient to prove could reasonably foresee the animals could attack/injure someone iii. CL Could be intent to do serious bodily injury to substitute for intent to kill, but w/MPC there must be intent to kill c. ASSISTED SUICIDE i. Active participation in final overt act is murder ii. Just participating in events leading up to it is assisted suicide III. JUSTIFICATION AND EXCUSE i. Justification v. excuse 1. Justification a. Complete defense, D did no wrong (conduct) i. Must be reasonable belief about need for deadly force ii. Focus on circumstances and actions iii. Subjective and objective elements, what actor was thinking and what a reasonable person would have been thinking 2. Excuse a. D not responsible for crime committed b. Focus on actor, like insanity or incompetence (belief) c. Can be a complete defense 7.01 Justification Defenses A justification defense deems conduct that is otherwise criminal to be socially acceptable and non-punishable under the specific circumstances of the case. Justification focuses on the nature of the conduct under the circumstances. Examples include: Self-defense Defense of others Defense of property and habitation Use of lawful force Necessity 7.02 Excuse Defenses Excuse defenses focus on the defendants moral culpability or his ability to possess the requisite mens rea. An excuse defense recognizes that the defendant has caused some social harm but that he should not be blamed or punished for such harm.

Examples include: Duress Insanity Diminished capacity Intoxication (in very limited circumstances) Mistake of fact Mistake of law (in very limited circumstances) IV. DEFENSIVE FORCE Defensive force Elements Common law Imminent attack, response necessary and proportional, goal to thwart attack Yes, but not from home/business Yes MPC D held to mental state of what D thought was true about immediacy and necessity of force Yes Yes WI Reasonably believe unlawful attack, response necessary/proportional Yes Yes

Retreat? Aggressor rule? Orig. aggressor cant claim SD unless remove and inform Defense of others?

Must be family, marriage, employment relationship

Can defend anyone w/amount of force aggressor used (goes w/WIs duties to act)

D is required to raise self-defense as an affirmative defense. Once the D successfully raises the defense, the prosecution is required to disprove the defense BARD. 3 factor test o (1) Imminence The time for the defensive act is NOW, D cannot wait any longer w/o being harmed. Does NOT include preemptive strike e.g. killing an abusive husband while he is asleep is not an imminent threat. o (2) Ds response must be necessary and proportional Necessity Must show that each of the 5 shots were required. Cannot be an effective response, less drastic. Words alone are not enough, must be accompanied by conduct. Proportionality Amount of force used must be proportional to the threat received. Tenn. v. Garner: feeling from a minor crime does not allow deadly force from officer. Deadly force may not be used unless it is necessary to prevent escape and the officer has probable cause to believe that the suspect poses a significant threat of death or serious bodily injury. o (3) Ds response must be with intent to thwart the attack Response must be to thwart, NOT to hurt the attacker. Perfect Self Defense o Complete Defense to the crime, entitled to acquittal. All 3 factors = JUSTIFICATION Reasonable Mistake = EXCUSE o Perfect self defense to a charge of 1st degree intentional homicide, actor must (1) reasonably believe that she was preventing or terminating an unlawful interference with her person and

(2) reasonably believe that the force she used was necessary to prevent imminent death or great bodily harm. Requires objective reasonableness! o La Voie: mild-tempered pharmacist, no conviction; killing was justified. The Ds threatened deadly or grievous bodily harm, or he reasonably perceived them as doing so. Imperfect Self Defense o Unreasonable Mistake = mitigates the crime to manslaughter or negligent homicide o Imperfect self defense (unnecessary defensive force): D must show (1) some evidence that she actually (subjectively) believed that she was in imminent danger of death or great bodily harm and (2) actual belief that the force used was necessary to defend herself; even if both are unreasonable. Regaining Defensive Force o Gleghorn Gleghorn initiated the deadly attack; he is not justified in beating attackers to death b/c he presented the initial threat. o Wisconsin 939.48(2)(b) D required to attempt to retreat Aggressor rule applies Defense of others Can defend anyone (goes along w/WIs duty to act) Only amount of force the aggressor used Reasonable Self-Defender o Battered Woman Syndrome o Leidholm: (case of the battered spouse) In N.D. they used a subjective standard which was affected by battered woman syndrome. No justification because she did not have all 3 factors. Her beliefs were reasonable so she has an excuse. Use subjective standard, easier to prove battered woman. Some states have allowed self-defense claims on battered women even if the husband was asleep at the time of killing; however, no courts have allowed this defense in cases where a battered wife has hired a killer. o What is a reasonable belief? Subjective Look to the whether the circumstances are sufficient to induce the accused an honest and reasonable belief that he must use force to defend himself against imminent harm. Allow the jury to assume the physical and psychological properties unique to the accused; place yourself in the shoes of the accused. MPC o Purely subjective belief, defensive force is justifiable when the actor believes that such force is necessary. o Only consider what the defender believes (not reasonable defender). The belief does not have to be reasonable. o Treat self-defense as a mistake of fact. Mitigated (kicked down to reckless/negligence) with a reckless or negligent belief. o Harm must be immediately necessary, replaces imminence. o May permit self-defense sooner than under common law. Objective View the surrounding circumstances at the time the accused used force from the standpoint of the hypothetical reasonable, prudent person. Unique physical characteristics are NOT taken into account. Subjective/Objective Goetz: D killed 4 boys on the subway who tried to steal his money. He did not have a justification (no imminence). He had an excuse, because they

kept the objective (reasonable) standard but added a subjective (actors belief) standard. Looked to whether Goetz (actor) reasonably believed he was in imminent threat of serious bodily harm. Wisconsin o Combines objective and subjective standards o person reasonably believes o No mistake of fact analysis like MPC

Duty to Retreat o Traditionally, the retreat rule had created a clear exception for attack in ones own residential premises. A person assaulted in his own dwelling need not retreat before using deadly force, even if a safe opportunity exits. o Must be able to retreat to safety. o Common Law: No duty to retreat in your own home. Must be imminent fear of seriously bodily harm. Minority position today. o Wisconsin Maintains the duty to retreat. o MPC Incorporated into 3.04(2)(b)(ii) Agree this duty is falling out of favor; likely to remain a factor for juries, not bright line duty. a b c d e La Voie: D reasonably in fear for safety may use deadly self defense Gleghorn: aggressor rule: original aggressor cannot claim self-defense unless remove and inform Leidholm: battered spouse, may be defense based on subjective belief Goetz: subjective and objective belief, if unreasonable, may mitigate murder to manslaughter Duty to retreat (i) If can safely retreat, then use of force isnt necessary (ii) Not required to retreat from home/place of business (iii) Only exists in some jurisdictions

Summary

INSANITY Insanity ALI test? Federal law Modifiedno concern w/whether D was able to conform conduct to the law MPC D must suffer mental disease/defect that renders D unable to appreciate right from wrong OR unable to conform Ds conduct to the law On state to disprove insanity BARD WI D must suffer mental disease/defect that renders D unable to appreciate right from wrong OR unable to conform Ds conduct to the law On D to prove insanity by preponderance of the evidence

Burden on whom?

On D to prove insanity by clear and convincing evidence

Rationale Choice o Criminal law believes that people act the way they do because they choose to. Look to their acts to determine intent; its too difficult to gets inside someones mind. Insanity Defense is Vague o Difficult to measure when a person ought to be held liable for their choices o Conflicts with the goal of deterrence; the public sees a guilty actor getting away with it. Criminal law vs. Mental Health o Blame vs. healing

The prevailing clinical understanding of mental health is not useful in the criminal arena. Mental health is therapeutic and looks to heal; they dont look to moral wrongs and whether they are too sick to heal. Merely have a disease/defect does not automatically render an actor irresponsible for committing the offense. Must answer specific questions of the test in the affirmative. Two Prong Test o (1) Did the D suffer from a mental disease or defect? Jury decision. No. Then no further questions. Yes. (2) Did the disease/defect render the D incapable of telling right from wrong? OR Did the disease /defect render the D incapable of conforming his behavior to the requirements of the law? o Either the person was confused about what they were doing e.g. thought they were changing the channels on the TV with a remote but were killing with a gun. o Or, the D knew the conduct was wrong but could not help but listen to the voices in his head and commit the crime. o Guilt Phase Prosecution has the burden to prove BARD that the D committed the crime. If D pleads guilty, they waive this right and move to the second phase. o 2nd Phase of a Bifurcated Trial D has the burden of proof to show they qualify for the insanity test by a preponderance of the evidence WI uses MPC tests? o Dahmer argued mental disease (necrophilia) that rendered him incapable of conforming his behavior to the law. (2nd prong). o CLASH OF LEGAL/MEDICAL PROFESSIONALS Blame vs. healing 3 ways of looking at mental problems Disease o Brain chemistry Dimension o Axes and characteristics that put us on those axes Psychological explanation o Determinist viewpoint, like Freud, old-school psych stuff o Ignores blame o MPC ALI test D must suffer mental disease or defect that renders D o Unable to appreciate right from wrong; OR o Unable to conform conduct to the law Burden on the state to disprove insanity BARD o FEDERAL LAW Modified ALI test D must suffer mental disease or defect that renders D unable to appreciate right from wrong Not concerned with whether D was able to conform his conduct to the law Burden on D to prove not guilty by reason of insanity by clear and convincing evidence o WI (971.15) ALI test w/shifted burden Bifurcated, guilt then insanity phases D must suffer mental disease or defect that renders D o Unable to appreciate right from wrong; OR

o o

o Unable to conform conduct to the law Burden on D to prove not guilty by reason of insanity by preponderance of the evidence COMPETENCY TO STAND TRIAL Focuses on Ds ability to assist in his own defense He must be competent to stand trial in order to be tried for criminal conduct Jeffrey Dahmer found sane, couldnt prove that he was insane P claims D doesnt suffer from mental disease If he does have one, should have been able to conform his behavior to that required by law o Showed control, planning, knew what he was doing was wrong by the way he did it D claims Mental disease of paraphilia of necrophilia o Sexual obsession, love of death o Completely compliant partner Couldnt conform his behavior to that required by law, couldnt help himself Evidence Eliminated evidence of the murders o Selection of Vs Street hustlers/gay prostitutes Had been kicked out of their own homes, had nowhere to go and no one to watch out for them Had no one with them, were alone at the bars o Getting rid of bodies Carve them up, triple bagged in garbage bags, put in dumpster right before it was collected, otherwise animals would come, or would soak them in acid and flush it down the toilet Preserved some body parts by boiling them in preserving fluid o Making it difficult to trace him when he went home Only left at end of evening with them Didnt use his own name, didnt talk to many people Took cabs and had them drop him off 4-5 blocks away from his home Could only do what he did when he was drunk Waited until he got them back to his place o Tried to do things in bathhouse or hotel, but was too messy, people would have suspected something ALI test to be found not guilty by reason of mental disease or defect, a D must suffer from a mental disease or defect, and this disease or defect must render the D either (defense burden by clear and convincing evidence) unable to appreciate the wrongfulness of his/her act or unable to conform his/her conduct to the requirements of the law WISCONSIN 971.15 Mental Responsibility of Defendant (1) a person is not responsible for criminal conduct if at the time of such conduct as a result of mental disease or defect the person lacked substantial capacity either to appreciate the wrongfulness of his or her conduct or conform his or her conduct to the requirements of the law. (2) the terms mental disease or defect do not include an abnormality manifested only be repeated criminal or otherwise antisocial conduct (3) mental disease or defect excluding responsibility is an affirmative defense which the D must establish to a reasonable certainty by the greater weight of the credible evidence.

V.

Attempt

VI.

Attempt Common law Proximity testD has w/in power to complete crime almost immediately Beyond prep, overt act Traditionally, no. Now yes if voluntary/complete renunciation Yes, circumstances must make it legally impossible to commit the offense Intent to do overt act in furtherance of crime Intent to commit actual crime No attempts for felony murder/reck./neg. crimes Historically misdemeanors, now 50% MPC Substantial step testDs conduct strongly corroborates criminal intent Lying in wait, enticement, recon work Yes if complete/voluntary No. Purposely engage in conduct that constitutes a crime if the circumstances are as D believes them to be Ds purpose to commit the crime WI Unequivocality testDs conduct, standing alone, unambiguously manifests intent to commit the crime Yes if reason for it is not an extraneous factor (must be voluntary) No

Conduct

Abandonment?

Circumstances (Impossibility?)

Mens rea

Intent to perform the act Intent to commit the crime

Grading VII.

Same level as completed crime

Punished at 50%

ATTEMPT a. CL i. Specific intent crime, even if the target offense is a general intent crime ii. Conduct 1. Proximity test a. Attempt does not arise unless an actor has it w/in her power to complete the crime almost immediately b. Look at how close Ds conduct is to completing the crime i. Must go beyond preparation ii. Requires overt act iii. Where D shoots at someone and the gun doesnt work or D misses 2. Abandonment traditionally not a defense a. Today, applies only if D voluntarily and completely renounces criminal purpose i. Voluntary = when result of repentance or a genuine change of heart 1. Not voluntary if D motivated by circumstance that might increase the likelihood of arrest or unsuccessful consummation of the offense ii. Complete: not if the actor merely postpones the criminal endeavor until a better opportunity presents itself iii. Circumstances 1. Legal impossibility as a defense a. Circumstances must make it legally impossible to commit the offense 2. Booth: cannot attempt to receive stolen property if property is not actually stolen 3. Hybrid legal impossibility a. Ds goal is illegal, but commission of the offense is impossible due to a factual mistake regarding the legal status of some attendant circumstance that constitutes an element of the charged offense iv. Mens rea 1. Must have 2 intentions: a. Intent to do overt act in furtherance of a crime, and b. Intent to commit the actual crime 2. Cannot attempt felony murder a. Attempted murder requires specific intent to kill; Ds intent to commit a felony doesnt substitute for the intent to kill

3.

Even if a person by committing a felony acts recklessly, illogical to say that D intended to kill V by being reckless (for below too) Lyerla: cannot attempt a reckless or negligent crime a. So no involuntary manslaughter b/c no intent to kill, cant intend to commit unintentional crime

b.

b.

c.

v. Grading 1. Historically, attempts are misdemeanors 2. Today, attempt to commit felony = felony, but lesser than actual crime, often punished at 50% MPC i. Conduct 1. Substantial step test: ( 5.01) (subjective test) a. Under the circumstances, does Ds conduct strongly corroborate the criminal intent b. Examples of substantial steps i. Lying in wait or following V ii. Enticing V iii. Reconnaissance of location iv. Unlawful entry v. Possession of unlawful materials under the circumstances vi. Soliciting an innocent agent to engage in crime 2. Limited abandonment defense allowed if its complete and voluntary a. Renunciation of criminal purpose b. Not guilty if: i. D abandons effort to commit the crime or prevents it from being committed, and ii. Ds conduct manifests a complete and voluntary renunciation of her criminal purpose 1. Similar to CL here ii. Circumstances 1. No impossibility defense a. D guilty if she purposely engaged in conduct that would constitute a crime if the attendant circumstances were as D believes them to be 2. Is pure legal impossibility defense? a. Not going to charge you w/crime if you didnt actually commit a crime iii. Mens rea 1. Must be Ds purpose to commit the crime a. Although the mens rea of purpose or belief does not necessarily encompass the attendant circumstances of the crime i. Here, sufficient that the actor possesses the degree of culpability required to commit the target offense 2. Allows knowingly if considering result iv. Grading 1. Punishes attempt at same level as target offense 2. Except felony in 1st degree becomes attempted felony in 2d degree WI i. Conduct 1. Unequivocality/probable desistence test (b/w CLs proximity and MPCs substantial step tests) a. Ds conduct, standing alone, unambiguously manifests the intent to commit a crime b. Looking at frames of a movie: the next frame is D committing the act c. Looking at Ds conduct, can we say that the conduct indicates BARD that D is going to commit the crime or is unlikely to desist from committing the crime d. If equivocal, if can go either way, then havent met the test

If shoot a gun that doesnt work, under CL not guilty, under MPC and WI b/c only reason it didnt work was b/c the gun didnt work 3. Abandonment a. P must first establish that there unequivocally was an attempt b. Reason for abandonment cannot be an extraneous factor, not voluntary ii. Circumstances 1. No impossibility defense, look at Ds subjective mind 2. Subjectively, still goes to mind and what you wanted the circumstance to be iii. Mens rea 1. Intent to perform the act 2. Intent to commit the crime iv. Grading 1. 50% of punishment for target crimes 2. Exception: child sex crimesstatutes spell out actions and punishment for attempt VIII. Attempt X 2 ATTEMPT 2. IX. X. XI. (1) moving from mere preparation to attempt (2) impossibility Actus Reus/ Conduct Tests a. Common Law i. Proximity: the over act required for attempt must be proximate to the completed crime, or directly tending toward the completion of the crime, or must amount to the commencement of the consummation. Must be very close to the commission. ii. Narrow scope of attempt liability iii. Murray 1. D ran away with his niece, took her to a chapel, and he was waiting for someone to perform the incestuous ceremony. Court held this was not close enough to the completed crime. Possible conviction if they were in the middle of their vows. 2. Convictions under MPC substantial step, and WI unequivocality test. iv. Rizzo 1. Court reversed conviction for attempted robbery. No conviction under proximity test, not close enough. 2. MPC and WI would both sustain convictions under this fact pattern. b. Wisconsin i. Unequivocality/probable desistance: attempt is committed when the actors conduct manifests an intent to commit a crime. ii. Film Frame Analysis: if the move stopped just before the crime happened, what would the next frames show? Look at conduct, not words. iii. If its unequivocal, it could go either way, and we do not have attempt. c. MPC i. Substantial Step: one who engages in such purposive conduct is sufficiently dangerous to justify state intervention, even if she is not yet close to consummation of the offense. ii. Broadens the scope of attempt liability iii. strongly corroborative of the criminal purpose iv. Factors to consider 1. Lying in wait 2. Searching for, or following the contemplated victim of the crime 3. Unlawful entry into a structure or building in which the crime will be committed 4. Possession of the materials to commit the offense v. McQuirter 1. Black man convicted of assault with intent to rape. The court affirms the conviction using the substantial step test (lying in wait). Seems to be a weak case and MPCs test has a broad scope. vi. E.g. Erin gives a gun to Kelley, so that Kelley can kill Natalie. Kelley, however, does NOT attempt to commit the offense. Kelley is NOT guilty of attempted murder. Erin is guilty under

XII.

P K R N

XIII.

MPC of attempted murder. She is guilty b/c if Kelley had attempted the crime, Erin would have been an accomplice in the attempt, by furnishing the gun for the purpose of homicide. vii. Rationale: A person who attempts to aid in the commission of an offense is as dangerous as one who successfully aids in its commission or attempted commission. Under subjectivist principles, her conduct justifies punishment. Culpability Requirements a. Common Law/MPC i. Conduct purpose ii. Result purpose iii. Circumstance actor must possess the degree of culpability required to commit target offense Conduct Result Circumstance CL/MPC CL/MPC* except if result CL has occurred WI WI/ x MPC- as required by target offense No Reckless attempt X X No negligent attempt X X iv. E.g. a fire started from someone elses cigarette, actor did not do anything to stop the fire b/c they want the insurance money. She did not have the purpose to start the fire, but she purposely took a substantial step towards the commission of a crime thus she is charged as attempt to commit arson. v. Morales 1. D was charged with violating a statute which makes it a crime for any deported person to enter, or attempt to re-enter, the U.S. w/o express permission of the Attorney General. Court affirmed attempt conviction because this was a general intent crime, which he only needed to purposely re-enter, not have purpose to violate the law. vi. Markowski 1. D was HIV-positive and tried to sell blood to a plasma donation center. D was acquitted b/c he sold him blood only to make money, did not have the purpose mens rea to hurt anyone. vii. Jackson 1. Ds conviction is upheld, used the MPC substantial step test. Have guns, handcuffs, masks, and evidence of them surveying the area twice. 2. No conviction with proximity, not close enough. 3. Probably a conviction under WI test, would have gone through with it had they not been caught. viii. Reckless Endangerment 211.2 1. MPC recognized the gap they left b/w a purposeful attempt and the great number of crimes with a lesser mens rea. 2. They filled this gap with a reckless endangerment provision. 3. Gap filler for a reckless attempt. b. Wisconsin: need intentional conduct i. No reckless or negligent attempts! Attempt is a specific intent crime, must be P or K. ii. WI uses other statutes to holds these people liable. 1. e.g. 941.20(3)(a) Endangering Safety by the use of a Dangerous Weapon 2. whoever intentionally discharges a firearm from a vehicle while on a highway, or in a vehicle parking lot that is open to the public under any of the following circumstances... Abandonment a. Applied very narrowly; very few cases won on abandonment. b. Common law: traditionally NOT a defense i. Staples: D devised to rob a bank below a room he rented. He began drilling through the floor. Conviction with the substantial step and unequivocality test. D withdrew, but the court affirmed the conviction. After the drilling the attempt is complete, NO going back.

c.

XIV.

XV.

MPC i. Limited abandonment defense permitted if complete and voluntary renunciation of criminal purpose. Not a voluntary abandonment if motivated by difficulties encountered, or if motivated by a decision to postpone. Impossibility a. Legal Impossibility (circumstance) i. If the act were completed it would not be criminal. ii. DEFENSE, only under common law iii. Booth 1. The coat lost its stolen status, this constituted a legal impossibility and precluded D from conviction for attempt to receive stolen goods. Had the attempt been carried out successfully, this would NOT be a crime. iv. Sobrilski: Drug sales to an undercover officer, not subject to legal impossibility defense because of a drug abuse act which eliminated it. v. Modern Attempt 1. Dlugash a. Two men shot the victim, D shot the victim after he was already dead but he believed him to be alive. Court affirmed the conviction for attempted murder because a murder would have been committed had the attendant circumstances been as D believed them to be. 2. Thousand a. Rule that legal and factual impossibility are indistinguishable and abolished impossibility as a defense. Prosecution only needed to prove that D attempted to commit an offense prohibited by law, coupled with conduct toward the commission of the offense. b. Factual Impossibility (conduct or result) i. NO DEFENSE ii. Where the basic crime is impossible to complete b/c of some physical or factual condition unknown to the D iii. e.g. a man putting his hand into a pocket intending to steal, but the pocket is empty. iv. e.g. D who shoots into the intended victims bed, believing he is there but he is not. v. e.g. where the D erroneously believes that the gun is loaded points it at his wifes head and pulls the trigger. c. Wisconsin i. WI does not distinguish b/w legal and factual impossibility, neither are a defense. ii. Kordts: motorcycle case, he is guilty of attempt in WI. This is because he attempted to commit a crime with intent to perform and attain a resultif accomplished would constitute a crime. Even though motorcycle does not have a stolen status anymore, no legal impossibility defense. Attempt a. Introduction i. COMMON LAW an attempt occurs when a person, with the intent to commit an offense, performs any act that constitutes a substantial step toward the commission of that offense. ii. two varieties of attempt 1. complete (but imperfect) attempts: actor completes all necessary preparations, but fails in completeing the act. Actor accomplishes everything but the result 2. Incomplete attempts: the actor has not completed all necessary preparations, and quits or is prevented from continuing his course. iii. Subjectivism (MPC): Determination of guilt for attempts should be based primarily on mens rea. Any act, no matter how innocuous, justifies punishment for the inchoate crime. iv. Objectivism: Focus should be on actus reus. Conduct should not be punished unless its criminality is objectively discernable at the time that it occurs. Criminality is articulated by the conduct alone. b. Punishment for Attempt i. COMMON LAW:

c.

d.

Attempts are misdemeanors at common law, except for attempted felonies, which are graded as felonies but punished less severly, fraction approach. ii. MODEL PENAL CODE 1. Attempts are of the same grade and degree as the completed act. Attempts at felonies in the first-degree are graded as second-degree felonies 2. Judicial discretion: Judge is allowed to reduce the punishment if attempt could not possibly succeed. (e.g. sinking a ship w/ a BB gun) iii. WIS. STAT. 939.32: 1. Attempted felonies and the three misdemeanors listed (940.19, battery, 940.195, battery to unborn child, 943.20, theft) are the be punished at no more than the maximum penalty for the completed act (WI buys into the fraction approach, unlike MPC) Other misdemeanors are to be punished at the same level; life imprisonment felonies are to be punished as Class B felonies. Mens Rea of Attempt i. COMMON LAW: 1. An attempt is a specific-intent offense (even if the substantive crime is a general intent offense) in that one must act with a purpose to engage in the conduct or to bring about the result (mens rea for an attempt is higher than the completed act because there must exist an intent to commit the crime). 2. State v. Lyerla Lyerla was charged with attempted second-degree murder; D fired shots in the direction of a passing van, killing one person. To attempt second-degree murder, actor must intend to have a criminally reckless state of mind. ii. MPC 5.01 ( Hardly any states follow because of difficult mens rea) 1. Distinction must be made: a. Completed conduct crimes mens rea elevated to purpose for conduct that would constitute the crime if the attendant circumstances were as he thought them to be. b. Complete result crimes mens rea elevated to purpose to cause the result or a belief that the result will occur. c. ALL incomplete crimes mens rea elevated to purpose (substantial step approach if the actor has taken a substantial step toward commission of the crime). iii. WIS. STAT. 939.32(3) 1. Actor must have the intent to produce result or intent to act in a way that would accomplish result; must have either purpose to do thing, purpose to bring about result, or must be practically certain that result would follow (from definition of intent). 2. Under WI law, D in Lyerla likely would have been found guilty of first-degree murder because intent includes awareness of practical certainty 3. NOTE: Common law and MPC allow for the conviction of attempted recklessness and attempted negligence. There is nothing within these systems that expressly forbids this apparently illogical result. In contrast, a D in WI cannot be convicted of an attempt when the completed version involves something less than intent. The rationale is that one cannot intentionally commit an unintentional act. For example, one cannot be charged with attempted reckless homicide; one would instead be charged with reckless endangerment, etc. Actus Reus of Attempts i. Issue: When has D crossed the line from preparation to perpetration? ii. Subjectivists favor an early attachment of guilt (focus on mens rea), while objectivists favor a later attachment of guilt (focus on actus reus). iii. Common Law Tests: 1. Last Proximate Act Test (People v Murray, p 765) A criminal attempt only occurs when a D has performed all of the acts that he/she believed were necessary to commit the offense. EX: no attempted murder until trigger pulled, no attempted theft until property is in the process of being stolen. Attachment of guilt as late as it gets.

1.

e.

f.

Physical Proximity Test: An actor must be in close proximity to where the crime is to occur; D must have power to complete the crimes almost immediately. Attachment of guilt middle to late in process. 3. Dangerous Proximity Test (People v Rizzo, p 772): D guilty of attempt when conduct is in dangerous proximity to success or when act is so near to result that the danger of success is very great: act must be immediately and not remotely connected with the commission of an offense. Attachment of guilt middle to late in process, but earlier than physical proximity test: Three factors a. Nearness of danger b. Greatness of harm c. Degree of apprehension felt by the victim 4. Unequivocality Test (res ipsa Loquitur): Attempt occurs when Ds conduct, standing alone, unambiguously manifests her criminal intent. stop the film look at movie without sound and determine if the actions alone indicate attempt. Attachment of guilt late in process. 5. Probably Desistance Test: attempt if the D reached a point where it was highly unlikely that he would have voluntarily desisted from his effort to commit the crime (point of no return). Attachment of guilt middle to late in process, somewhat similar to dangerous proximity test. 6. Indispensable Element test: an attempt occurs once an actor has gained control of an indispensable element of the crime. Usually the element is an instrumentality vital to the commission of a crime, such as gun for murder. Test is rather arbitrary, but attachment of guilt tends to be somewhat early in the process. iv. Model Penal Code: 1. Substantial Step Test: 5.01(1)(c) provides that one is not guilty of an attempt unless one has taken a substantial step toward commission of the crime. Conduct must strongly corroborate the actors criminal purpose. Attachment of guilt is very early in the process. v. WIS. STAT. 939.32(3) 1. Provides a blend of the unequivocality tests. Attachment of guilt is late in the process. Two questions to answer: a. Does the actor have the intent to perform the acts and attain a result which, if accompanied, would constitute a crime? b. Has the actor performed acts that unequivocally demonstrate that intent? 2. Key question has the actor done enough to demonstrate his intent? Abandonment as a Defense to Attempt Liability i. Issue: Can defendant avoid conviction for an attempt if he abandons his criminal conduct before consummation of the target offense? ii. COMMON LAW: abandonment is NOT a defense. iii. MODEL PENAL CODE 5.01(4): allows for the defense as long as it is voluntary. The MPC calls the defense renunciation of criminal purpose. Two parts: 1. Actor abandons efforts to commit the crime OR prevents it from being committed; AND 2. Actors conduct manifests a complete and voluntary renunciation of criminal purpose. Complete renunciation means the actor does not intend to postpone the criminal conduct or to transfer the effort to another objective or victim. Voluntary renunciation mean the actor does NOT abandon his effort based on changed circumstances that make it more difficult to commit the offense, or that make it more likely that he will be caught. iv. WISCONSIN: abandonment is NOT a defense The Odyssey of Impossibility as a Defense to Attempt Liability i. A crime cannot be completed because it is either factually impossible or legally impossible. Thus, the actor has the requisite mens rea, but not the requisite actus reus. ***This is the mirror image of mistakes of fact and mistakes of law, in which the actor has the requisite actus reus but not the requisite mens rea.***

2.

Factual impossibility: The actor has the requisite mens rea, but does not have the requisite actus reus due to a lack of attendant circumstance unknown to the actor or out of the actors control (ex. Picking an empty pocket, shooting into an empty bed where the intended victim usually sleeps). Had the circumstances been as the actor believed them to be, or hoped that they were, the crime would have been consummated. 2. Legal Impossibility: The completed act would not be criminal. The actor makes a mistake as to the legal status of an element of the crime (ex. Receiving unstolen coat). The actor makes an inculpatory mistake (i.e. a guilty mind mistake). ii. COMMON LAW: factual impossibility is NOT a defense. However, legal impossibility is a defense. iii. MODEL PENAL CODE 5.01: provides that neither factual not legal impossibility is a defense. This is in accord with subjectivist principles. 1. Note that only in incomplete attempts is the actors conduct required to be strongly corroborative of the actors intent. In the more common complete attempts, corroboration of the actors criminal purpose is NOT required. The danger of convicting a person for innocuous conduct, then, is unmitigated. 2. The Codes does, however, grant the trial judge the authority to dismiss a prosecution of or reduce the sentence of an inchoate offense that is inherently impossible. That is, an actors conduct is such that he does not poase a reasonable threat to society (ex. Trying to sink a battleship with a pop gun.) iv. WISCONSIN: neither factual nor legal impossibility is a defense. This is established through case law, not by statute.

1.

Punishable attempts Punishment level

Actus reus

Mens Rea

Relationship to target offense (merger rule) Attempts of reck./neg. crimes? Abandonment defense

COMMON LAW Felonies and misdemeanors Misdemeanor, except for felonies, which are punished at a fraction Six possible tests: Last act test Physical Proximity test Dangerous proximity test Equivocality test Probable desistance test Indispensable Element test Dual: (1) intent to commit acts that constitute the actus reus; and (2) the specific-intent to engage in the conduct or bring about the result NOTE: Knowledge might/might not be sufficient for #1 Attempt merges with completed offense Conduct crimes, yes Result crimes, only if voluntary No defense

MODEL PENAL CODE All crimes Same as completed offense, except 1st degree becomes 2nd degree Substantial step test

WIS. STAT. 939.32 Felonies and three misdemeanors No more than completed offense, with some exceptions Combination of equivocality tests: gone so far that intent unequivocal; unlikely to desist

Complete conduct crimes = purpose to engage in conduct Complete result = purpose to cause result OR belief that result will occur Incomplete crimes =purpose plus a substantial step Attempt merges with completed offense Yes

Conduct crimes=purpose to engage in conduct Result crimes=purpose to cause result OR awareness that result is practically certain to occur

Attempt merges with completed offense No

Yes, if complete and voluntary and either effort abandoned, or commission

No defense

Impossibility defense (legal, factual) Requirements

Factual no, legal yes

(1) specific-intent to engage in conduct or bring about a result; and (2) An act in furtherance of that intent.

prevented Neither, but judge can reduce penalty or dismiss for inherent impossibility (1) purpose to engage in conduct; or purpose to cause a result or belief that a result will occur; and (2) an act in furtherance of that intent.

Neither

(1) purpose to engage in conduct; or purpose to cause a result or awareness that a result is substantially certain to occur; and (2) an act in furtherance of that intent.

XVI. Act

g. Complicity Common law Knowingly/actually assist perp No Yes MPC Aid/agree to aid/attempt to aid perp in planning/committing offense No Yes (Or solicit another or omit to act when have duty) Knowledge of perps criminal intent Conduct: Purpose of promote/facilitate Result: culpability level required according to statute Not liable Can be different than perp Yes if neutralize/prevent Yes if purpose to aid another in commission of offense and that assistance would have made her an accomplice in the commission of the crime under the complicity statute if the offense had been committed/attempted WI Knowingly aid/abet in commission of crime Yes Yes

Perp knows about aid Perp knows about abet

Mens rea As to accomplices acts As to target offense Natl/prob. conseq. Punishment Abandonment? Attempt to aid

Intent to assist perp Knowledge of perps criminal purpose Purpose of committing/encouraging/facilitating crime

Knowledge of perps criminal intent Knowledge for aiding/abetting, purpose for conspiracy

Liable Can be same as perp, could be more Yes if communicate and neutralize Must be some overt act/agreement to aid

Liable

Yes (probably if neutralize/prevent) Must be some overt act/agreement to aid

XVII.

COMPLICITY a. GENERAL CONCEPT i. Complicity is not a distinct crime, but a way to commit a crime ii. Not necessary that another person be found guilty of the offense 1. Can find accessories guilty when dont have perp iii. Accomplice can be guilty even if perp is excused unless the accomplice can come up w/an excuse as well 1. Accomplice can be liable for more serious crime than would the perp b. COMMON LAW i. Act

ii. iii.

iv. v.

vi.

vii.

Must knowingly and actually assist the perp in the commission of an offense Can be minimal assistance, as long as not just a bystander a. Perp must know about abetting/encouraging b. Does not have to know about accomplices actual aid 3. Any aid, no matter how trivial, suffices a. Even if aid was causally unnecessary to commit the crime 4. Types a. Physical conduct (procuring, driving getaway car) b. Psychological influence (advising/encouraging) c. Omission (if have a duty to act) Accomplice liable for natural and probable consequences of perps action in committing the offense Mens rea 1. Intent to assist the primary party to engage in the conduct that forms the basis of the offense; and 2. Mental state required for commission of the offense, provided in statutory definition 3. As to accomplices conduct: knowledge of perpetrators criminal purpose a. Know what perp is going to do b. Know what accomplice is doing by aiding perp 4. As to target offense: must have purpose of committing, encouraging, or facilitation of the offense a. Accomplice has purpose of transaction that 2d party would sell to 3d party, but did not have explicit purpose that 3d person would sell to 4th person b. Except from Backun ct i. Decided on knowledgeif I sell a gun to someone whom I know wants to commit murder, I cant wash my hands b/c I dont have purpose that murder occur 5. As long as accomplice acts w/the purpose of assisting perp in the conduct that constitutes the offense (and has the level of culpability required as to the prohibited result) he should be deemed an accomplice if his culpability as to the attendant circumstances would be sufficient to convict him as a principal a. Mens rea policies regarding the substantive offense should control the accomplices situation 6. Liability for crimes of recklessness/negligence a. Accomplice liability allowed as long as the accomplice has the 2 mental states described above: intent to assist perp to engage in the conduct that forms the basis for the offense and the mental state required for the commission of the substantive offense Historically, principal needed to be convicted before an accomplice 1. Not true anymore Punishment 1. Can be same as principal or for lesser offense 2. Accomplice can be charged w/greater offense in certain instances Defenses 1. Abandonment defense allowable a. Spontaneous and unannounced w/drawal will not suffice b. Accomplice must communicate w/drawal to perp and make efforts to neutralize the effect of his prior assistance 2. P acquitted b/c actions were justified, S should also be acquitted in absence of wrongdoing by P, no crime to impute to S 3. P acquitted b/c of excuse (insanity, duress, etc.), that wont bar conviction for S who does not have that excuse (excuses are always personal to the actor) Attempt to aid 1. Where do nothing is not enough, must be some overt act or agreement to aid for there to be attempt to aid 2. In most cases, accomplice is a co-conspirator and a co-conspirator is an accomplice

1. 2.

c.

MPC i. Conduct 1. Aid/agree to aid/attempt to aid perp in planning/committing the offense; or a. Perp does not need to know if the accessory gives actual aid or attempts to aid i. Perp does need knowledge of accessorys encouragement ii. Un-communicated attempt to aid qualifies as complicity iii. Liable for aiding even if accessory does not fulfill promise of agreement to aid 2. Solicits another to commit the crime; or 3. Has legal duty to prevent offense but chooses not to act ii. Mens rea 1. As to accomplices conduct: knowledge, know of perps criminal intent a. For conduct elements and result elements of the crime 2. As to the target offense a. Conduct elements: accomplice has purpose of promoting or facilitating the offense b. Result elements: whatever culpability level required for criminal liability according to elements of the crime i. Must act w/culpability required for result of crime, usually the same as the perps ii. Dont elevate mental state to purpose for result crimes under MPC iii. Participating in a crime as a feigned accomplice to entrap the perp is not complicity iii. No natural and probable consequences doctrine 1. Focus on actors own mental culpability 2. Cannot prove that accomplice intended for something that may have been foreseeable or not down the road iv. Perp need not be convicted/charged w/same crime 1. Accessory can be held liable w/an innocent/insane perp b/c only need subjective mens rea of accessory v. Victim cannot be accomplice vi. Abandonment defense exists 1. Must first neutralize his assistance, give timely warning to the police of the impending offense, or attempt to prevent the commission of the crime vii. Attempt to aid 1. D may be convicted of a criminal attempt although a crime was neither committed nor attempted by another if: a. The purpose of Ds conduct is to aid another in the commission of the offense, and b. Such assistance would have made her an accomplice in the commission of the crime under the complicity statute if the offense had been committed or attempted

d.

WI i. Conduct 1. Knowingly aid/abet in commission of the crime 2. Is ready and willing to assist and perp knows D is willing to assist ii. Mens rea 1. As to accomplices conduct: knowledge, know of perps criminal intent 2. As to target offense: knowledge for aiding/abetting, purpose for conspiracy 3. Difficulty is that case law says knowledge, when JI says purpose iii. Perp needs to know that accessory is aiding perp 1. Un-communicated attempt to aid is not being an accomplice 2. Attempt where do nothing is not enough, must be some overt act or agreement to aid for there to be attempt to aid iv. Natural and probable consequences of intended crime

In light of ordinary experience, the result is expected (not extraordinary nor surprising) 2. Probability that 1 crime will result from another is: a. Judged by facts and circumstances known to accessory at the time the events occurred 3. Can find accessory guilty if accessory knew, or reasonable person in accessorys position would have known, that the crime charged was likely to result from commission of the intended crime v. Abandonment exists in WI

1.

XVIII. CONSPIRACY a. ELEMENTS i. 1+ conspirators must have committed a crime ii. 2+ conspirators must have agreed to commit a crime iii. Parties must have intended as the object of their agreement the commission of an unlawful act 1. Must be a purpose standard, not enough to have knowing 2. Substantive crime iv. Different from aiding/abetting 1. Can aid/abet w/o ever having an agreement XIX. Need agreement for conspiracy XX. Complexity and Conspiracy X 2 COMPLICITY Accomplices are held liable for aiding, abetting, and encouraging the offense of another. NOT a distinct crime, it is a way of committing a crime. Liability depends on the accomplices relationship to the principal. Can aid and abet reckless and negligent crimes! o Two people, two mental states. Conduct of handing an intoxicated person a gun. knowledge Intoxicated person takes the gun, and shoots someone. reckless MPC o A person is an accomplice of the another person in the commission of an offense if, with the purpose of promoting or facilitating the commission of the offense, he solicits the other person to commit it, or aids or agrees to attempts to aid the other person in planning or committing it, or (having a legal duty to prevent the crime) fails to make the proper effort to prevent it. The Accessorial Act (1) Must establish the perpetrator committed an offense. (2) Then what the acts or omissions were that the perpetrator did. Common Law o The principal MUST be found guilty in order for accomplices to be held liable. o A child under age 10 cannot commit the crime, not mens rea. o Perpetrator not guilty accomplices not guilty o Ochoa The intent to kill, or aid and abet in the commission thereof, may be formed at the scene of the crime, even though the accused may have gone there w/o such criminal intention. Mere presence, if unaccompanied by outward expression of such approval is insufficient. o Mere presence is not enough Gains: It is not necessary that the aider or abettor be physically present aiding and abetting his partner or partners in the crime. However, he must be sufficiently near or so situated as to aid or encourage or to render assistance to the actual perpetrator. Tally: D may be any distance from the commission of the crime, he is present in the legal sense if he is 1. at the time performing any act in furtherance of the crime 2. is in a position to give info to the principal, or 3. prevents others from doing any act which would warn the intended victim

o o

Inaction as abetting Walden: D convicted as accomplice b/c she was present when her boyfriend struck her 1 year old son with a belt. Court upheld the conviction holding that failure of a parent who is present to take all steps reasonably possible to protect the parents child from an attack by another person constitutes an act of omission by the parent showing the parents consent and contribution to the crime being committed.

Accomplice is held liable even if the principal is not found guilty. Although, the state must prove that the acts constituting the crime occurred. o Accessory must 1. aid and encourage 2. agree to aid 3. attempt to aid* o The accessory is guilty as accomplice even if the principal does not have knowledge of the aiding and abetting. e.g. bank robbery: person is outside holding a gun but never needs to use the gun or do anything; the bank robber does not know she is there. She is still guilty as an accomplice. Substantial Step Test: it is a substantial step towards the crime, even if the bank robber does not know the accomplice is outside the bank with a gun. Under Wisconsin, the accomplice is NOT guilty because there were not physical acts, so the principal needs knowledge of the accomplices presence. o Accessory after the fact plays no role in the prep or commission of the crime, but assists the felon in eluding capture or destroying evidence. Must be intentional, rather than mere failure to notify authorities Wisconsin o Theory 1 (939.05) As a matter of objective fact, the accessory to a crime has to aid or abet in the commission of the crime, through physical action or verbal conduct. o Require purpose to aid and abet the commission of the crime. o Can become an accomplice without actual physical acts. o WI does NOT include attempt to aide. (1) Action is not required, but the principal must have knowledge or awareness of the willingness to aid the commission of the offense. e.g. A tells bank robber she is going to act as a lookout, but then remembers she needs to call her mom and doesnt act as a lookout. Still guilty under 939.05, giving the bank robber knowledge of her as an accomplice, she does not need physical acts. (2) If accomplice is physically acting to aid and abet, the principal does NOT need to have knowledge. o Theory 2 (939.31) Conspire with another to commit a crime Cant be charged with conspiracy to commit a crime AND complete separate crime of conspiracy. MENS REA OF COMPLICITY Common Law o Accomplices Conduct: most jurisdictions adopted the purpose standard for conduct Beeman: reversed conviction b/c JI should have instructed on purpose standard Peoni: D sold counterfeit money to Regno, who then sold it to Dorsey. Judge Hand reversed the conviction which charged Peoni as an accessory; he needed purpose but only had knowledge. Wilson: Wilson suspected Pierce of stealing his watch, helped him commit a burglary (conduct offense) and then called the cops. He had knowledge as to the target offense, the court held that he need purpose. MPC o Accomplices Conduct: need purpose for conduct o Target Offense: must have purpose of committing, encouraging, or facilitating the offense

MPC o

Result Crimes 2.06 (4) Whatever the mens rea is for the result crime, that is the mens rea required for accomplice. e.g. reckless homicide: recklessness e.g. intentional homicide: purpose or knowledge e.g. negligent homicide: negligence Etzweiler: Should have been convicted under MPC, court screwed up on the holding. Should have required a negligent mens rea, BUT the court required a purposeful mens rea. Wisconsin o Accomplices Conduct: requires knowledge for conduct However, JI requires purpose o Target Offense Result Crimes WI State v. Hohenstein Hohenstein hands a drunk Macy a gun and he shoot s and kills someone. This is kicked down to recklessness under the WI intoxication statute. His handing the gun was reckless, and he aided and abetted the crime. Target crime: shooting a gun while drunk intentional crime in WI Thus, he aids and abets this crime. Is the result expected? Yes, its not extraordinary or surprising. A reasonable person in Hohensteins position would have realized that Macy would shoot and kill someone. If the accessory is also drunk, can he form the intent to aide and abet? NO knowledge/intent if he is also drunk. You can intentionally aide and abet in a reckless crime in WI? Accessory must be at least knowing or purposeful as to the result!! Relations of Parties (1) The Perpetrator is Excused o A perpetrator may be guilty even if the principal is excused (insanity, duress, reasonable mistake of law), unless the accomplice can avail herself of the same excuse. (2) The Perpetrator is Justified o If Jones, a sane person, aids Smith, an insane person, when Smith is irrationally attacking Green, Jones certainly cannot escape liability simply b/c Smith is excused on grounds of insanity. (3) The Perpetrator Lacks Mens Rea o When the actor is both the but-for cause and the proximate cause of the proscribed conduct, the law will impute criminal liability to one person for the innocent actions of another. liable as a principal, under the doctrine of perpetrator-by-means. (4) Discrepant Mens Rea o The trend in courts has been to permit discrepant liability based on differential mens rea where the evidence warrants it. o McCoy: If the mens rea of the aider and abettor is more culpable than the actual perpetrators, the aider and abettor may be guilty of a more serious crime than the actual perpetrator. A jury could have found that Lakey did not act under unreasonable self-defense even if McCoy did. Thus, his conviction of murder and attempted murder can stand, notwithstanding that on retrial McCoy might be convicted of a lesser crime or even acquitted. (5) One of the Parties Lacks a Required Status for the Crime o Many jurisdictions sustains convictions in this situation o Simplot (Wisconsin): court refused to extend the parents immunity to an accomplice. o The Perjury Examples Sadacca: Court of appeals upheld the conviction. The court found the requisite causation factor was satisfied by Ds use of an innocent agent to commit a crime. CONSPIRACY Inchoate Crime

Like attempt, it punishes anticipatory action that aims at, but does not necessarily ever reach, a criminal object. The conspiracy is itself a crime. Doctrine of Accessorial Liability This doctrine implicates all the co-conspirators in each others acts. Agreement to commit an unlawful act was criminal conspiracy. Elements (1) One or more of the conspirators must have committed a crime. (2) Two or more of the conspirators must have agreed to commit the crime. (3) The parties must have intended as an object of their agreement the commission of the an unlawful act (mens rea) How is this different from aiding and abetting? Must agree before the crime. WI 939.31 a substantive crime. Cepeda The court overturned the conviction. Here, there was no evidence of a sale. Appellants intent to enter into a conspiracy cannot be inferred from the presence and her mere possession of paraphernalia usable, and previously used in drug cutting. XVI. Complicity A. Introduction 1. Accomplice defined: One is an accomplice in the commission of an offense if he intentionally assists (aids, abets, encourages, solicits, advises, etc.) another to engage in the conduct that constitutes the crime. 2. Types of Assistance: a. Physical b. Psychological (includes solicitation) c. Omission (when duty to act exists) B. The Accessorial Act 1. COMMON LAW: Accomplices are liable for the target offense (if completed), an attempt at the target offense, consequential crimes (if completed), and attempts at the consequential crimes. 2. The common law casts the net of liability to the following four groups: a. Principal in the first-degree: The actor who directly commits the crime, or the one responsible for the use of an innocent instrumentality (non-human agent or non-culpable human agent) to commit the crime. b. Principal in the second-degree: An accomplice who assists in the commission of a crime, and is either actually (immediate presence) or constructively (close by) present. (situated in a position to assist the principal during the commission of the crime, e.g., getaway car driver) c. Accessory before the fact: An accomplice who assists in the commission of the crime who is not present (like principal in second-degree but without actual or constructive presence). This group includes solicitors, counselors, etc. those who assist but remain in the background. d. Accessory after the fact: With knowledge of anothers guilt, intentionally assists the felon to avoid arrest, trial, or conviction. Only takes place when all acts constituting the crime have ceased. 3. Accomplice liability does not kick in unless a crime has been committed. 4. Defendant can be in more than one category, although he will only face one charge per crime. 5. Accomplices need to be placed in the correct categories for prosecution. If not, they will be acquitted. 6. Accomplices need to be tried either concurrently or after the principal(s). 7. Effect of principals acquittal: If the principal is not convicted, the accomplices cannot be convicted. 8. No merger rule: one can be convicted of both inchoate conspiracy AND the target offense (differs by jurisdiction).

9. With the exception of criminal homicide, an accomplice cannot be convicted of a more serious offense than the principal(s). 10. Assistance can take the form of physical conduct, psychological influence, or assistance by omission (assuming one has a duty to act). 11. For liability, the principal must actually know of the assistance proffered. Once assistance is established, however, the amount of assistance is irrelevant (no overt act required). Thus, the accomplices assistance need NOT be a cause-in-fact of the crime. 12. Consequential crimes: The common law adheres to the natural-and-probableconsequences doctrine: accomplices are also liable for any crimes committed that, in the ordinary course of things, might reasonably ensue from the planned events. 13. Pinkerton Doctrine: A party to a conspiracy is responsible for any criminal acts committed by an associate if it: a. Falls within the scope of the conspiracy; or b. Is a foreseeable consequence of the unlawful agreement C. MPC 2.06: Accomplices are liable for the targer offense (if completed), or an attempt at the target offense. Defendant guilty of an offense if he commits it by his own conduct or by the conduct of another person for which he is legally accountable, or both. 1. The MPC casts the net of liability to the following groups: a. Direct committer: the actor who directly commits the crime, or the one responsible for the use of an innocent instrumentality to commit the crime. b. Solicitor: an accomplice who solicits the commission of the target crime. c. Aiding: An accomplice who aids the direct committer. d. Agreeing to aid: an accomplice who agrees to aid the direct committer, but doesnt actually assist. e. Attempting to aid: an accomplice who tries to aid the direct committer, but doesnt actually assist. f. Omission: Defendant is an accomplice if there is a legal duty to act ( and does nothing) and has the mental state of promoting or facilitating the commission of the offense. 2. Accomplice liability does not kick in unless a crime has been committed. 3. One can be in more than one category, although one will only face one charge per crime. 4. Accomplices do NOT need to be placed in the correct categories for prosecution. 5. Accomplices can be tried and convicted regardless of whether the direct committer was even charged. 6. Merger rule: one cannot be convicted of both inchoate conspiracy and the target offense. 7. Consequential crimes: The MPC rejects the natural-and-probable consequences doctrine. Rather, each person is analyzed under 2.06 as to whether each person can be places into one of the six categories listed above. 8. The MPC rejects the Pinkerton doctrine. D. WIS. STAT. 939.05: Accomplices are liable for the target offense (if completed), an attempt at the target offense, consequential crimes (if completed), and attempts at consequential crimes. 1. The Wis. Crim. Code casts the net of liability to the following four groups: a. Direct committer: the actor who directly commits the crime, or the one responsible for the use of an innocent instrumentality to commit the crime. b. Aider/abettor: An accomplice who either physically or psychologically assists in the commission of an offense. c. Conspirator: as defined in 939.31. 2. Accomplice liability does not kick in unless a crime had been committed. 3. One can be in more than one category, although one will only face on charge per crime. Furthermore, the jury does not need to agree on which category an accomplice fits into. 4. Accomplices can be tried and convicted regardless of whether the direct committer was even charged. 5. MERGER RULE: 939.72 one cannot be convicted of both inchoate conspiracy and the target offense. Furthermore, one cannot be charged with an attempt and the completed crime. One can be charged, however, with an inchoate offense and an attempt (although not necessarily convicted of both).

6. Consequential crimes: WISCONSIN adheres to the natural-and-probable-consequences doctrine: accomplices are also liable for any crimes committed that, in the ordinary course of things, might reasonably ensue from the planned events. 7. Pinkerton Doctrine: a party to a conspiracy is responsible for any criminal act committed by an associate if it: a. Falls within the scope of the conspiracy; or b. Is a foreseeable consequence of the unlawful agreement E. The Mens Rea of Complicity 1. COMMON LAW: a. An accomplice must possess two intents: i. The intent to assist the primary party to engage in the conduct that forms the basis of the offense; and ii. The mental state required for the commission of the offense, as provided in the definition of the substantive crime. b. Liability extends to crimes of recklessness and negligence as well. The accomplice simply has a higher mens rea than is required. c. For attendant circumstances, the accomplice need only possess the mens rea required by the target offense. 2. MPC: a. An accomplice must posses the purpose of promoting or facilitating the offense. b. Liability extends to crimes of recklessness and negligence as well. The accomplice simply has a higher mens rea than required. 3. Wis. Criminal Code: a. An aider/abettor must act with knowledge or belief that another person is committing or intends to commit a crime, AND must knowingly do one of two things: i. Assists the person who commits the crime; OR ii. Be ready and willing to assist and the direct committer knows of this willingness to assist. COMMON LAW MODEL PENAL CODE WIS. CRIM. CODE 2.06 939.05 Accomplice categories Principal 2nd degree, Solicitor, Aiding, Aider/abettor, solicitor, (actus reus thus varies) accessory before fact (inc. Agreeing to aid, conspirator solicitor), conspirator attempting to aid, Legal duty to prevent Procedural aspects Accomplices cannot be Accomplices can be Accomplices can be convicted unless principal convicted no matter what convicted no matter what 1st deg. Is; must be placed in correct categories Mens Rea Dual: (1) Intent to assist; Purpose (not K) of Knowledge that another is and (2) mental state facilitating the offense committing or intending to required for substantive commit the crime, AND offense knowingly provide either physical or psychological assistance Amount of assistance Actual assistance that is Varies by category Actual assistance that is required effective, even if minute effective even if minute (except conspirators) (except conspirators) Liability (target offense) Attempted/completed Attempted/completed Attempted/completed target offense target offense (if no crime, target offense see 5.01(3)) Liability for consequential Yes Only if an accomplice to it Yes crimes (nat./prob. Conseq.) Liability for reck./neg. Yes, just have higher than Yes, if accomplice to Yes, just have higher than

offenses

required mesn rea (intent)

Abandonment defense

Yes, but only if communicated to others and efforts made to neutralize prior help

Legislative-exemption defense COMMON LAW types of assistance: -physical; -psychological (other must know of intent to assist); and - omission (intent to not act despite a legal duty to do so).

Victims

Requires: 1. accomplice must intentionally assist in the crime; and 2. possess the mental state required for the substantive offense.

conduct that caused the result, and if possessing mental state (if any) of target offense Yes, if complete and voluntary and either neutralizes its effectiveness or gives timely warning to authorities Victims; those whose conduct is necessary to commit the crime Requires: 1. accomplice must assist or attempt to assist in the commission of an offense; and 2. must act with the purpose of promoting or facilitating the offense.

required mens rea (intent)

Yes, but only for solicitors and conspirators if the withdrawal is communicated to the others Victims

Requires: 1. accomplice must provide tangible assistance to another in the commission of an offense.

XVII.

Conspiracy (Inchoate) A. The Nature and Punishment of Conspiracy 1. Inchoate conspiracy defined: An agreement by two or more persons to commit a criminal act or series of criminal acts. 2. At COMMON LAW, a conspiracy to commit a felony or a misdemeanor is a misdemeanor. 3. Under the MPC, a conspiracy to commit a crime is punishable at the level of the target offense, except for first-degree felonies. 4. Under the Wis. Crim. Code., a conspiracy to commit any crime is punishable at the level of the target offense, except when the punishment is life imprisonment (becomes Class B felony). B. The Agreement 1. COMMON LAW: An express agreement is not required, as long as the parties involved are aware of the agreement. A conspiratorial agreement can be proved entirely through circumstantial evidence (it can be inferred from the surrounding circumstances). a. The common law generally does NOT require an overt act in furtherance of the conspiracy (some jurisdictions do). b. The common law also holds liable those who conspire to commit lawful acts by unlawful means such as fraudulent, dishonest, immoral, etc. 2. MPC: Four types of agreement fall within the definition of a conspiracy. Agreements to: (1) commit an offense (2) attempt to commit an offense; (3) solicit another to commit and offense; and (4) aid another in the planning or commission of an offense. a. The MPC requires an overt act to consummate the agreement, except for conspiracies to commit first and second-degree felonies. b. The MPC requires that the object of the conspiratorial agreement be a criminal offense. 3. WIS. CRIMINAL CODE: A conspiracy is formed if one agrees with or combines with another for the purpose of committing a crime. a. The WIS. CRIM. CODE requires an overt act to consummate the agreement, albeit by only one of the parties involoved.

C.

D.

E.

F.

G.

The WIS. CRIM. CODE requires that the object of the conspiratorial agreement be a criminal offense. The Mens Rea of Conspiracy 1. COMMON LAW: Conspiracy is a specific-intent offense. A criminal conspiracy does not occur unless two or more persons: (1) intend to agree; and (2) intend that the object of their agreement be achieved. a. It is unclear whether intent means purpose or knowledge. Look to the actual knowledge one has of the criminal plans. It is also unclear whether intent applies to attendant circumstances. b. Corrupt-motive doctrine: Some jurisdictions require that, above and beyond the normal mens rea requirement, the parties to a conspiracy must also possess a corrupt or wrongful motive for their actions. 2. MPC: Conspiracy is a specific-intent offense. That is, defendant is not guilty of conspiracy unless the agreement was made with the purpose of promoting or facilitating the commission of the substantive offense. a. The MPC requires purpose (knowledge insufficient) as the mens rea. However, the MPC does not state what is required for attendant circumstances. b. The MPC rejects the corrupt-motive doctrine. 3. Under the WIS. CRIM. CODE, conspiracy is a specific-intent offense. That is, defendant is not guilty of conspiracy unless that person enters into a conspiracy with the intent that a crime be committed. a. The Wis. Crim. Code requires intent as the mens rea. Plurality Requirement 1. COMMON LAW: A conspiracy does not exist unless at least two persons possess the requisite mens read (bilateral approach). 2. MPC: A conspiracy exists even if only one person possesses the requisite mens rea (unilateral approach). Proof of an agreement is still needed, however. 3. WIS. CRIM. CODE: A conspiracy exists even if only one person possesses the requisite mens rea (unilateral approach). Objectives of the Conspiracy 1. COMMON LAW: The presumption is against multiple conspiracies. Rather, many courts treat the initial agreement as implicitly incorporating later objectives. 2. MPC: The presumption is against multiple conspiracies. The question to ask is whether there was a single and continuous association for criminal purposes. If yes, then the agreement will be treated as one conspiracy. 3. WIS. CRIM. CODE: The presumption is against multiple conspiracies. The initial agreement will be treated as implicitly incorporating later objectives. Defenses to Conspiracy 1. COMMON LAW: Impossibility is not a defense to inchoate conspiracy. Neither is abandonment, although abandoning a conspiracy before commission of the target offense will relieve that person of liability beyond inchoate conspiracy. CLASS NOTES ON CONSPIRACY: 1. 5.03 MPC in Choate offenses p 1194 2. Renunciation of conspiracy (6) renunciation of criminal purpose it is an affirmative defense that the actor, after conspiring to commit a crime, thwarted the success of the conspiracy, under circumstances manifesting a complete and voluntary renunciation of his criminal purpose 3. Subjective rather than objective 4. Can also renounce attempt 5. 939.72(2) Wis. Stat 6. 107 (1)(b) MPC bars conviction 7. 5.05(3) 8. MPC cannot be convicted of two in choate offenses, like conspiracy and attempt know Wis. And MPC for conspiracy, timeline from thought to completed crime 9. act of agreement is seen as moving beyond thought and provides evidence of actors criminal intent

b.

10. agreement is actus reus for conspiracy 11. in addition to agreement most jurisdictions, WI, require an overt act in addition to the agreement MPC disagrees on one point, agreement is all that is needed for murder, first-degree felony, overt act is required of only one of the conspirators, and it binds the other two 12. Bilateral Conspiracy a. Common law agreement requires agreement by both parties, A agrees w/ B, B agrees w/ A. to be liable each conspirator must agree, this is bilateral. b. If three parties, and one is officer, can still be conspiracy if X and Y agree together and Z, the cop, doesnt agree, could still be viewed as an agreement amongst all parties, commonality between X and Y here 13. Unilateral Agreement conspiracy WI and MPC a. Permits liability if actor agrees with another person b. No meeting of minds necessary c. If other conspirator is under cover cop, and does not intend to agree, it doesnt matter 14. Unconvictable co-conspirator a. If B is insane cannot be a co-conspirator b. Common law defense, insanity defense to bilateral agreement c. MPC rejects all forms of this 5.04(1) d. 939.31 WI conspiracy law 15. Consistence law a. X, y, and z are all a part of a conspiracy. X is acquitted, y is acquitted, therefore z cannot be found guilty because others were acquitted. b. Under MPC this does not work because it is a subjective standard and Z is still guilty c. Wis. Law suggested that courts may hold what MPC does but it has not yet been decided.because unilateral has been adopted d. Completed crimes 939.05(2)(c) party to a crime conspiracy MENS REA Conduct circumstance result Purpose under CL must be purpose, under MPC and WI, as to conduct, circumstance, result, under MPC, there must be purpose as to result, high culpability requirement, MPC circumstance is not clear, at least a knowledge standard as to the circumstance, WI intent as to circumstance and result, and times seems like purpose Knowing Reckless Negligence Strict Liability Gap between conspiracy and complicity will be on test Pinkerton doctrine Pinkerton doctrine a. A party to a conspiracy is responsible for any criminal act committed by an associate if it: i. (1) falls within the scope of the conspiracy; or ii. (2) Is a foreseeable consequence of the unlawful agreement 4. ex. Of burglary in conspiracy to kill Kissinger 940.01 5. MPC doesnt adopt the pinkerton doctrine 6. MPC requires proof 7. Pinkerton and WI JI 411 on exam pinkerton in furtherance of conspiracy 8. 2.06 MPC liability of conduct of another if at time of offense has conspiracy begun, etc 9. scope what if conspiracy has two different objectives? Ex. Kidnap Kissinger and rob helmsley. Two objectives, one conspiracy is whole group a conspiracy, or is it a group of smaller conspiracy. 10. Drug lord, wholesaler,down the chain, many people dont know each otherchain conspiracybut all one conspiracy 11. P 952, 953different types of conspiracy 1. 2. 3.

12. Duration ends when object of conspiracy complete 13. To withdraw, must notify co-conspirators, must do acts that contradict/inconsistent with conspiracy. COMMON LAW Punishable conspiracies Punishment level Actus reus (the agreement) Crimes, lawful acts by unlawful means Misdemeanor Express not required as long as the parties are aware of the agreement NO OVERT ACT MODEL PENAL CODE 5.03 Crimes Target offense (but 1 deg. = 2nd deg) Agreement to attempt/commit crime, aid in commission or solicitation PLUS OVERT ACT (except 1, 2 deg. Felonies) Purpose of promoting or facilitating commission of offense Merges with attempted/completed crime Only if an accomplice to it WIS. CRIM. CODE 939.31 Crimes Target offense, except life = Class B Agree or combine with another for the purpose of committing a crime PLUS OVERT ACT

Mens Rea

Relationship to target offense (merger rule) Liability for consequential crimes (nat./prob. Conseq.) Plurality Abandonment defense

Dual (1) intend to agree; (2) intend that the object be achieved (some say corrupt motive) Does not merge with attempted/completed crime YES (Pinkerton)

Intent that the crime be committed

Merges with attempted/completed crime Yes (pinkerton)

Whartons rule defense

Legislative-exemption defense

Must be at least two conspirators No defense to inchoate, but can preclude attempted/completed crime liability if communicated to others Cant prosecute as conspirators if those two are required for the crime No conspiracy conviction if doing so frustrates legislative purpose to exempt that person from prosecution Requires: (1) an intended agreement to commit a crime, or to perform a lawful act by unlawful means; and (2) Intent that the object of the agreement be achieved

Only need one conspirator Defense if complete and voluntary renunciation and conspiracy thwarted

Only need one conspirator No defense to inchoate, but can preclude attempted/completed crime liability to others Not recognized (unilateral)

Not recognized (unilateral)

Victims, those whose conduct is necessary to commit the crime

Unknown

Requires: (1) an agreement to commit a crime (2) the purpose of promoting or facilitating commission of the offense; and (3) an overt act in furtherance of that agreement (except for 1, 2 deg. Fel.)

Requires: (1) an agreement to commit a crime; (2) the intent that the crime be committed; and (3) an overt act in furtherance of that agreement.

Defensive Force

8.01 Use of Non-deadly force [A] Common Law A non-aggressor is justified in using force upon another if he reasonably believes that such force is necessary to protect himself from imminent use of unlawful force by the other person. However, the use of

force must not be excessive in relation to the harm threatened. One is never permitted to use deadly force to repel a non-deadly attack. [B] Model Penal Code A person is justified in using force upon another person if he believes that such force is immediately necessary to protect himself against the exercise of unlawful force by the other on the present occasion. [MPC 3.04(1)] In a departure from common law principles but in accord with the modern trend, a person may not use force to resist an arrest that he knows is being made by a police officer, even if the arrest is unlawful (e.g., without probable cause). [MPC 3.04(2)(a)(i)] However, this rule does not prohibit use of force by an arrestee who believes that the officer intends to use excessive force in effectuating the arrest. The provision does not specifically require the defendants belief to be reasonable. However, nearly all of the Code justification defenses, including the defense of self-protection, are modified by 3.09, which re-incorporates a reasonableness component. 8.02 Use of Deadly force [A] Common Law Deadly force is only justified in self-protection if the defendant reasonably believes that its use is necessary to prevent imminent and unlawful use of deadly force by the aggressor. Deadly force may not be used to combat an imminent deadly assault if a non-deadly response will apparently suffice. [B] Model Penal Code The Code specifically sets forth the situations in which deadly force is justifiable: when the defendant believes that such force is immediately necessary to protect himself on the present occasion against: (1) death; (2) serious bodily injury; (3) forcible rape; or (4) kidnapping. The Code prohibits the use of deadly force by a deadly aggressor, i.e., one who, with the purpose of causing death or serious bodily injury, provoked the use of force against himself in the same encounter. [MPC 3.04(2)(b)(i)] 8.03 Retreat Rule [A] Common Law If a person can safely retreat and, therefore, avoid killing the aggressor, deadly force is unnecessary. Nonetheless, jurisdictions are sharply split on the issue of retreat. A slim majority of jurisdictions permit a non-aggressor to use deadly force to repel an unlawful deadly attack, even if he is aware of a place to which he can retreat in complete safety. Many jurisdictions, however, provide that a non-aggressor who is threatened by deadly force must retreat rather than use deadly force, if he is aware that he can do so in complete safety. A universally recognized exception to the rule of retreat is that a non-aggressor need not ordinarily retreat if he is attacked in his own dwelling place or within its curtilage [the immediately surrounding land associated with the dwelling], even though he could do so in complete safety. [B] Model Penal Code One may not use deadly force against an aggressor if he knows that he can avoid doing so with complete safety by retreating. Retreat is not generally required in ones home or place of work. However, retreat from the home or office is required: (1) if the defendant was the initial aggressor, and wishes to regain his right of self-protection; or (2) even if he was not the aggressor, if he is attacked by a co-worker in their place of work. However, the Code does not require retreat by a non-aggressor in the home, even if the assailant is a codweller. 8.04 Reasonable Belief The privilege of self-defense is based on reasonable appearances, rather than on objective reality. Thus, a person is justified in using force to protect himself if he subjectively believes that such force is necessary to repel an imminent unlawful attack, even if appearances prove to be false.

Courts are increasingly applying a standard of the reasonable person in the defendants situation in lieu of the reasonable person standard. Factors that may be relevant to the defendants situation or circumstances include: (1) the physical movements of the potential assailant; (2) any relevant knowledge the defendant has about that person; (3) the physical attributes of all persons involved, including the defendant; (4) any prior experiences which could provide a reasonable basis for the belief that the use of deadly force was necessary under the circumstances. 8.05 Imperfect Self-Defense Claims [A] Common Law The traditional common law rule is that if any element necessary to prove self-defense is lacking, the defense is wholly unavailable to a defendant. Some states now recognize a so-called imperfect or incomplete defense of self-defense to murder, which results in conviction for the lesser offense of either voluntary or involuntary manslaughter. For example, a defendant who fails to satisfy the reasonableness component, although his belief was genuine, might be able to assert an imperfect or incomplete claim of self-defense, mitigating his crime to manslaughter. [B] Model Penal Code The Model Penal Code likewise recognizes an imperfect defense where the defendant asserts a justification defense, evaluated in terms of the defendants subjective belief in the necessity of using the force or other material circumstances. However, justification defenses are subject to section 3.09(2), which provides that when the defendant is reckless or negligent in regard to the facts relating to the justifiability of his conduct, the justification defense is unavailable to him in a prosecution for an offense for which recklessness or negligence suffices to establish culpability. 8.06 Battered Woman Syndrome A special type of self-defense is the battered woman syndrome defense. Cases in which this defense arise may occur under three scenarios: (1)Confrontational homicides, i.e., cases in which the battered woman kills her partner during a battering incident. In such cases, an instruction on self-defense is almost always given. It is now routine for a court to permit a battered woman to introduce evidence of the decedents prior abusive treatment of her, in support of her claim of self-defense. (2) Non-confrontational homicide, where the battered woman kills her abuser while he is asleep or during a significant lull in the violence. Courts are divided on whether self-defense may be claimed if there is no evidence of threatening conduct by the abuser at the time of the homicide, although the majority position is that homicide under such circumstances is unjustified. (3) Third-party hired-killer cases, in which the battered woman hires or importunes another to kill her husband, and then pleads self-defense. Courts have unanimously refused to permit instructions in third-party hired-killer cases. 8.07 Risk to Innocent Bystanders [A] Common Law Courts apply a transferred-justification doctrine, similar to the transferred-intent rule: a defendants right of self-defense transfers (just as intent to kill does) from the intended to the actual victim. While the defense is absolute in some jurisdictions, other courts do not treat this rule as absolute. If the defendant, acting justifiably in self-defense against an aggressor, fires a weapon wildly or carelessly, thereby jeopardizing the safety of known bystanders, some courts hold the defendant guilty of manslaughter (or of reckless endangerment if no bystander is killed), but not of intentional homicide. [B] Model Penal Code If a person justifiably uses force against an aggressor, but uses such force in a reckless or negligent manner in regard to the safety of an innocent bystander, the justification defense, which is available to the person in regard to the aggressor, is unavailable to him in a prosecution for such recklessness or negligence as to the bystander.

Chapter 9 DEFENSE OF OTHERS 9.01 Common Law Rule Generally speaking, a person is justified in using force to protect a third party from unlawful use of force by an aggressor to the extent that the third party is justified in acting in self-defense. This so-called alter ego rule, as applied in early common law, required that the third party had to in fact have been justified in self-defense, irrespective of how the situation would have appeared to a reasonable person. Today, however, the majority view is that the use force may be justified if it reasonably appears necessary for the protection of the third party. 9.02 Model Penal Code [A] General Rule Subject to retreat provisions, Section 3.05(1) justified force by an intervenor in order to protect a third party if: (1) he uses no more force to protect the third-party than he would be entitled to use in self-protection, based on the circumstances as he believes them to be; (2) under the circumstances as he believes them to be, the third party would be justified in using such force in selfdefense; and (3) he believes that intervention is necessary for the third partys protection. [B] Effect of Retreat Rules The Codes retreat rules have applicability in very limited circumstance here: (1) the intervenor is only required to retreat before using force in protection of a third party in the unlikely circumstance that he knows that such retreat will assure the third partys complete safety. [MPC 3.05(2)(a)] (2) the intervenor must attempt to secure the third partys retreat if the third party herself would be required to retreat, if the defendant knows that the third party can reach complete safety by retreating. [MPC 3.05(2)(b)] (3) neither the intervenor nor the third party is required to retreat in the others dwelling or place of work to any greater extent than in her own dwelling or place of work. [MPC 3.05(2)(c)]

Chapter 10 DEFENSE OF PROPERTY AND HABITATION 10.01 Defense of Property [A] Common Law A person in possession of real or personal property is justified in using non-deadly force against a would-be dispossessor if he reasonably believes that such force is necessary to prevent imminent and unlawful dispossession of the property. Under no circumstances may a person use deadly force to prevent dispossession. [1] Possession versus Title to Property The privilege of defense-of-property entitles a person to use necessary force to retain rightful possession of, as distinguished from title to, personal or real property. [2] Threat to Use Deadly Force Although states universally prohibit use of deadly force to protect property, they are divided as to whether one may threaten it as a way to prevent dispossession. [3] Claim of Right When a person asserts a claim of right to property in the possession of another and seeks to reclaim such property, the possessor is not justified in using force to thwart the dispossession if he knows, believes, or as a reasonable person should believe, that the claimant has a legitimate claim of right to possession of the property in question. Since the use of force to protect property is legitimate only if the act/attempted act of dispossession is unlawful, in such cases of a legitimate claim to property, the act of dispossession is lawful. [4] Recapture of Property A person may not ordinarily use force to recapture property of which he has been unlawfully dispossessed except if he acts promptly after dispossession. One may follow the dispossessor in hot pursuit in order to recapture his property and if necessary, use non-deadly force in the process. [B] Model Penal Code [1] General Rule Allowing Use of Non-deadly Force The Model Penal Code essentially conforms to the common law. Section 3.06(1)(a) provides that a person may use non-deadly force upon another person to prevent or terminate an entry or other trespass upon land, or to prevent the carrying away of personal property, if he believes that three conditions exist: (1) the other persons interference with the property is unlawful; (2) the intrusion affects property in the defendants possession, or in the possession of someone else for whom he acts; and (3) non-deadly force is immediately necessary. [2] Limitations on Use of Non-deadly Force Non-deadly force that is otherwise permitted in defense of property is unjustified in two circumstances. (1) Force is not immediately necessary unless the defender first requests desistance by the interfering party. A request is not required, however, if the defender believes that a request would be useless, dangerous to himself or to another, or would result in substantial harm to the property before the request can effectively be made. [MPC 3.06(3)(a)] (2) One may not use force to prevent or terminate a trespass to personal or real property if he knows that to do so would expose the trespasser to a substantial risk of serious bodily injury. [MPC 3.06(3)(b)] [3] Recapture of Property Section 3.06(1)(b) permits the use of non-deadly force to re-enter land or to recapture personal property if: (1) the defendant believes that he or the person for whom he is acting was unlawfully dispossessed of the property; and either (2a) the force is used immediately after dispossession; or (2b) even if it is not immediate, the defendant believes that the other person has no claim of right to possession of the property. Here, however, re-entry of land (as distinguished from recapture of personal property) is not permitted

unless the defendant also believes that it would constitute an exceptional hardship to delay re-entry until he can obtain a court order. [4] Deadly Force to Prevent Serious Property Crimes The Model Code goes beyond the common law in permitting deadly force to protect any type of property in limited circumstances, where the defendant believes that: (1) the other person is attempting to commit arson, burglary, robbery, or felonious theft or property destruction; (2) such force is immediately necessary to prevent commission of the offense; and either (3a) the other person previously used or threatened to use deadly force against him or another person in his presence, or (3b) use of non-deadly force to prevent commission of the offense would expose him or another innocent person to substantial danger of serious bodily injury. [MPC 3.06(3)(d)(ii)] 10.02 Defense of Habitation [A] Common Law Use of Deadly Force It is generally accepted that a person may use deadly force to defend his home, but the extent to which such exercise of deadly force is justified varies. Some courts allow deadly force only to prevent entry into the home. In such jurisdictions, once entry has occurred, the defendant is only justified in using deadly force if based on another ground such as self-defense. Others permit deadly force in the home even after entry has been completed. There are three approaches to the use of deadly force in defense of habitation. [1] Early Common Law Rule Early common law broadly defined this rule to permit a home-dweller to use deadly force if he reasonably believed that such force was necessary to prevent an imminent and unlawful entry of his dwelling. [2] Middle Approach A more narrow approach to the defense of habitation provides that a person may use deadly force if he reasonably believes that: (1) the other person intends an unlawful and imminent entry of the dwelling; (2) the intruder intends to injure him or another occupant, or to commit a felony therein; and (3) deadly force is necessary to repel the intrusion. [3] Narrow Approach A narrow version of the defense provides that a person is justified in using deadly force upon another if he reasonably believes that: (1) the other person intends an unlawful and imminent entry of the dwelling; (2) the intruder intends to commit a forcible felony a felony committed by forcible means, violence, and surprise, such as murder, robbery, burglary, rape, or arson or to kill or seriously injure an occupant; and (3) such force is necessary to prevent the intrusion. [B] Model Penal Code A person may use deadly force upon an intruder if he believes that: (1) the intruder is seeking to dispossess him of the dwelling; (2) the intruder has no claim of right to possession of the dwelling; and (3) such force is immediately necessary to prevent dispossession. [MPC 3.06(3)(d)(i)] The defendant may use deadly force even if he does not believe that his or another persons physical well-being is jeopardized. This provision is broader than the common law in that the right to use deadly force is not predicated on the defendants right to safe and private habitation, but rather is founded on his right to possession of the dwelling. On the other hand, this provision does not authorize deadly force merely to prevent an unlawful entry into the home, as the common law originally permitted. 10.03 Spring Guns

An increasing number of states now prohibit the use of a mechanical device designed to kill or seriously injure an intruder to protect property, even if the possessor would be justified in using deadly force in person. The Model Penal Code bans the use of such devices as well. [MPC 3.06(5)(a)] Chapter 14 INTOXICATION 14.01 Voluntary Intoxication Voluntary intoxication does not excuse criminal conduct; however, in limited circumstances, intoxication may negate the necessary state of mind for a given offense and thus prove exculpatory. Intoxication resulting from alcoholism or drug addiction is considered voluntary under common law principles. [A] Mens Rea Defense While there are several approaches to evaluating the mens rea portion of criminal activity involving an intoxicated defendant, the most common approach distinguishes between general-intent and specific-intent crimes. Under this common law approach, voluntary intoxication is not a defense to generalintent crimes. Voluntary intoxication is a defense to specific intent crimes. [B] Special Rule for Homicide Two states, Virginia and Pennsylvania, limit the defense of voluntary intoxication to first-degree murder prosecutions. In states that recognize the defense in all specific-intent crimes, if the crime of murder is divided into degrees, a defendant may generally introduce evidence that his intoxication prevented him from being able to form the requisite state of mind for first-degree murder. [C] Voluntary Act Where a defendants intoxication was so severe as to render him unconscious at the time of the commission of the crime, some courts have barred a defense based on unconsciousness if such condition resulted from the voluntary consumption of alcohol or drugs. Others courts allow the defendant to argue that the criminal act was not a voluntary one due to his unconscious state, but only in defense to specificintent offenses. [D] Intoxicated-Induced Insanity The common law does not recognize a defense of temporary insanity based on intoxication where the defendants intoxication was voluntary. Some jurisdictions do recognize a defense based on fixed insanity, a condition which results from long-term use of drugs or alcohol 14.02 Involuntary Intoxication [A] Definition Intoxication is involuntary if the defendant is not to blame for becoming intoxicated. It may result from: coerced intoxication; intoxication by innocent mistake as to the nature of the substance being consumed; unexpected intoxication from a prescribed medication provided the defendant did not purposely take more than the prescribed dosage; or pathological intoxication, a temporary psychotic reaction, often manifested by violence, which is triggered by consumption of alcohol by a person with a pre-disposing mental or physical condition, e.g., temporal lobe epilepsy, encephalitis, or a metabolic disturbance. The defense only applies if the defendant had no reason to know that he was susceptible to such a reaction. [B] Availability of the Defense Under common law, a defendant found to have been involuntarily intoxicated may avail himself of the defense of temporary insanity. Furthermore, one who committed an offense while involuntarily intoxicated can otherwise seek acquittal by asserting the mens rea defense. 14.03 Model Penal Code [A] General Rule Model Penal Code 2.08(4)(5) distinguishes three types of intoxication: (1) voluntary (self-induced) intoxication; (2) pathological intoxication; and (3) involuntary (non-self-induced) intoxication.

[B] Exculpation Based on Intoxication [1] Mens Rea Defense Any form of intoxication is a defense to criminal conduct if it negates an element of the offense. [MPC 2.08(1)] Since the Code does not distinguish between general intent and specific intent offenses, the mens rea defense is broadly applied, with one exception. In the case of crimes defined in terms of recklessness, a person acts recklessly as to an element of the crime if, as the result of the selfinduced intoxication, he was not conscious of a risk of which he would have been aware had he not been intoxicated. [MPC 2.08(2)] [2] Insanity Pathological and involuntary intoxication are affirmatives defenses, if the intoxication causes the defendant to suffer from a mental condition comparable to that which constitutes insanity under the Code. [MPC 2.08(4)] Chapter 15 INSANITY 15.01 Legal Tests to Determine Insanity Insanity is a legal term that presupposes a medical illness or defect but is not synonymous with mental illness, mental disorder, and mental disease or defect. Mental illness is a more encompassing term than insanity, and thus, a person can be mentally ill medically speaking without legally being insane. Five tests of insanity have been applied at one time or another. [A] MNaghten Test The MNaghten rule focuses exclusively on cognitive disability. Under this rule, a person is insane if, at the time of the criminal act, he was laboring under such a defect of reason, arising from a disease of the mind, that he (1) did not know the nature and quality of the act that he was doing; or (2) if he did know it, he did not know that what he was doing was wrong. This test requires total cognitive disability and does not allow for degrees of incapacity and nor does it recognize volitional incapacity in which a person is aware that conduct is wrong yet cannot control his behavior. [B] Irresistible Impulse Test Some jurisdictions have broadened the scope of MNaghten to include mental illnesses that affect volitional capacity. Generally speaking, a person is insane if, at the time of the offense: (1) he acted from an irresistible and uncontrollable impulse; (2) he was unable to choose between the right and wrong behavior; (3) his will was destroyed such that his actions were beyond his control. [C] Model Penal Code Test The Model Penal Code provides that a person is not responsible for his criminal conduct if, at the time of the conduct, as the result of a mental disease or defect, he lacked substantial capacity to: (1) appreciate the criminality (or wrongfulness) of his conduct; or (2) to conform his conduct to the requirements of the law. This test does not require total mental incapacity. [D] The Product (Durham) Test This rule, now defunct, provided that a defendants criminal behavior may be excused if he was suffering from a mental disease or defect at the time of the offense and the criminal conduct was the product of the mental disease or defect. [E] Federal Test In 1984, Congress enacted a statutory definition of insanity applicable to federal criminal trials. [18 U.S.C. 17(a) (2000)] The federal law provides that a defendant may be excused based on insanity if he proves by clear and convincing evidence that, at the time of the offense, as the result of a severe mental disease or defect, he was unable to appreciate: (1) the nature and quality of his conduct; or (2) the wrongfulness of his conduct. This test requires complete cognitive incapacity. Chapter 20 ATTEMPT

20.01 Attempt, Generally A criminal attempt occurs when a person, with the intent to commit an offense, performs any act that constitutes a substantial step toward the commission of that offense. Criminal attempts are of two varieties: complete (but imperfect); and incomplete. A complete, but imperfect, attempt occurs when the defendant performs all of the acts that he set out to do, but fails to attain his criminal goal. In contrast, an incomplete attempt occurs when the defendant does some of the acts necessary to achieve the criminal goal, but he quits or is prevented from continuing, e.g., a police officer arrives before completion of the attempt. 20.02 Mens Rea of Criminal Attempts [A] General Rule The defendant must intentionally commit the acts that constitute the actus reus of an attempt, i.e., acts that bring him in proximity to commission of a substantive offense or which otherwise constitute a substantial step in that direction, and he must perform these acts with the specific intention of committing the target crime. An attempt is a specific-intent offense, even if the substantive crime is a generalintent offense. [B] Result Crimes When the target crime is a result crime, the general rule is that a person is not guilty of an attempt unless his actions in furtherance of the target crime are committed with the specific purpose of causing the unlawful result. 20.03 Actus Reus of Criminal Attempts Courts have developed a number of tests to determine the point at which a defendant passes beyond the preparation stage and consummates the criminal attempt. (1) Last act test an attempt occurs at least by the time of the last act but this test does not necessarily require that each and every act be performed on every occasion. (2) Physical proximity test the defendants conduct need not reach the last act but must be proximate to the completed crime. (3) Dangerous proximity test an attempt occurs when the defendants conduct is in dangerous proximity to success, or when an act is so near to the result that the danger of success is very great. (4) Indispensable element test an attempt occurs when the defendant has obtained control of an indispensable feature of the criminal plan. (5) Probable desistance test an attempt occurs when the defendant has reached a point where it was unlikely that he would have voluntarily desisted from his effort to commit the crime. (6) Unequivocality (or res ipsa loquitur) test an attempt occurs when a persons conduct, standing alone, unambiguously manifests his criminal intent. 20.04 Defense of Impossibility [A] General Rule At common law, legal impossibility is a defense; factual impossibility is not. However, today, most jurisdictions no longer recognize legal impossibility as a defense. [B] Factual Impossibility Factual impossibility exists when a persons intended result constitutes a crime, but he fails to consummate the offense because of an attendant circumstance unknown to him or beyond his control. Examples of factual impossibility are a pickpocket putting his hand in the victims empty pocket; shooting into an empty bed where the intended victim customarily sleeps; or pulling the trigger of an unloaded gun aimed at a person. [C] Inherent Factual Impossibility Although largely academic, the doctrine of inherent factual impossibility has been recognized as a statutory defense in at least one state (Minnesota). Where recognized, the defense applies if the method to accomplish the crime was one that a reasonable person would view as inadequate to accomplish the criminal objective.

[D] Pure Legal Impossibility Pure legal impossibility arises when the law does not proscribe the result that the defendant seeks to achieve. [E] Hybrid Legal Impossibility Hybrid legal impossibility (or legal impossibility) exists if the defendants goal is illegal, but commission of the offense is impossible due to a factual mistake (and not simply a misunderstanding of the law) regarding the legal status of an attendant circumstance that constitutes an element of the charged offense, e.g., receiving unstolen property under the belief that such property was stolen, or shooting a corpse believing it is alive. Today, most states have abolished the defense of hybrid legal impossibility on the theory that a defendants dangerousness is plainly manifested in such cases. 20.05 Defense: Abandonment Many courts do not recognize the defense of abandonment. Where recognized, it applies only if the defendant voluntarily and completely renounces his criminal purpose. Abandonment is not voluntary if the defendant is motivated by unexpected resistance, the absence of an instrumentality essential to the completion of the crime, or some other circumstance that increases the likelihood of arrest or unsuccessful consummation of the offense, or if the defendant merely postpones the criminal endeavor until a better opportunity presents itself. 20.06 Model Penal Code [A] Elements of the Offense Generally speaking, a criminal attempt under the Code contains two elements: (1) the purpose to commit the target offense; and (2) conduct constituting a substantial step toward the commission of the target offense. [B] Mens Rea In general, a person is not guilty of a criminal attempt unless it was his purpose, i.e., his conscious object, to engage in the conduct or to cause the result that would constitute the substantive offense. A person is likewise guilty of an attempt to cause a criminal result if he believes that the result will occur, even if it were not his conscious object to cause it. [MPC 5.01(1)(b)] The mens rea of purpose or belief does not necessarily encompass the attendant circumstances of the crime. For these elements, it is sufficient that the defendant possesses the degree of culpability required to commit the substantive offense. [C] Actus Reus The Code shifts the focus of attempt law from what remains to be done, i.e., the defendants proximity to consummation of the offense, to what the defendant has already done. Subsection 5.01(1)(c) provides that, to be guilty of an offense, a defendant must have done or omitted to do something that constitutes a substantial step in a course of conduct planned to culminate in his commission of the crime. Section 5.01(2) provides a list of recurrent factual circumstances in which a defendants conduct, if strongly corroborative of his criminal purpose, shall not be held insufficient as a matter of law, including lying in wait; searching for or following the contemplated victim of the crime; reconnoitering the contemplated scene of the crime; unlawful entry into a structure or building in which the crime will be committed; and possession of the materials to commit the offense, if they are specially designed for a criminal purpose. [D] Attempt to Aid Under 5.01(3), a person may be convicted of a criminal attempt, although a crime was neither committed nor attempted by another, if: (1) the purpose of his conduct is to aid another in the commission of the offense; and (2) such assistance would have made him an accomplice in the commission of the crime under the Codes complicity statute if the offense had been committed or attempted. [E] Hybrid Legal Impossibility There is no defense of hybrid legal impossibility under MPC 5.01(1). [F] Pure Legal Impossibility The Code does not expressly address the defense of pure legal impossibility. [G] Renunciation (Abandonment) Under the Code, a person is not guilty of an attempt if: (1) he abandons his effort to commit the crime or prevents it from being committed; and (2) his conduct manifests a complete and

voluntary renunciation of his criminal purpose. [MPC 5.01(4)] Under this provision, renunciation is not complete if it is wholly or partially motivated by a decision to postpone the criminal conduct until a more advantageous time or to transfer the criminal effort to another but similar objective or victim or if motivated by circumstances . . . that increase the probability of detection or apprehension or that make more difficult the accomplishment of the criminal purpose. Chapter 22 CONSPIRACY 22.01 General Principle Generally speaking, a conspiracy is an agreement by two or more persons to commit a criminal act or series of criminal acts, or to accomplish a legal act by unlawful means. [A] The Agreement [1] Common law At common law, a conspiracy need not be based on an express agreement. Furthermore, an agreement can exist although not all of the parties to it have knowledge of every detail of the arrangement, as long as each party is aware of its essential nature. [Blumenthal v. United States, 332 U.S. 539, 55758 (1947)] Moreover, a conspiracy may exist even if a conspirator does not agree to commit or facilitate each and every part of the substantive offense.[Salinas v. United States, 522 U.S. 52, 63 (1997)] It is enough that each person agrees, at a minimum, to commit or facilitate some of the acts leading to the substantive crime. [2] Model Penal Code Four types of agreement fall within the definition of conspiracy. A person is guilty of conspiracy if he agrees to: (1) commit an offense; (2) attempt to commit an offense; (3) solicit another to commit an offense; or (4) aid another person in the planning or commission of the offense. [B] Overt Act [1] Common and Statutory Law A common law conspiracy is complete upon formation of the unlawful agreement. No act in furtherance of the conspiracy need be proved. [United States v. Shabani, 513 U.S. 10, 13 (1994)] Today, many statutes require proof of the commission of an overt act in furtherance of the conspiracy. In jurisdictions requiring an overt act, the act need not constitute an attempt to commit the target offense. Instead, any act (and perhaps an omission), no matter how trivial, is sufficient, if performed in pursuance of the conspiracy. A single overt act by any party to a conspiracy is sufficient basis to prosecute every member of the conspiracy, including those who may have joined in the agreement after the act was committed. Most states apply the overt-act rule to all crimes. [2] Model Penal Code The Codes requirement of proof of an overt act only applies to cases involving a misdemeanor or a felony of the third degree. [MPC 5.03(5)] [C] Plurality Requirement [1] Common Law Common law conspiracy requires proof that at least two persons possessed the requisite mens rea of a conspiracy. For example, no conspiracy conviction is possible if one of the two persons is an undercover agent feigning agreement, or lacks the capacity to form the agreement due to mental illness. [2] Model Penal Code; Majority Rule The Model Code departs significantly from the common law by establishing a unilateral approach to conspiracy liability. The Code focuses on the culpability of the defendant whose liability is in issue, rather than on that of the larger conspiratorial group. Specifically, the Code provides,

A person is guilty of conspiracy with another person if he agrees with such other person to commit an offense. The unilateral approach has been adopted in most states. 22.02 Mens Rea [A] In General [1] Common law Common law conspiracy is a specific-intent offense, requiring that two or more persons: (1) intend to agree; and (2) intend that the object of their agreement be achieved. Absence of either intent renders the defendants conduct non-conspiratorial. However, courts are divided over the interpretation of intent. Some require that the parties have the unlawful result as their purpose and others allow conviction for conspiracy based on the parties mere knowledge that such result would occur from their conduct. [2] Model Penal Code The Code specifically provides that the conspiratorial agreement must be made with the purpose of promoting or facilitating the commission of the substantive offense. Thus, in jurisdictions following the Code, a conspiracy does not exist if one is aware of, but fails to share, another persons criminal purpose. [B] Corrupt-Motive Doctrine Some common law jurisdictions apply what has come to be known as the corrupt motive doctrine. This doctrine states that in addition to the usual mens rea requirements of conspiracy (i.e., intent to agree, and intent to commit the substantive offense), the parties to a conspiracy must also have a corrupt or wrongful motive for their actions. The Model Penal Code does not recognize the corrupt-motive doctrine. 22.03 Parties to a Conspiracy [A] Liability of Parties for Substantive Offenses Each party to a conspiracy is liable for every offense committed by every other conspirator in furtherance of the unlawful agreement. Thus, an important issue in conspiracy trials may be to determine the precise confines of a conspiratorial enterprise. [B] Overt-Act Requirement The structure of a conspiracy is critical in jurisdictions recognizing an overtact requirement. In these jurisdictions, an act of one conspirator in furtherance of the agreement renders a prosecution permissible against every other party to the same agreement. [C] Common Law Analysis [1] In General To be regarded as a co-conspirator, a person does not need to know the identity, or even of the existence, of every other member of the conspiracy, nor must he participate in every detail or event of the conspiracy. However, to be a co-conspirator he must have a general awareness of the scope and the objective of the criminal enterprise. [2] Wheel Conspiracies A wheel conspiracy is characterized by a central figure or group (the hub) that engages in illegal dealings with other parties (the spokes) and there exists a shared criminal purpose among all spokes and the hub. Parallel but separate objectives between similarly situated people do not make a wheel conspiracy (instead this would constitute multiple chain conspiracies). In Kotteakos v. United States, 328 U.S. 750 (1946), a broker obtained fraudulent loans from the government for thirty-one people. All were tried under a theory of wheel conspiracy. However, evidence at trial demonstrated that the loan recipients were part of eight or more independent groups, none of which had any connection with any other group except that each used the same broker. Absent a single shared objective, the parties constituted eight or more chain conspiracies and not a single wheel conspiracy. [3] Chain Conspiracies Chain conspiracies ordinarily involve a criminal enterprise that cannot thrive unless each link successfully performs its part in the arrangement. In Blumenthal v. United States, 332 U.S. 539 (1947), the owner of a liquor wholesale agency distributed whiskey through two men, Weiss and

Goldsmith, who arranged with Feigenbaum and Blumenthal to sell the whiskey to local tavern owners at a price in violation of the law. The Supreme Court held that the prosecutors charge of a single conspiracy was proper, finding that each salesman by reason of [his] knowledge of the plans general scope, if not its exact limits, sought a common end, to aid in disposing of the whiskey. An opposite conclusion was reached in United States v. Peoni, 100 F.2d 401 (2d Cir. 1938). Peoni sold a small quantity of counterfeit money to Regno, who in turn sold the money to Dorsey, who passed the money in commerce to innocent persons. No common interest was found between Peoni and Dorsey, and thus the court concluded that there existed two independent conspiracies, one between Peoni and Regno and another between Regno and Dorsey. [D] Model Penal Code In addition to 5.03(1), which sets forth a unilateral approach to conspiracy, 5.03(2) provides that if a person knows that a person with whom he conspires to commit a crime has conspired with another person or persons to commit the same crime, he is guilty of conspiring with such other person or persons, whether or not he knows their identity, to commit such crime. The Model Penal Code provides that a person with multiple criminal objectives is guilty of only one conspiracy if the multiple objectives are: (1) part of the same agreement; or (2) part of a continuous conspiratorial relationship. [MPC 5.03(3)] 22.04 Relationship to Target Offense [A] General rule [1] Common Law and Non-Model Penal Code Statutes At common law, a conspiracy to commit a felony or a misdemeanor was a misdemeanor. Under modern statutory law, the seriousness of the crime of conspiracy varies. Some states continue to treat all conspiracies, regardless of the seriousness of their objectives, as misdemeanors. More often, however, the sanction for conspiracy corresponds to the contemplated crime so that conspiracy to commit a felony is graded as a felony, and conspiracy to commit a misdemeanor is a misdemeanor. In most states, a conspiracy to commit a felony is punished less severely than the target offense. [2] Model Penal Code As with other inchoate offenses, the Model Penal Code sanctions a conspiracy to commit any crime other than a felony of the first degree at the same level as the target offense. [MPC 5.05(1)] If a conspiracy has multiple objectives, e.g., to rape and to steal, the conspiracy is graded on the basis of the most serious target offense. [B] Punishment When the Target Offense is Committed [1] Common Law Unlike the crimes of attempt and solicitation, the offense of conspiracy does not merge into the attempted or completed offense that was the object of the conspiracy. [Callanan v. United States, 364 U.S. 587, 59394 (1961)] [2] Model Penal Code The Code merges a conspiracy with the object of the conspiracy or an attempt to commit the target offense, unless the prosecution proves that the conspiracy involved the commission of additional offenses not yet committed or attempted. [MPC 1.07(1)(b)] 22.05 Defenses [A] Impossibility [1] Common Law The majority, but not universal, rule is that neither factual impossibility nor legal impossibility is a defense to a criminal conspiracy. [2] Model Penal Code The Model Penal Code does not recognize a defense of factual or hybrid legal impossibility in conspiracy cases. [MPC 5.03(1)]

[B] Abandonment [1] Common Law The crime of conspiracy is complete the moment the agreement is formed or, in some jurisdictions, once an overt act is committed in furtherance of a criminal objective. However, if a person withdraws from a conspiracy, he may avoid liability for subsequent crimes committed in furtherance of the conspiracy by his former co-conspirators if he communicated his withdrawal to each co-conspirator. [2] Model Penal Code The Model Codes abandonment defense to the crime of conspiracy is more onerous than that of the common law as it requires the conspirator to not only renounce his criminal purpose but to also thwart the success of the conspiracy under circumstances demonstrating a complete and voluntary renunciation of his criminal intent. [C] Whartons Rule An agreement by two persons to commit an offense that by definition requires the voluntary concerted criminal participation of two persons e.g., adultery, bigamy, incest, receipt of a bribe cannot be prosecuted as a conspiracy. Whartons Rule does not apply if more than the minimum number of persons necessary to commit an offense agree to commit the crime or if the two persons involved in the conspiracy are not the two people involved in committing the substantive offense. The Model Penal Code does not recognize Whartons Rule. [D] Legislative-Exemption Rule [1] Common Law A person may not be convicted of conspiracy to violate an offense if his conviction would frustrate a legislative purpose to exempt him from prosecution for the substantive crime. [2] Model Penal Code Unless the legislature otherwise provides, a person may not be prosecuted for conspiracy to commit a crime under the Model Code if he would not be guilty of the consummated substantive offense: (1) under the law defining the crime; or (2) as an accomplice in its commission. A person is not guilty as an accomplice in the commission of an offense if he was the victim of the prohibited conduct, or if his conduct was inevitably incident to its commission. [MPC 2.06(6)(a)(b)] Chapter 23 COMPLICITY 23.01 General Principles [A] Common law One is an accomplice in the commission of an offense if he intentionally assists another to engage in the conduct that constitutes the crime. Accomplice activity may include aiding, abetting, encouraging, soliciting, advising, and procuring the commission of the offense. Accomplice liability is derivative in nature. In general, the accomplice may be convicted of any offense committed by the primary party with the accomplices intentional assistance. Most jurisdictions extend liability to any other offense that was a natural and probable consequence of the crime solicited, aided or abetted. [B] Model Penal Code The Code rejects the common law natural-and-probable-consequences rule. Thus, an accomplice may only be held liable under the Code for acts that he purposefully commits. 23.02 Parties to the Complicity [A] Principal in the First Degree A principal in the first degree is one who, with the mens rea required for the commission of the offense: (1) physically commits the acts that constitute the offense; or (2) commits the offense by use of an innocent instrumentality or innocent human agent. The innocent-instrumentality rule provides that a person is the principal in the first degree if, with the mens rea required for the commission of the offense, he uses a non-human agent (e.g., a trained dog) or a non-culpable human agent to commit the crime.

[B] Principal in the Second Degree A principal in the second degree is one who intentionally assisted in the commission of a crime in the presence, either actual or constructive, of the principal in the first degree. A person is constructively present if he is situated in a position to assist the principal in the first degree during the commission of the crime, e.g., serving as a lookout or getaway driver outside a bank that the principal in the first degree robs. [C] Accessory Before the Fact An accessory before the fact is one who is not actually or constructively present when the crime is committed; often such person solicits, counsels, or commands (short of coercing) the principal in the first degree to commit the offense. [D] Accessory After the Fact An accessory after the fact is one who, with knowledge of anothers guilt, intentionally assists him to avoid arrest, trial, or conviction The conduct of the accessory after the fact occurs after the completion of the crime. If an accomplice is involved prior to the completion, i.e., up to the point when the principal in the first degree has reached a place of temporary safety, the accomplice is in fact a principal in the second degree. Today, nearly all jurisdictions treat the offense of accessory after the fact as separate from, and often less serious than, the felony committed by the principal in the first degree. 23.03 Acts Giving Rise to Accomplice Liability [A] Common Law [1] Types of Assistance An accomplice is a person who, with the requisite mens rea, assists the primary party in committing an offense. Generally speaking, there are three basic types of assistance: (1) assistance by physical conduct (e.g., furnishing an instrumentality to commit an offense, casing the scene in advance, locking the door to keep an assault victim from escaping, or driving a getaway car from the scene of the crime); (2) assistance by psychological influence (e.g., incitement, solicitation, or encouragement); and (3) assistance by omission (if there exists a duty to act). A person is not an accomplice simply because he knowingly fails to prevent the commission of an offense, but such failure to act may serve as a critical factor in determining that he assisted by psychological influence. [2] Amount of Assistance Required A person is not an accomplice unless his conduct (or omission) in fact assists in the commission of the offense. However, the degree of aid or influence provided is immaterial; even trivial assistance suffices. Furthermore, a secondary party is accountable for the conduct of the primary party even if his assistance was causally unnecessary to the commission of the offense. [3] The Pinkerton Doctrine In Pinkerton v. United States, 328 U.S. 640 (1946), two parties conspired to violate certain provisions of the Internal Revenue Code and thereafter, while one co-conspirator was in prison for unrelated reasons, the other carried out the plan. Emerging from this case is the Pinkerton doctrine under which a party to a conspiracy is responsible for any criminal act committed by an associate if it: (1) falls within the scope of the conspiracy; or (2) is a foreseeable consequence of the unlawful agreement. [B] Model Penal Code A person is guilty of an offense if he commits it by his own conduct or by the conduct of another person for which he is legally accountable, or both. Accomplice liability is founded on: (1) Accountability through an innocent instrumentality The Code explicitly provides that the innocentinstrumentality doctrine applies only if one causes another to engage in the conduct in question. (2) Accomplice accountability One is an accomplice if, with the requisite mens rea, he solicits, aids, agrees to aid, or attempts to aid in the planning or commission of the offense, or has a legal duty to prevent the commission of the offense, but makes no effort to do so. [MPC 2.06(2), (3)(a)]

(3) Miscellaneous accountability Legislatures may enact special laws of accomplice liability, e.g., prohibiting the aiding and abetting of a suicide attempt, [MPC 210.5(2)] or criminalizing the knowing facilitation of a prison escape. [MPC 242.6] The Model Code rejects the Pinkerton doctrine of conspiratorial liability. Thus, a person is not accountable for the conduct of another solely because he conspired with that person to commit an offense. The liability of one who does not personally commit an offense must be based on accountability through an innocent instrumentality, accomplice accountability, or miscellaneous accountability. 23.04 Mens Rea in Complicity Offenses [A] Common and Statutory Law [1] Intent The mens rea of accomplice liability is usually described in terms of intention. As with the crime of conspiracy, however, there is considerable debate regarding whether a person may properly be characterized as an accomplice if he knows that his assistance will aid in a crime, but he lacks the purpose that the crime be committed. Most courts, however, hold that a person is not an accomplice in the commission of an offense unless he shares the criminal intent with the principal. [2] Recklessness and Negligence Although courts and statutes frequently express the culpability requirement for accomplice liability in terms of intent, the majority rule is that accomplice liability may nevertheless attach in cases of crimes involving recklessness or negligence. [B] Model Penal Code The Code person resolves the common law ambiguity as to whether complicity requires purpose or mere knowledge of the consequences of their conduct. Under the Code, accomplice liability exists only if one assists with the purpose of promoting or facilitating the commission of the offense. [MPC 2.06(3)(a)] Accomplice liability may also be found in cases involving recklessness or negligence when causing a particular result is an element of a crime: (1) he was an accomplice in the conduct that caused the result; and (2) he acted with the culpability, if any, regarding the result that is sufficient for commission of the offense. [MPC 2.06(4)] 23.05 Liability of the Secondary Party In Relation to the Primary Party [A] Common Law At common law, an accessory could not be convicted of the crime in which he assisted until the principal was convicted and, with the limited exception of criminal homicide, could not be convicted of a more serious offense or degree of offense than that of which the principal was convicted. [B] Modern Rule Today, the majority rule is that a conviction (or even a prosecution) of the principal in the first degree is not a prerequisite to the conviction of a secondary party. Non-prosecution of the principal might result from any one of numerous factors extraneous to his guilt (e.g., death, flight from the jurisdiction, or immunity from prosecution), and thus, does not in itself prove that a crime did not occur. Furthermore, even if the principal is prosecuted but acquitted on the basis of an excuse defense, his acquittal should not bar a prosecution and conviction of a secondary party to whom the excuse does not extend. An acquittal on the ground of an excuse means that the actions of the primary party were wrongful, but that he was not responsible for them because of the excusing condition. However, since accomplice liability is derivative, there must be proof at the accomplices trial of the principals guilt. An accomplice or accessory may be convicted of a more serious offense than is proved against the primary party. [C] Model Penal Code An accomplice in the commission of an offense may be convicted of a crime, upon proof of its commission by another person, regardless of whether the other person is convicted, acquitted, or

prosecuted. Furthermore, an accomplice may be convicted of a different offense or different degree of offense than is the primary party. [MPC 2.06(7)] The Code also expressly provides that a person who is legally incapable of committing an offense personally may be held accountable for the crime if it is committed by another person for whom he is legally accountable. [MPC 2.06(5)] 23.06 Limits to Accomplice Liability [A] Common Law [1] Legislative-Exemption Rule A person may not be prosecuted as an accomplice in the commission of a crime if he is a member of the class of persons for whom the statute prohibiting the conduct was enacted to protect. For example, in a case of statutory rape, an underage female who engages in sexual intercourse cannot be prosecuted as a secondary party to her own statutory rape since the law was enacted to protect young females from immature decisions regarding sex. [2] Abandonment As with the law of conspiracy, many courts hold that a person who provides assistance to another for the purpose of promoting or facilitating the offense, but who subsequently abandons the criminal endeavor, can avoid accountability for the subsequent criminal acts of the primary party. The accomplice must do more than spontaneously and silently withdraw from the criminal activity. He must communicate his withdrawal to the principal and attempt to neutralize the effect of his prior assistance. [B] Model Penal Code A person is not an accomplice in the commission of an offense if: (1) he is the victim of the offense; or (2) his conduct is inevitably incident to the commission of the offense; or (3) he terminates his participation before the crime is committed, and he: (a) neutralizes his assistance; (b) gives timely warning to the police of the impending offense; or (c) attempts to prevent the commission of the crime.

You might also like